ASCP MT

Ace your homework & exams now with Quizwiz!

neonatal, brain trauma

"Fresh" blood should be given to patients in these groups:

Urinary cast

"Urinary cast"

Ammonium Biurate

"thorny apple" can indicate liver issues

Concentration diluted unknown =

(A diluted unk/ A standard) X Concentration standard X dilution factor

Concentration unknown =

(A unk/ A standard) X Concentration standard

% Amount renal function remaining =

(Createnine clearance mL/min / 130 mL/min) X 100

Creatinine Clearence Formula

([U creatinine]/ [P createnine]) X (Volume of Urine/ 1440 min per day) X (1.73/ BSA)

concentration of unknown

(absorbance of unknown/absorbance of standard) * concentration of standard =

Sodium

(mEq/L = mMol/L) Adult Plasma 136-146 mMol/L

BE

+/- 3 mMol/L

-megaloblastic anemia -hypersegmented PMN very typical of MB anemia -can be due to folate or B12 deficiency (disrupts DNA synthesis)

-what anemia is this morphology indicative of? -what is the large, basophilic cell at the top right corner, and why is it important? -what can cause this anemia?

Plasma Lactic Acid (Lactate)

0.5 to 1.9 mMol/L

30

1 fluid ounce = ____ milliliters

One bag of red blood cells will increase the hemoglobin by what amount

1 gm/dL

28

1 ounce = ____ grams

454

1 pound = _____ grams

0.95

1 quart = ____ liters

At what temperature should the patient's sample and the segment from the bag of blood be stored

1 to 6 degrees Celsius

Serum Calcium Ionized

1.11 to 1.30 mMol/L

Serum Magnesium

1.6 to 2.3 mg/dL 0.65 to 1.05 mMol/L

Calculated Osmolality Formula

1.86X[Na] + ([glucose mg/dL]/ 18) + ([BUN mg/dL]/ 2.8)

What is the upper age limit at which a person can have had hepatitis and still be an allogeneic blood donor

10 years of age

BUN: Createnine Ratio

10:1 to 20:1

What is the minimum hemoglobin and hematocrit for autologous donations

11 gm/dL and 33%

What is the minimum weight to be an allogeneic blood donor

110 pounds

What is the minimum hemoglobin for allogeneic blood donations

12.5 g/dL

What percent cell suspension is used when testing cells for ABO antigens

4%

0.6

A 0.1 change in pH corresponds to a ____ mmol/L change in potassium.

15

A BUN/creatinine ratio of <____ mg/dL indicates intra-renal azotemia.

24

A BUN/creatinine ratio of >___ mg/dL indicates pre-renal azotemia.

10

A GFR <___ mL/min from creatinine clearance is considered end-stage renal failure.

>10 nm

A broad bandpass spectrophotometer has a bandpass of:

panic value

A result which is so abnormal as to signal a condition which requires immediate intervention is a(n):

Cytokine Pleiotropism

A single --- induces multiple different effects.

monosaccharide

A single simple sugar is termed a(n):

Which of the following cells is the atypical lymphocyte seen on the peripheral blood smear of patients with infectious mono? A. T lymphs B. B lymphs C. monos D. mast cells

A. T lymphs

140. Which nematode has a characteristic double-operculated egg? A. Trichuris trichuria B. Ascaris lumbricoides C. Necator americanus D. Ancylostoma duodenale E. Strongyloides stercoralis

A. T. Trichuris, the whipworm, has an extremely tapered anterior end and double-operculated egg. It may coinfect with Ascaris. QCCP2, Trichuris

An unexplained elevation of the prothrombin time (PT) in a 72 y/o smoker who has been diagnosed with chronic pulmonary obstructive disease is most likely due to: A. a eleveated Hct B. a decreased Hct C. vitamin K deficiency D. decreased thrombin activity

A. a eleveated Hct

Plasma from a patient with lupus coagulation inhibitor can show: A. a prolonged APTT and normal PT B. a prolonged thrombin time C. no change with plt neutralization D. complete correction when incubated with normal plasma

A. a prolonged APTT and normal PT

Suprabital staining is important retics since the cells must be living in order to stain the: A. remaning RNA in the cell B. iron before it precipitates C. cell membrane before it dries out D. denatured Hgb in the cell

A. remaning RNA in the cell

Costimulatory Surface Molecule

Accessory molecule, used alongside MHC, allows T Cell to recognize cell type, origin, and circumstances of presenting Cell. Induces Anergic response when absent.

Antibody Diversity

Achieved via expression and recombination of Germline DNA, unique to lymphocytes. Follows ribosomal path after transcription. Yields ~4 million variations, and can undergo frameshift and post-transcriptional mutations for arms-race evolution. Also occurs with T-Cell TCRa/b development.

hypoglycemia

Addison's disease causes:

FAS Receptor

Aka. CD95, second mechanism to induce apoptosis. Receptor present in membrane of all nucleated cells, binding via matching ligand causes extrinsic apoptosis.

Alveolar Macrophage

Aka. PAM, wandering --- in the lung, act to clean inhaled particles and pathogens from the lumen, may leave by migration up the trachea (if indigestible) or to lymph (if pathogen to perform antigen-presenting).

Pulmonary Intravascular Macrophage

Aka. PIM, fixed --- in the lung capillary lumen, acts to filter blood of pathogens.

hexokinase

An enzyme which is able to phosphorylate glucose is known as a(n):

erythropoietin

Anemia is common in renal failure because of deficiency of:

Receptor Antagonism

Another ligand binds first and does not induce a response, physically blocks regular ligand from binding.

picric acid

Another term for tri-nitro-phenol is:

Quiz Question: The Ration of Peptone to carbohydrate in OF medium is: A) 1:15 B) 1:5 C) 5:1 D) 15:1

Answer: B) 1:5

Quiz Question: This organism is found is marine water but can also find its way into meats and other grocery products. It causes wound infection,gastroenteritis and bacteremia. It is oxidase and indole positive, and motile. One of its virulence factors is the production of aerolysin. A) Vibrio parahaemolyticus B) Campylobacter jejuni C) Vibrio vulnificus D) Aeromonas hydrophilia

Answer: D) Aeromonas hydrophilia

Quiz Question: A Tech reported a fruity odor and did an oxidase test which was positive. the Tech should suspect which organism? A) Pseudomonas flourescens B) Acinetobacter baumannei C) Stenotrophomonas multophilia D) Pseudomonas aeruginosa

Answer: D) Pseudomonas aeruginosa

Quiz Question: A nonfermenter should be suspected for all of the following except: A) Oxidase Positive B) Growth is better on Blood agar (SBA) C) TSI : K/K D) TSI : K/A + H2s Production

Answer: D) TSI: K/A + H2S Production

Quiz Question: Due to the increase peptones in TSI agar the small amount of acid produced by a nonfermenter is masked by alkaline reactions from peptone utilization, this is why OF medium is used. (TRUE or FALSE)

Answer: TRUE

Which of the following antibodies are implicated in delayed hemolytic transfusion reactions

Anti-A1, anti-P1, anti-Jk(a)

If a patient is blood type A, which of the following antibodies are in his serum/plasma

Anti-B

decrease

As pO2 decreases, O2Sat will ______, and vice versa.

pre-renal

Azotemia which is caused by a factor prior to the kidneys/urinary tract is classified as:

intra-renal

Azotemia which is caused by a problem with the kidneys themselves is classified as:

post-renal

Azotemia which is caused by a problem with the urinary tract is classified as:

beer's law

Aλ = abc is:

Affinity Maturation

B-Cells alter their Hypervariable Region sequences as the immune response progresses to improve ---.

Normal plts have a circulating life-span of approximately: A. 5 days B. 10 days C. 20 days D. 30 days

B. 10 days

Refer to the following illustration: Which area of the automated cell counter histogram indicates the nonlymphocytic curve? A. B B. C C. D D. E

B. C

178. Which organism is the cause of adiaspiromycosis? A. Fusarium B. Chrysosporium C. Beuveria D. Paecilomyces E. Gliocladium

B. C. Adiaspiromycosis is a benign granulomatous infection with characteristic large thick-walled spherules. QCCP2,Conidia occurring singly

The disease most frequently present in patients with atypical lymphocytosis and persistently negative test is: A. toxoplasmosis B. CMV infection C. herpes virus infection D. viral hepatitis

B. CMV infection

The smear represented below displays: A. congenital ovalocytosis B. Hgb C disease C. poor RBC fixation D. delay in smear prep

B. Hgb C disease

Which of the following stains can be used to differentiate siderotic granules (Pappenheimer bodies) from basophilic stippling? A. Wright B. Prussian blue C. crystal violet D. PAS

B. Prussian blue

Which of the following coagulation factors is considered to be labile? A. II B. V C. VII D. X

B. V

The term "shift to the left" refers to: A. a microscopic adjustment B. immature cell forms in the peripheral blood C. a trend on a Levy-Jennings chart D. a calibration adjustment on an instrument

B. immature cell forms in the peripheral blood

nomogram

BSA is determined from height and weight using a(n):

blood buffer system

Bicarbonate is the salt of the:

exogenous insulin

C-peptide can be used to determine how much insulin a patient is making because it is not affected by the administration of:

longer

C-peptide has a ____ half life in the body compared to insulin.

A patient has a normal prothrombin time and a prolonged activated partial thromboplastin time (APTT) using a kaolin activator. The APTT corrects to normal when the incubation time is increased. These results suggest that the patient has: A. hemophilia A (Factor VIII deficiency) B. Hegeman Factor (XII) deficiency C. Fletcher Factor deficiency (prekallikrein) D. Factor V deficiency

C. Fletcher Factor deficiency (prekallikrein)

Which of the following is the formula for mean corpuscular volume (MCV)? A. (Hgb x 10)/RBC B. Hgb/Hct C. (Hct x 10)/RBC D. RBC/Hct

C. Hct x 10)/RBC

Which of the following factors is used only in the extrinsic coagulation pathway? A. II B. V C. VII D. VIII

C. VII

carbonic acid

CO2 combines with water to form a small amount of:

pH

CO2 is measured by watching for an decrease in _____ within the electrode.

2/3

CSF glucose should be _____ of plasma glucose.

Immature Lymphocytes

Circulate from blood -> node -> Thoracic Duct. Bind addressins present in High Endothelia (upregulated with DAMP and inflam).

milliliters of blood filtered every minute

Creatinine clearance measures:

Which of the following is the formula for absolute cell counts? A. number of cells counted/total count B. total count/number of cells counted C. 10x total count D. % of cells counted x total count

D. % of cells counted x total count

Which of the following is the formula for mean corpuscular hemoglobin (MCH)? A. Hct/(RBC x 1000) B. Hgb/Hct C. RBC/Hct D. (Hgb x 10)/RBC

D. (Hgb x 10)/RBC

Which curve represents the production of beta polypeptide chains of hemoglobin? A. B B. C C. E D. D

D. D

Which curve represents the production of delta polypeptide chains of hemoglobin? A. B B. C C. D D. E

D. E

A 4 y/o boy presents with chronic ear infections and is on prophylactic antibiotics. He presents with bleeding. Factor assays reveal: (see picture) Possible causes are: A. Factor II deficiency B. lupus anticoagulant C. hemophilia D. Vitamin K deficiency

D. Vitamin K deficiency

Hematology standards include: A. stabilized RBC suspension B. latex particles C. stabilized avian RBCs D. certified cyanmethemoglobin solution

D. certified cyanmethemoglobin solution

Langerhans Cell

Dendritic Cells within the epidermis.

Adaptive Immunity

Develops after antigen exposure (days-weeks), specific, memory and tolerance, activates and is initially activated only by Innate Immunity. Includes Antibodies, T-Cells (helper, cytotoxic, gd) and B-Cells.

renal, vascular, neurological

Diabetes complications most commonly affect these organ symptoms:

Glucose in the urine is indicative of

Diabetes mellitus (false+ seen in Vitamin C)

gestational diabetes

Diabetes which is diagnosed for the first time during pregnancy and disappears after delivery is known as:

Ketones in the urine is indicative of

Diabetic ketoacidosis or starvation

ferrous

Dye binding assays will only detect ______ iron, so ascorbic acid and sodium metasulfite are included in the reagent to reduce iron.

personal protective devices

Equipment that is worn and provides a physical barrier against chemicals and pathogens, such as gloves, fluid resistant lab coats, and glasses, are collectively termed:

serum iron, TIBC, transferrin saturation

Evaluation of ________ is necessary to diagnose iron deficiency or toxicity.

7.4

Even when a condition is fully compensated, pH go "past" ____ in the other direction.

Cortical Thymus Epithelia

Express full range of their DNA and both MHC receptors to test self-recognition in T0 Cells, can include fetal pathogens.

Cryoprecipitate Antihemophilic Factor that is harvested from fresh frozen plasma can be used to treat which coagulation factor deficiencies

Factor I Factor VIII Factor XIII

GTT Normal Results

Fasting 70 to 110 mg/dL 1/2 Hr Less than 170 mg/dL 1 Hr Less than 170 mg/dL 2 Hr Under 110 mg/dL 3 Hr Fasting level 4, 5 Hr Fasting level

What portion of the IgG molecule functions to move the molecule across the placenta

Fc portion

3+

Ferric iron has this charge:

IgM Function

First and rapid response by newly activated B-Cells, for complement activation and agglutination, stays in circulation.

Spleen

Fully functional lymphoid organ, filters blood instead of lymph, uses venous sinuses instead of capillaries, hemorrhages when damaged.

glycosylated hemoglobin

Glucose in the bloodstream binds to hemoglobin at a rate which is dependent on glucose concentration, which is the basis of this test:

glycogenesis

Glucose is stored in the liver as glycogen in this process:

rising

Glucose level should be _____ by the 1/2 hour of the 5HGTT.

peaking

Glucose level should be ______ by the 1st hour of the 5HGTT.

-Greater than 95% normal adult hemoglobin -2 alpha and 2 beta chains (α2β2)

Hb A

-About 2% normal adult hemoglobin -2 alpha and 2 delta chains (α2δ2) -elevated in β-thalassemia.

Hb A2

-4 gamma chains (ɣ4) -lack all 4 alpha globin genes -100% of Hb in homozygous α-thalassemia -hydrops fetalis; not compatible with life because biologically useless

Hb Barts

Fc

Heavy chain tail of antibody, imparts biological activity (complement activation, opsonization, and ADCC).

hemolytic anemia, megaloblastic anemia, heme synthesis blockage, severe hepatitis, iron intoxication

High serum iron is correlated with these conditions:

TH1 Cytokines

IL2, IFNg, TNF. Stimulate Macrophages, Neutrophils, NK Cells, Memory B Cells, etc.

Tripple phosphate crystal

Identify

1

If an employee is aware of a breach, he or she must inform his or her supervisor within ____ business day(s) or be liable as an accessory.

low sodium, high chloride

If analytical error is causing abnormally low anion gaps, this pattern should be seen:

rejected

If both levels of QC exceed 2σ from the mean, it should be:

potential interference OR analyte

In a bichromatic analysis, the secondary λ should be one which is absorbed by:

glucose

In a fasting patient, only ______ should be found in the bloodstream.

7.35-7.4

In a fully compensated condition, a pH of ____-____ indicates acidosis.

absorbance

In the equation Aλ = abc, A is:

Monomeric IgA

Isotope of IgA that circulates in the blood.

What is the characteristic of a clinically significant antibody

It is IgG.

0.1 C

It is imperative that the temperature of an electrode be controlled to within:

If the Direct antiglobulin test is positive due to IgG, how will this look microscopically and what test must be done to identify the IgG antibody on the cells

It will look mixed field microscopically and the IgG antibody must be eluted from the cells. Test this eluate against a panel to identify the antibody.

Which is a low frequency antigen

Js(a)

Dioctophyma Renale

Kidney Worm

Red Blood Cells in the urine is indicative of

Kidney trauma, stones, nephritis, or cancer

cathode, diodes, anode

Light enters a photomultiplier and hits a(n) ______ then is bounced between several ______, liberating electrons each time, then is collected by a(n) ______.

stray light

Light that doesn't pass through the specimen but hits the photomultiplier is known as:

absorbed

Light which strikes an object and does not pass through it is said to be:

24-Hour Urine Creatinine

Males 800 to 1800 mg/dL Females 600 to 1600 mg/dL

Creatinine Clearance (GFR)

Males 97 to 137 mL/min Females 88 to 128 mL/min

glucose

Maltose is a dissacharide composed of:

respiratory acidosis

Metabolic alkalosis may occur as compensation for:

Eosinophil Function

Migrate beneath epithelia to pursue and fight parasites traveling through tissues (not gut lumen), kill large pathogens via surface degranulation. Can also function like less efficient Neutrophils.

dye binding reagents

Most iron assays use:

TH2 Stimulation

NK Cells release IL4.

eGFR

NKDEP now recommends the inclusion of _____ with every serum creatinine result.

Anion Gap Calculation

Na - (Cl + Bicarb)

Lymph Node Circulation

Naive B and T Cells enter and cross endothelium, may continue if not activated.

TH0

Naive cell, stimulation by Dendritic or NK Cell cytokines cause varying differentiation. Occurs in lymph nodes as they travel via blood -> node -> Thoracic Duct.

cuvette

Name this part of a spectrophotometer.

light source

Name this part of a spectrophotometer.

monochromator

Name this part of a spectrophotometer.

spectrophotometric accuracy

Objects of known absorbance are run through a spectrophotometer to test:

Urobilinogen - absent is indicative of

Obstructive jaundice

post-hepatic jaundice

Obstructive jaundice is also known as:

Anamnestic Response

Occurs after initial exposure to a pathogen, cells have expanded and matured, antibodies bind tighter and titer is high, more rapid and effective.

adulthood

Onset of acromegaly is in:

Transitional Epithelium

Originates in bladder, or 2/3 of urethra Twice as big as a WBC Always note clumping or atypical cells.- very important.

hyperglycemia

Pheochromocytomas cause:

fiske-subbarow / molybdenium blue

Phosphate is most typically measured with this reaction:

phosphate buffer system

Phosphates are part of the body's:

Which component must be stored with constant shaking

Platelets

blockage of the urinary tract

Post-renal azotemia results from:

KCl

Potassium ISEs contain a solution of:

electrolytes

Potentiometric assays are especially useful for quantifying:

Lung Filtering

Predominant in cat, pig, calf, and sheep. Performed by PIMs.

1 hr, 3 hr

Pregnant patients should be screened for gestational diabetes with a _____ GTT, abnormal results should be followed up with a ____ GTT.

Immature Dendritic Cell

Present in tissues to ingest and process antigens, little to no presenting. High FcR, low MHC and cytokine levels.

Thymus

Primary lymphoid tissue where T-Cells mature, T0 enter at Cortex and progress through to Medulla, leave as gd, TH0 or TC0 (inactive).

IL-1

Pro-inflammatory cytokine, mobilizes amino acids via muscle breakdown, causes malaise, upregulates Integrins, fibroblasts and structural cells (healing), T-Helper Cells, WBC degranulation, and activates Sentinels. Can also kill neoplastic cells.

Nitrous Oxide

Pro-inflammatory released by M1 cells, synthesized from Arginine with an activated enzyme.

resistance

Purity of water is measured by the level of _____ to transmission of an electric current.

Urine/Serum Osmolality Ratio

Random: 1.0 to 3.0 12 hr fluid restriction: > 3.0

RIG

Receptor type in cytoplasm, detect bacterial R/DNA, useful for escaped phagocytosed microbes.

NOD

Receptor type, like RIG but for peptides.

TLR

Receptor type, turned inward within phagosomes, used to detect factors released by dead/dying phagocytosed pathogens, recognize free particles only.

Pattern Recognition Receptors

Recognize D/PAMPs, may be Soluble (for complement system), Vesicular (phagosome membrane), Cytoplasmic, or Membrane-bound.

Type 2 Interferons

Released by activated TH1 memory cells, includes IFNg. Antiviral effects, but can be stimulated by whatever antigen binds a TH1.

IL2

Released by stimulated T Cells, activates self and causes activation of nearby cells only with matching receptor present (via stimulation). Causes clonal expansion.

If a patient's sample is positive with all cells (screens, auto, and crossmatches) after 37oC incubation, but negative after the addition of AHG, what could be the cause

Rouleaux

testing to compliance

Running failed QC many times to try to get an in-range value is termed:

yes

Running many standards of caffeine concentration for a specific methodology gives this chart. Is MLS Student Caffeine Concentration a linear method?

your value, peer value, peer standard deviation

SDI = (____-_____)/____

nephrotic syndrome

Sediments

Monocyte

Slightly rare, circulatory life is 1-2 days, migrate into tissues to become Macrophages.

microbiology, blood bank, immunology

Soda lime glass may be used for these departments of the laboratory:

NaCl

Sodium ISEs contain a solution of:

Effector Antibodies

Soluble --- produced by Plasma Cells.

Antibody Character

Solution binds to equilibrium and behaves like any chemical reaction (higher rate with temp, diffusion behavior, etc). Affected by pH and ion character of surroundings.

ketones, lactates, salicylates, formates, oxalates

Some examples of common anions which cause abnormal anion gaps are:

accession number

Some, but not all, laboratories consider this PHI:

very small concentrations

Specifically, polyolefin plastics are unsuitable for analysis of proteins at:

acidic

Stored blood bank blood can become so ______ that the enzymes which make 2,3-DPG are inhibited.

triiodothyronine

T3 is shorthand for this thyroid hormone:

thyroxine

T4 is shorthand for this thyroid hormone:

calculation

TCO2 is determined by:

1.11

The assayed glucose concentration of a sample of plasma will be about ____ times greater than a sample of whole blood from the same patient.

10-12

The assayed glucose concentration of a sample of whole blood will be ___-___% lower than that of plasma from the same patient.

lipolysis

The breakdown of lipids for use as energy is:

calomel, potassium salt bridge

The cathode of a pH electrode is used as a standard and is made from _____ with a ______.

calomel

The cathode of a pH electrode usually contains this chemical:

hydrogen gas

The cathode of a pO2 electrode produces:

NH3

The chemical formula of ammonia is:

beta-d-galactosidase

The deficient enzyme in lactose intolerance is:

reference range

The defined set of expected values found among non-diseased people for a given test is its:

age at onset

The difference between osteomalacia and ricketts is:

high, high

The hepatocellular pattern for serum/plasma is _____ tBili and _____ dBili.

lower

The higher the λ, the ____ the energy.

hyperventillation

The only cause of primary respiratory alkalosis (not as a compensation mechanism) is:

oxygen permeable rubber

The outer covering of a pO2 electrode is:

HCl

The pH electrode in a blood gas analyzer is a sealed anode which contains:

respiration, metabolism

The pH of blood is regulated by these functions:

6.103

The pKa of carbonic acid is:

50

The pO2(50) is a measure of how much oxygen is required to saturate fully desaturated blood to ___%.

cuvette

The part of a spectrophotometer which holds the sample to be tested is the:

light source

The part of the spectrophotometer which provides the incident light is the:

monochromator

The part of the spectrophotometer which splits the incident light into different λ is the:

pos, neg

The post-hepatic pattern for urine is _____ bilirubin and _____ urobilinogen.

absorbance change per unit

The reaction rate is defined in the context of kinetic assays as:

Which of the following are essential criteria when transfusing the unborn child

The red blood cells must be irradiated The red blood cells must be as fresh as possible (Usually no more than 5 days old) The cells must be negative for hemoglobin S.

atomic absorption spectroscopy

The reference method for calcium and magnesium is:

0.03

The solubility coefficient of CO2 in water is:

platelets

The source of excess potassium in serum is:

heparinized plasma, serum

The specimens of choice for a phosphate assay are:

heparinized plasma, serum

The specimens of choice for a total magnesium or total calcium assay are:

serum, heparinized plasma

The specimens of choice for an Iron/TIBC assay are:

serum, heparinized whole blood

The specimens of choice for an ionized magnesium assay are:

1 cm

The standard depth of a cuvette is:

EDTA or potassium oxalate

These anticoagulants are most frequently the source of absurdly high potassium results:

group a strep, viral hepatitis

These are the main causes of glomerulonephritis:

nessler, berthelot, glutamic dehydrogenase

These are the main methodologies for ammonia:

elevated indirect bilirubin, positive urine urobilinogen

These are the only significant test results of Gilbert's syndrome and type II Crigler Najjar Syndrome:

kymex, pyrex

These are trademarked names for special tempered borosilicate glass which does not shatter when exposed to extreme temperatures:

hepatic necrosis, acute hemolytic anemia, acute iron poisoning

These conditions are correlated with high iron and high TIBC:

chronic hemolytic anemia, chronic iron poisoning

These conditions are correlated with high iron and low TIBC:

bicarb, calcium

These electrolytes are decreased in uremic syndrome:

acidosis, fever

These non-hematological conditions will cause a high pO2(50) as a side effect:

hypothermia, alkalosis

These non-hematological conditions will cause a low pO2(50) as a side effect:

henry-caraway phosphotungstate reaction, uricase

These reactions are used to measure uric acid:

USP, NP

These reagent grades are of unknown purity and impurities are not identified but are guaranteed to not interfere with the function of the chemical:

purified, practical, commercial, technical

These reagent grades are of unknown purity and impurity and give no guarantees as to function:

wavelength calibration, photometric accuracy

These should be tested as part of daily maintenance:

hypoglycemia

This 5HGTT curve indicates:

impaired glucose tolerance

This 5HGTT curve indicates:

post prandial glucose tolerance test

This test uses a meal as a replacement for the glucose drink used in oral glucose tolerance tests:

urine, sweat

Two samples which are typically tested using a chloridometer are:

100

Urine BUN should be at least ____ times higher than serum BUN.

100

Urine creatinine should be at least ____ times higher than serum creatinine.

the kidney's ability to concentrate

Urine osmolality is a measure of:

Pyelonephritis

Urine sediment (WBC cast)

130

Urologists consider a ____ mL/min GFR to represent 100% renal function.

Fish Passive Transfer

Use IgM, method depends on birth mechanism by species - egg, mouth brooding, or live.

Lysozymes

Used by Neutrophils, act to dissolve G+ bacterial cell walls.

Complement System

Uses plasma and cell surface proteins, rapid and exponential amplification cascade. Categories are Classical, Lectin, and Alternative, all resulting in a common Terminal Pathway. Results in pathogen lysis, opsonization, increases vascular permeability, and directs WBC chemotaxis to site.

Agar Immunodiffusion

Uses wells punched into --- and components in separate wells, forms visible lines midway as positive result. Type of Precipitation Assay.

Serosal Macrophage

Wandering --- along the pleural and peritoneal surfaces, removes dead cells and pathogens.

If a patient has anaphylactic transfusion reactions, how are future transfusions handled

Wash the cellular products to be infused.

If a patient's serum reacts with the diluent in the commercial cells, what can be done to resolve the problem

Wash the commercial cells to remove the diluent and retest Perform the antibody screen with the cells from another manufacturer Suspend the patient's cells in the commercial diluent that accompanies each panel and retest. If this is positive, it proves that the diluent is the problem.

tap water

Water that comes out of a faucet and is not filtered before use is:

deionized

Water that is run through special anion/cation columns to remove charged molecules is:

Waxy Cast

Waxy casts indicate chronic tubular degeneration and must be distinguished from hyaline casts.

cuvette

We use blanks to zero a spectrophotometer to cancel out the absorbance of the:

-CD 34 -CD 31 -Kit

What are the cell markers for Hematopoietic stem cells?

reflectance

What concept is illustrated by line A in this diagram of red and green light striking a red object?

transmittance

What concept is illustrated by line B in this diagram of red and green light striking a red object?

absorbance

What concept is illustrated by line C in this diagram of red and green light striking a red object?

increase

When blood pH is normal, ionized calcium will _____ as total calcium increases.

urine, serum

When calculating creatinine clearance, ____ creatinine is divided by ____ creatinine.

1.73

When calculating creatinine clearance, ____ is divided by BSA.

1440

When calculating creatinine clearance, urine volume is divided by:

hepatocellular

_____ jaundice is the result of conditions which affect the enzymatic activity of the liver or its ability to secrete waste products.

obstructive

_____ jaundice is the result of conditions which damage or occlude the biliary tract, preventing transport of metabolites from the liver to the intestines.

naturally fluorescent chemicals

______ can be present in serum and may interfere with fluorescence assays:

accuracy

______ is the ability to get correct values.

precision

______ is the ability to get the same value over and over.

sucrose

______ is the only non-reducing disaccharide.

acanthocyte

abnormal red blood cell with spiny projections ("spur cell")

Leucine Crystal

acidic

Primary Lymphoid Tissue

aka Bursa, site where B-Cells go to mature after release from marrow, location varies by species. Cells are destroyed if they bind while inside (to body proteins).

Antibody

aka Humoral Immunity, produced by Plasma Cells, held together by disulfide bonds, 110 AA per domain, IgG, IgA, IgE, and IgD (named for Heavy Chain type). Most effective type varies by pathogen, bacteria may use shedding slime layer to avoid.

Neutrophil Extracellular Trap

aka. NET, occurs when cell is unable to phagocytose a target, dissolves DNA and nucleus before breaking open and forming a sticky histone-enzyme net, active process (Netosis).

If the antibody screen is positive after the addition of AHG and the autocontrol is negative after the addition of AHG, this means that

an alloantibody is present.

reticulocyte

an immature RBC with a network of precipitated basophilic material in the cytoplasm (beaded chains of ribosomes)

-decreased RBC production: hematopoietic cell damage, deficiency of factors -increased RBC loss: external blood loss, RBC destruction (hemolysis)

causes of anemia

Uric acid crystals

common in Dalmatians - metabolic defect alkaline urine

Cystine

common in Newfoundlands acidic

ethylene glycol

crystal (star fruit ingestion)

-decrease in whole body red cell mass, RBC, MCV, Hb, or HCT

define anemia

spherocyte

dense, small, round RBCs lacking central pallor

A person can make an alloantibody as a result of

exposure to a foreign antigen through a blood transfusion exposure to a foreign antigen through pregnancy exposure to a foreign antigen in the environment

schistocyte

fragmented RBCs usually formed as a result of shearing of the RBC by intravascular trauma. wedge-shaped

The procedure used to detect and identify clinically significant antibodies is the

indirect antiglobulin test

eGFR expressed as

mL/minute/1.73 m2

-acute blood loss -decreased RBC production (stem cell or progenitor defects) -ineffective hematopoesis -increased destruction of RBCs (after release from BM, either intra or extracorpuscular)

name the pathophysiological categories for causes of anemia

Urticarial transfusion reactions have no definitive cause, but it is thought IgE is involved. What are the symptoms

pruritus hives erythema

If a cell is positive for the c and e antigens and negative for the D, C and E antigens, what is the most probable Rh phenotype

rr

-symptomless carrier state resulting from heterozygous inheritance of thalassemic variants in beta globin gene -anemia is mild or absent -RBCs are hypochromic and microcytic with a low MCV -normal serum ferritin and iron stores -often raise Hb A2 and Hb F -no treatment necessary; iron contraindicated

thalassemia minor (trait)

-long-term folic acid supplements -regular transfusions -splenectomy sometimes helps -deal with iron overload by giving chelators -bone marrow transplant is the only curative treatment

treatment of thalassemia major

Commercially prepared red blood cells for the purpose of detecting unexpected antibodies in serum/plasma are

typed for 18 common antigens

anisocytosis

variation in the size of red blood cells

-no α chain synthesis and only Hb Barts (ɣ4) is present; not compatible with life since Hb Barts cannot carry oxygen -infants are stillborn or die soon after birth -hydrops fetalis

α-thalassemia four-gene deletion

-blood is usually normal

α-thalassemia single-gene deletion

% Amount renal function lost =

100 - [(Createnine clearance mL/min / 130 mL/min) X 100]

If a dad is R1R2 and the mom is rr, what percent of the offspring will be positive for the D antigen

100%

100% renal function equals

130 mL minute

Urea

15 to 38 mg/dL

What percent of the caucasian population is negative for the D antigen

15%

eGFR=

186 X(P creat)to the -1.154 power X (Age)to the -0.203 power X 0.742 female (or 1.0 male) X 1.210 black (1.0 white)

decrease

2,3-DPG serves to ______ the oxygen binding affinity of hemoglobin.

TCO2

22 to 29 mMol/L

If a bag of leukoreduced red blood cells is washed, how long is this product good

24 hours from the start of the washing procedure.

Transferrin Iron Saturation

25% - 55%

Serum TIBC

250 to 450 ug/dL

Serum Osmolality

289 to 308 mOsm/Kg

body surface area

BSA is short for:

False positive results and false negative results can be obtained by sloppy technique. Which of the following causes a false positive result

Bacterial contamination of the reagent

Gram Positive

Bacterial type, cell wall has an inner membrane and a slimy peptidoglycan outer capsule.

Acid-Fast

Bacterial type, has an inner membrane and a thick mycolic acid wax capsule.

95

Bicarbonate is usually about ___% of the total CO2.

400, 530

Bilirubin absorbs light between _____ and ____ nm.

unconjugated

Bilirubin released into the bloodstream from the breakdown of erythrocytes is of this type:

low

Blood pH is typically _____ in renal disease.

Chronic kidney disease

Broad cast

132. Which species of Plasmodium is responsible for quartan (72-hour) fevers? A. P. falciparum B. P. vivax C. P. malariae D. A & B E. A, B, C

C. P. . P. malariae differs from the other species of Plasmodium by having 72 hour or quartan fever spikes rather than the 48 hour tertian fevers. P. malariae can also have nephrotic syndrome and prefers to infect older red blood cells. QCCP2, Plasmodium spp

The characteristic morphologic feature in lead poisoning is: A. macrocytosis B. target cells (codocytes) C. basophilic stippling D. rouleaux formation

C. basophilic stippling

On an electronic particle counter, if the RBC is erroneously increased, how will other parameters be affected? A. increased MCHC B. increased Hgb C. decreased MCH D. increased MCV

C. decreased MCH

Cells that provide antibodies and lymphokines are: A. erythrocytes B. granulocytes C. lymphocytes D. thrombocytes

C. lymphocytes

In chronic meylocytic leukemia, blood histamine concentrations tend to reflect the: A. number of plts present B. serum uric acid concentrations C. number of basos present D. the total number of granulocytes

C. number of basos present

What cell shape is MOST commonly associated with an increased MCHC? A. teardrop cells B. target cells C. spherocytes D. sickle cells

C. spherocytes

grade I

CAP and JC only allow the use of this grade of reagent water in labs they certify:

analytical grade

CAP only allows the use of _______ reagents in certified laboratories.

Fasting CSF glucose

CSF 40 to 70 mg/dL OR 2/3 of the blood level

Metabolic Acidosis

Caused by deficit of bicarbonate

Respiratory Alkalosis

Caused by edepression of pCO2 only

Respiratory Acidosis

Caused by elevation of pCO2 only

Metabolic Alkalosis

Caused by excess of bicarbonate

more

Cells have _____ electrolytes than blood.

polyolefin plastics

Collectively, polystyrene and polypropylene are known as:

C3b

Complement protein, does immune regulation, opsonization, and antigen trapping in lymph nodes (for B-Cell maturation).

thermocouple

Condensation in VPD osmometry is measured by a(n):

urine

Controls for a urine protein test must be made from:

With this blood picture, an addiontional test indicated is: A. alkali desaturation B. alkaline phosphate stain C. peroxidase stain D. Hgb electrophoresis

D. Hgb electrophoresis

The APTT: A. test the extrinsic coag pathway B. monitors Coumadin therapy C. requires tissue thromboplastin D. monitors heparin therapy

D. monitors heparin therapy

Which of the following is associated with Alder-Reilly inclusions? A. membrane defect of lysosomes B. Döhle bodies and giant plts C. two-lobed neutrophils D. mucopolysacchridosis

D. mucopolysacchridosis

When plts concentrate at the edges and feathered end of a blood smear, it is usually due to: A. abnormal proteins B. inadequate mixing of blood anticoagulant C. hemorrhage D. poorly made wedge smear

D. poorly made wedge smear

Which of the following represents characteristics features of iron metabolism in patients with anemia of chronic disorder? A. result A B. result B C. result C D. result D

D. result D

The most characteristic peripheral blood smear finding in multiple myeloma is: A. plasmacytic satellitosis in the bone marrow B. many plasma cells in the peripheral blood C. many Mott cells in the peripheral blood D. rouleaux formation of red cells

D. rouleaux formation of red cells

The results on a patient are: (see picture) These results reflect: A. thrombophilia B. Factor IX deficiency C. heparin D. warfarin

D. warfarin

respiratory alkalosis

Decreased pCO2 corresponds with this condition:

high

Dehydration or excessive dietary intake will cause sodium levels to be:

T Cell Response

Determined by MHC type, accessory molecules, and cytokines present. Usually functions well with infections and live vaccines.

uric acid crystal

Diamond-shaped, rhombic prism *Pathology*: Acute setting --> tumor lysis syndrome Chronic setting --> Gouty arthritis with hyperuricemia

gallbladder

Disease of the _____ is a common cause of obstructive jaundice.

Which of the following lectins will discern group A1 cells from A2 cells

Dolichos biflorus

camera film

Dry slide technology for chemistry assays is based on:

144. What is the most worrisome outcome of Strongyloides infection, especially in immunocompromised patients? A. autoinfection B. Loeffler syndrome C. chronic carrier state D. larva currens E. hyperinfection

E. hyperinfection. Hyperinfection is a potentially lethal complication where infective worms break through the intestinal barrier (autoinfection) and then disseminate intravascularly. Needless to say, that's bad. QCCP2, Strongyloides

108. All of the following are potential consequences of in utero rubella infection, except: A. glaucoma B. congenital heart disease C. sensorineural deafness D. microcephaly E. midzonal hepatic necrosis

E. midzonal hepatic necrosis. Congenital rubella is a potentially devastating infection, especially if acquired in the first trimester. While hepatosplenomegaly can occur, midzonal necrosis is not seen. It is, however, a prominent feature of yellow fever infection. QCCP2,Rubella virus

103. This virus is responsible for nearly all cases of infantile respiratory bronchiolitis: A. parainfluenza virus B. influenza A virus C. metapneumovirus D. Coxsackie A virus E. respiratory syncytial virus

E. respiratory syncytial virus. Respiratory syncytial virus also causes 1/2 of all cases of lower respiratory tract infections in children. Persistence of immunity does not often occur, so reinfection is common. Among other techniques, the characteristic formation of syncytia in Hep-2 cells is used in the diagnosis of RSV. QCCP2,RSV

potassium

EDTA contains this analyte, and will grossly inflate its value:

identical

Electrolyte distribution of blood and of interstitial fluid is approximately:

azotemia

Elevated blood levels of non-protein nitrogenous waste products is a condition known as:

2+

Ferrous iron has this charge:

Myeloid Stem Cell

First in Macrophage lineage, gives off Promonocytes.

Killer Cell Granules

First mechanism to induce apoptosis, includes Granzymes and Perforins, released onto target cell surface after binding via MHC1.

Hinge

Flexible region on antibodies, at the junction of Fab and Fc, enables binding to variable 3-D structures.

Lymph Node Chemokines

Follicular for B Cells, Parafollicular for T Cells and free Dendritic Cells.

100g

For confirmatory glucose tolerance tests done pregnant patients, the dosage of glucose should be:

scattered

For nephelometry, only light that is ______ will reach the detector.

y

For the dilution x:y, the dilution factor is:

Which of the following phenotypes is homozygous for the Fy(a) antigen

Fy(a+b-)

Interstitial Pneumonia

General inflammation in the lungs resulting from bacteria in pulmonary circulation.

VJ Segments

Germline Light Chain exons, undergoes similar recombination process as Heavy Chain region, but does not pair with own Constant region? Cell permanently selects from either Lambda or Kappa locus for higher variability.

hyperglycemia

Gigantism and acromegaly cause:

hyperglycemia

Grave's disease and thyrotoxicosis cause:

tsh

Grave's disease is characterized by production of anti-____ antibodies which fit certain receptors and stimulate the thyroid.

Upon initial typing, the cells from a person with the Bombay phenotype will appear to be which ABO type

Group O

year

Healthcare workers must receive training covering topics such as confidentiality, HIPAA compliance, and sexual harassment at least every:

pre-hepatic jaundice

Hemolytic jaundice is also known as:

release oxygen to the tissue

High 2,3-DPG reduces the ability of hemoglobin to:

metabolic alkalosis

High ammonia indicates that at least some level of ______ is present.

embden-meyerhof pathway

Humans use this metabolic pathway for glycolysis:

hypoglycemia

Hyperinsulinemia causes ______ which may be transient and thus difficult to diagnose.

hypertension, renal failure, strokes, heart failure

Hypernatremia can cause:

Natural Killer Cell Stimulation

IFNg TNF IL2

Secondary Lymphoid Cytokines

IL4, 5, 10, 13. Secreted by TH2 to induce B Cell class switching.

heme

Identify this molecule.

rejected

If four consecutive QC values are greater than 1σ from the mean, it should be:

rejected

If two consecutive QC values are greater than 4σ apart, it should be:

Innate Cytokines

Important in early infection responses, type can vary according to pathogen present, acts immediately. Can also affect Adaptive Immunity. Includes Pro-inflammatories.

increases

In acidosis, plasma potassium:

fluorescence

In direct fluorescent immunoassays, the amount of ______ is proportional to the concentration.

amount of light reflected

In dry chemistries, the _______ is proportional to the concentration of analyte.

probe-analyte antibody complex

In fluorescence polarization immunoassays, only the _____ spins slowly enough to reflect light in the same plane:

molar absorptivity constant

In the equation Aλ = abc, a is:

light path length

In the equation Aλ = abc, b is:

molarity

In the equation Aλ = abc, c is:

precipitate

In turbidemetric analysis, the amount of _____ is proportional to concentration.

parallel

In turbidimetric analysis, the light source is _______ to the photomultiplier.

delta-Bilirubin

Inconsistencies between Bu/Bc and TBIL slides in adults lead to the discovery of this new type of bilirubin:

Bilirubin

Indicates bilirubinuria due to conjugated (direct) bilirubin Bilirubinuria can be normal in dogs but is abnormal in other species.

Enteritis

Inflammation of the GI, allows lumen bacteria to enter portal veins more easily.

Bilirubine in urine

Intra-hepatic or post-hepatic jaundice

light scatter

Lipemic serum, being more turbid, increases:

muscle spasms, cramps

Low blood calcium may cause:

ionized magnesium

Magnesium is very similar physiologically to calcium, but unlike with calcium ______ is rarely measured.

MHC

Major Histocompatability Complex, used to present peptides to ab T-Cells.

Serum/Plasma Creatinine

Males 0.6 to 1.2 mg/dL Females 0.5 to 1.1 mg/dL

spectroscopy

Many chemistry tests rely on this principle:

Chronic Inflammation

Marked by accumulation of M2 and destruction of M1 cells, fibroblasts encapsulate site and body heals around it, uses stationed granulocytes and phagocytes, results in Granuloma Sequestration.

B-Cell Maturation

Marked by expression of surface IgD type receptors. Does apoptosis if binding occurs while still immature (surface IgM only).

Protopod

Mechanism by which Neutrophils move, pushed out in front of them and release digestive enzymes to remove obstructing tissue for easier motility, flowing movement, drags tail.

group I

Medical technologists are classified as _____ under the exposure control plan.

interference filter

Most automated clinical chemistry instruments use this monochromator:

liver

Most tissues can use free fatty acids as an energy source, except this organ which produces ketone bodies instead:

decrease

Not eating or throwing up will cause the blood pH to:

What is the ABO type of check cells

O

140

On pregnant patients, a fasting blood glucose >____mg/dL indicates that the glucose should not be administered.

physiologically active

Only electrolytes that are ionized are:

childhood

Onset of gigantism is in:

Precipitation Assay

Optimal levels of Ab and Ag causes cross-linking (multiple Ab on each Ag) and visible clumping. Can perform serial dilutions to determine titer (last well with visible precipitate).

carbon monoxide poisoning, hemoglobinophathy, anemia, congestive heart failure

Other than respiratory diseases, these are common causes of a low pO2:

chelates metals

Oxalate _______, such as calcium, to prevent clotting.

Thalassemia major

PE: *Marked pallor*,*moderate Jx* * hepatomegaly* spleen can't be palpated CBC: *Hct 17* *MCV 60* WBC 7,800 Plt 450,000 *RBC* : microcyte 2+, target cell 3+, elliptocyte 1+, teardrop 1+, anisocytosis 2+, poikilocytosis 3+, nucleated RBC seen *WBC* : normal WBC population *Plt* : increased platelet smear *Diagnosis* _______

intestinal malabsorption

Patients with this condition should be given an IV glucose tolerance test:

When a clinically insignificant alloantibody is identified in the patient's serum, what is done prior to transfusing the unit

Perform a crossmatch using AHG

probe-analyte, fluorescent-tagged anti-analyte antibody

Performing a fluorescence polarization immunoassay involves adding these to the specimen:

Eosinophil Granules

Phospholipase D, Lysophospholipase, Major Basic Protein, etc.

Lymphoid

Progenitor of Lymphocytes, Natural Killer Cells, and Dendritic Cells.

IL4

Released by Dendritic Cells to activate TH for parasites.

decreases

Resistance _____ over time when water is stored.

Anti-LW is negative (does not react) with which of the following cells

Rhnull cells

light

Samples for bilirubin analysis should be protected from:

Soluble Receptors

Secreted outside of cell, used to consume matching ligand away from target and reduce its intended effects. Can be used for drug therapy.

Tyrosine

Severe liver disease acidic

patterns

Shifts and trends indicate a problem with QC because by definition random error cannot form:

Primary Lymphoid Organ

Site where B-Cells undergo maturation, rearranges DNA to produce a random IgM receptor

Antibiotic

Substance that directly attacks pathogens, made by an outside source, does not cure infections by itself (requires competent immunity).

iron intoxication

TIBC is elevated in this condition because of high serum iron:

total iron binding capacity

TIBC stands for:

Sea Invertebrate Immunity

TLR, antibacterial peptides, and innate complement systems.

oncotic pressure

Technically, colloid osmotic pressure osmometers measure:

inorganic phosphorus

Technically, the phosphate measured in the lab is:

Indirect ELISA

Tests for serum antibodies. Ag fixed to sides, serum Ab added, then conjugated AAb added.

47 mmHg

The "standard" partial pressure of water vapor after humidification of calibrator gases is:

phosphate

The anion which normally shows an inverse relationship with calcium is:

How are the sensitized cells of the unborn destroyed in vivo

The baby's macrophages remove the sensitized cells which are destroyed in his spleen.

When performing a DAT, a control consisting of 6% albumin is run concurrently. What is the result of the control

The control should always be negative.

decrease in absorbance

The conversion of NAD or NADP to NADH or NADPH can be measured by a(n):

bicarbonate, carbonic acid, CO2 gas, carbonates

The four forms of CO2 in the blood are:

glucose-6-phosphate phosphotase

The missing enzyme in Von Gierke's disease is:

dust

The most common cause of stray light is:

serum

The most preferable specimen for bilirubin analysis is:

hydrogenation of NADP

The second step of the hexokinase method involves:

126

The simplest diagnostic criteria for DM is a fasting blood glucose of greater than _____ mg/dL documented on at least two separate occasions.

1

The valence of carbonic acid is:

exogenous insulin

Therapy for type I diabetes primarily consists of diet management and:

potassium, sodium, phosphate

These electrolytes are elevated in uremic syndrome:

photometric linearity, stray light, spectral bandwidth

These should be tested as part of weekly maintenance:

TBIL, Bu/Bc

These slides are used in bilirubin analysis on the Vitros instruments for adults:

slope

This calibrator is used for the high end of the linear range:

mercuric thiocyanate

This colorimetric reaction may be used to assay chloride based on competitive binding of chloride to mercury:

osteoporosis

This common condition results in a slight increase of calcium and no change in phosphate:

grave's disease

This condition is autoimmune and results in overproduction of thyroid hormones:

lactate dehydrogenase

This enzyme converts pyruvate into lactate during anaerobic conditions:

beta-d-glucuronotransferase

This enzyme in the liver is responsible for conjugating bilirubin:

creatine kinase

This enzyme removes a phosphate group from ATP and attaches it to creatine:

2.00-log(%T)

This equation can convert between %T and absorbance: Abs=

crigler-najjar syndrome

This genetic condition is caused by deficiency or total absence of the enzyme which conjugates bilirubin:

gilbert's syndrome

This genetic condition results in physiological jaundice because the sinusoids of the liver are inefficient at absorbing unconjugated bilirubin, allowing it to back up into the bloodstream:

adrenocorticotropic hormone

This hormone stimulates the release of corticosteroids, particularly cortisol:

analytical measurement range

This is another term for linear or reportable range:

chloride

This is the most concentrated anion in plasma:

creatinase

This method converts creatinine to ammonia and then measures ammonia:

glycolysis

This process involves breaking down glucose into pyruvate:

fluorescence polarization immunoassay

This proprietary methodology developed by Abbott Diagnostics uses polarized light to reduce interference:

analytical

This reagent grade is of known purity and all impurities are identified and quantified:

chemically pure

This reagent grade is of known purity but impurities are not identified:

sodium

This substance exerts the biggest influence on osmolality:

-anthroline

This suffix denotes an iron-binding dye:

Growth Hormone

Thymotrophic, decreases after puberty.

37, 0.1

To be accurate, arterial blood gas samples must be analyzed at ____°C within ∓___°C.

200

To diagnose diabetes, glucose at the 2nd hour of the 5HGTT should be <____ mg/dL.

total protein

Total calcium levels are tied with ________, which is why ionized calcium will need to be measured if that assay is high.

600-1600

Total daily urinary excretion of creatinine for females should be ___-___ mg/day.

Mature T-Cells

Travel to area of need via addressins, drain via lymph if they fail to bind antigens at site.

low

Turbidemetric analysis is performed by spectrophotometers at ____ λ.

Protein in the urine is indicative of

UTI, glomerulonephritis or nephrotic syndrome

renally

Urea is excreted:

liver

Urea is produced by the:

Urea to BUN conversion

Urea mg/dL divided by 2.14

Marker Vaccine

Uses a gene-deleted virus, ELISA tests for removed protein, prevents false positives.

Cytotoxic T-Cell

Uses its CD8 to bind a cell's MHC1, antigen presented is interpreted as belonging to the incorrectly presenting cell(s), triggers proliferation and attack of presenting cells.

DHF

V/S: *BT 40.0 C* CBC: *HCT 48* *WBC 3,000* PMN 40% *Plt. 85,000* *RBC*: stomatocyte 1+, normochromic *WBC*: *Atypical lymphocyte* *Plt*: adequate *Diagnosis* ______

Toxoid

Vaccine made from inactivated toxins, imparts active immunity.

100

Very nearly ____% of values in a gaussian distribution will fall within 3σ of the mean.

Endogenous Interference

Viruses interfering with this pathway may avoid NK Cell-induced apoptosis of host cell.

hypoglycemia

Von Gierke's disease causes:

reportable range

When absorbance vs concentration is plotted, the portion of the line that is linear is known as the:

2.8

When calculating osmolality, BUN (in mg/dL) must be divided by:

pentose

Xylose, ribose, and deoxyribose are ______ monosaccharides.

60

___% of renal function must be lost before BUN becomes elevated.

tungsten-halide

_______ bulbs provide a light source at λ=325-750 nm

bicarbonate

_______ makes up the vast majority of the CO2 in the blood.

gaseous

pCO2 electrodes only detect this form of CO2:

addition of acid to specimen

pCO2 electrodes work similarly to the ISE electrodes in normal chemistry instruments, but lack this step:

drop in resistance

pO2 electrodes measure the amount of oxygen as a function of:

Alkaline urine

triple phosphate, amorphous phosphates, ammonium biurate ( spiculated and non)

Acid urine

uric acid crystals(Dalmations), amorphous urates, calcium oxylate, cystine, tyrosine, leucine

hypochromic RBCs, slightly microcytic; associated with hypochromic microcytic anemia which can be due to iron deficiency, thalassemia, anemia of chronic disease, or sideroblastic anemia

what are the arrows indicating? what blood disorders is this associated with?

-basophilic stippling -seen in thalassemias (specific), sideroblastic anemia, megaloblastic anemia -most helpful in distinguishing iron deficiency from thalassemias in hypochromic microcytic anemias

what is the cell in the center displaying? what diseases is this indicative of?

-moderate anemia and splenomegaly -Hb A (α2β2), Hb Barts (ɣ4), and Hb H (β4) are present; Hb A2 (α2δ2) may be normal or reduced -not usually transfusion-dependent

α-thalassemia three-gene deletion

-α-thalassemia trait; microcytosis with or without mild anemia -Hb H (β4) bodies may be seen, splenomegaly may be present

α-thalassemia two-gene deletion

wavelength

λ is a Greek symbol which in the context of spectroscopy refers to:

If a unit of whole blood is drawn in CPDA-1, how long can it be stored and used

35 days

What is the minimum and maximum milliliters of blood that can be drawn in a standard bag

405 to 495 mLs

Hgb A1-C

5.0 to 9.0 %

What is the pulse range for an allogeneic blood donor

50 to 100 beats per minute

One bag of random donor platelets will increase the platelet count by how much

5000 to 10,000/uL

Henderson Hasselbach Eqn pH=

6.103 + log[(Bicarbonate)/(pCO2 x 0.03)

Anion Gap

7 to 16 mMol/L

Blood pH

7.35 to 7.45

After a sample is drawn from the patient, how long can the lab continue to crossmatch from

72 hours

Serum Calcium Total

8.4 to 10.2 mg/dL (2.1 to 2.6 mMol/L)

pO2

83 to 108 mmHg

20

>____ mg/dL is a critical bilirubin value for adults.

750

>____ μg/dL of iron is considered a fatal dose.

excess protein

A diet with _______ may result in pre-renal azotemia.

impaired fasting glucose

A fasting glucose level that is elevated but below 126 mg/dL implicates this condition:

impaired glucose tolerance

A glucose level that is elevated but below 200 mg/dL at the 2 hour mark of an 5HGTT implicates this condition:

trend

A gradual, incremental increase or decrease in QC values is termed a(n):

low

A patient experiencing muscle cramping, spams, and even seizures would likely have a potassium level that is:

high

A patient experiencing muscle weakness and poor coordination would likely have a potassium level that is:

dehydration

A urine-serume osmolality ratio of >3 is indicative of:

What is the MCH if the Hct is 20%, the RBC is 2.4 x 10^6/µL (2.4 x 10^12/L) and the Hgb is 5 g/dL (50 g/L)? A. 21 µm^3 (21 fL) B. 23 µm^3 (23 fL) C. 25 µm^3 (25 fL) D. 84 µm^3 (84 fL)

A. 21 µm^3 (21 fL)

Which curve represents the production of epsilon polypeptide chains of hemoglobin? A. A B. B C. C D. D

A. A

The specimen of choice for preparation of blood films for manual differential leukocyte counts is whole blood collected in: A. EDTA B. oxalate C. citrate D. heparin

A. EDTA

Of the following, the disease most closely associated with glucocerebrosidase deficiency is: A. Gaucher disease B. Chédiak-Higashi syndrome C. Pelger-Huët anomaly D. May-Hegglin anomaly

A. Gaucher disease

D-dimers are produced from: A. cross-linked and stabilized fibrin clot B. decreased fibrinogen and plts C. plasminogen converting to plasmin D. generation of thrombin from endothelial cells

A. cross-linked and stabilized fibrin clot

Lab findings in hereditary spherocytosis do NOT include: A. decreased osmotic fragility B. increased autohemolysis corrected by glucose C. reticulocytosis D. shorted erythrocyte survival

A. decreased osmotic fragility

In polycythemia vera, the leukocyte alkaline phosphatase activity is: A. elevated B. normal C. decreased

A. elevated

In polycythemia vera, the plt count is: A. elevated B. normal C. decreased D. variable

A. elevated

In polycythenmia vera, the Hgb, Hct, and RBC count and red cell mass are: A. elveated B. normal C. decreased

A. elevated

The most likely cause of the macrocytosis that often accompanies anemia of myelofibrosis is: A. folic acid deficiency B. increased retic count C. inadequate B12 absorption D. pryidoxine deficiency

A. folic acid deficiency

Which of the following is characteristic of Bernard-Soulier syndrome? A. giant plts B. normal bleeding time C. abnormal aggregation with ADP D. increased plt count

A. giant plts

The lab findings on a patient are as follows: -MCV: 55 μm^3 (55 fL) -MCHC: 25% -MCH: 17 ph A stained blood film on this patient would most likely reveal a red cell picture that is: A. microcytic, hypochromic B. macrocytic, hypocromic C. normalcytic, normochromic D. microcytic, normochromic

A. microcytic, hypochromic

The M:E ration in plycythemia vera is usually: A. normal B. high C. low D. variable

A. normal

Which of the following stains is closely associated with the lysosomal enzyme in primary (azurophilic) granules? A. peroxidase B. Sudan black B C. PAS D. Prussian blue

A. peroxidase

A leukocyte count and differential on a 40 y/o Caucasian man revealed: -WBC: 5.4 x 10^3/µL (5.4 x 10^9/L) Differential: -Segs: 20% -Lymphs: 58% -Monos: 20% -Eos: 2% This represents: A. relative lymphocytosis B. absolute lymphocytosis C. relative neutrophilia D. leukopenia

A. relative lymphocytosis

Elevation of the lymphocyte percentage above 47% is termed: A. relative lymphocytosis B. absolute lymphocytosis C. leukocytosis D. absolute neutrophili leukocytosis

A. relative lymphocytosis

A RBC is about 5 µm in diameter that stains bright red and shows no central pallor is a: A. spherocyte B. leptocyte C. microcyte D. macrocyte

A. spherocyte

In flow cytometric analysis, low angle or forward scatter of a laster light beam provides information that pertains to a cell's: A. volume B. viability C. granularity D. lineage

A. volume

Absorbance

A=abc

Full compatibility testing consists of

ABO (cells and serum) and Rh typing of the sample and ABO cell typing of the donor An antibody screen at 37oC and AHG on the patient's serum/plasma A major crossmatch consisting of mixing patient's serum with donor cells

95

About ___% of values in a gaussian distribution will fall within 2σ of the mean.

68

About ____% of values in a gaussian distribution will fall within 1σ of the mean.

primary adrenal insufficiency

Addison's disease is also referred to as:

fehling II

Addition of the _______ reagent to a Jendrassic-Grof assay shifts the pH very high and the color turns blue.

liver

Albumin transports unconjugated bilirubin to the:

retention, excretion

Aldosterone causes the ______ of sodium and the _______ of potassium.

What do the following antigens have in common k, Kp(b), Js(b)

All are antigens in the Kell Blood Group System All are high frequency antigens. All are destroyed when treated with a thiol-reducing agent.

What do the following antibodies have in common Anti-N, anti-P1, anti-Le(a)

All are considered clinically insignificant All are usually IgM

What do the following antigens have in common Fy(a), M, N, S

All are destroyed when treated with an enzyme

What do the following antigens have in common Js(b), U, I, Yt(a)

All are high frequency antigens

neonatal physiological jaundice

All newborn babies develop this condition to some extent:

If both the mom and the dad are Group O, what will be the ABO type of their offspring

All of the offspring will be Group O.

muscle mass, activity level

All other things being equal, creatinine production is a function of:

sexual harassment

All physical advances, requests for sexual/physical favors (spoken or implied), or physical conduct of a sexual nature is:

decrease

All respiratory diseases will _____ pO2.

renally

All xylose consumed will be excreted:

Selectins

Allows Neutrophils to bind loosely to venous endothelia and roll along vessel wall (Margination).

Struvite

Also known as triple phosphate alkaline

protein build up

Analyzers may produce inconsistant or slow results due to _______ in sample chambers or on electrodes.

Antigen

Anything that stimulates an immune response via antibodies, pathogens may have many types present on them.

If a donor is hooked to a machine and only one component is harvested from him, he is said to be donating by what technique

Apheresis

7

At 3 days, total bilirubin should be <___ mg/dL.

peptides, small proteins

At the right pH, polyolefin plastics absorb small amounts of:

What is the MCHC if the Hct is 20%, the RBC is 2.4 x 10^6/µL (2.4 x 10^12/L), and the Hgb is 5 g/dL (50 g/L)? A. 21% B. 25% C. 30% D. 34%

B. 25%

Which of the following factor deficiencies is associated with either no bleeding or only minor bleeding tendency, even after trauma or surgery? A. Factor X B. Factor XII C. Factor XIII D. Factor V

B. Factor XII

Of the following, the disease most closely associated with granulocyte hypsegmentation is: A. May-Hegglin anomaly B. Pelger-Huët anomaly C. Chédiak-Higashi anomaly D. Gaucher disease

B. Pelger-Huët anomaly

The electrical resistance method of cell counting requires: A. equal-sized particle B. a conductive liquid C. 2 internal electrodes for current D. three apertures for counting

B. a conductive liquid

Plt activity is affected by: A. calcium B. aspirin C. hyperclycemia D. hypoglycemia

B. aspirin

The large cell in the center of the image would be best described as a(n): A. neutrophil B. basophil C. eosinophil D. myelocyte

B. basophil

The presence of excessive rouleaux formation on a blood smear is often accompanied by an increased: A. retic count B. sed rate C. Hct D. erythrocyte count

B. sed rate

The Prussian blue staining of a peripheral blood identifies: A. Howell-Jolly bodies B. siderotic granules C. retics D. basophilic stippling

B. siderotic granules

The anticoagulant of choice for routine coag procedures is: A. sodium oxalate B. sodium citrate C. heparin D. sodium fluoride

B. sodium citrate

A screening test for paroxysmal nocturnal hemoglobinuria is: A. heat instability test B. sucrose hemolysis C. osmotic fragility D. dithionite solubility

B. sucrose hemolysis

In a the Clauss fibrinogen method, the time to clot formation in plasma is measured after the addition of: A. Ca B. thrombin C. phospholipids D. kaolin

B. thrombin

A common source of interference in the cyanmethemoglobin method is: A. hemolysis B. very high WBC count C. cold agglutinins D. clumped PLTs

B. very high WBC count

-thalassemia (non-specific) -target cells

-what disease is this indicative of? -what are the cells with a second red ring in the middle of the central pallor?

164. Which of the dimorphic fungi are associated with sclerosing mediastinitis? A. Histoplasma B. Blastomyces C. Coccidioides D. Sporothrix E. Paracoccidioides

164. Which of the dimorphic fungi are associated with sclerosing mediastinitis? A. Histoplasma B. Blastomyces C. Coccidioides D. Sporothrix E. Paracoccidioides

To be sure that a tech is 95% confident when identifying an antibody, how many cells, at the very least, must be positive for the antigen and yield a positive result and how many cells must be negative for the antigen and yield a negative result

3 positive and 3 negative

Lymphocyte Distribution

3/4 are T-Cells, 2:1 CD4 Helpers to CD8 Cytotoxics.

Urine, 24-Hour Osmolality

300 to 900 mOsm/Kg

pCO2

35 to 45 mmHg

If a patient has anti-Fy(a) in his serum, he must receive red blood cells that are negative for the Fy(a) antigen. What percent of the Caucasian population is negative for the Fy(a) antigen

35%

All blood that is drawn in CPD or CP2D can have its storage time extended by adding an additive solution (also called Adsol and Adenine Solution). When this is added within 72 hours of the unit's being drawn, what will be the new expiration date

42 days and no more.

How much anticoagulant/preservative must be in a standard bag to draw 450 mLs of whole blood

63 mL

BUN

7 to 18 mg/dL

Plasma Ammonia Nitrogen

7 to 27 uMol/L

O2 Sat.

95% - 98%

base excess

A calculated parameter which compares the bicarbonate to the pH is:

The amount of antigen present on a cell influences the first stage of agglutination. Which of the following cells has the most antigen

A cell that is homozygous for the antigen

viral hepatitis

A common cause of hepatocellular jaundice is:

severinghaus

A common name for a CO2 ISE is:

polysaccharide

A complex carbohydrate composed of many monosaccharides bonded together is termed a(n):

hyperparathyroid disease

A condition in which calcium is high and phosphates are low is:

spectrophotometer

A device which measures the absorbance of a solution is a(n):

von gierke's disease

A hereditary condition which stems from enzyme deficiency and traps glucose in the liver is:

metabolic alkalosis

A high bicarbonate level indicates:

copper ring around iris

A key finding of Wilson's disease is:

atomic absorption spectrophotometry

A legacy methodology for measuring electrolytes based on the absorption of energy by a ground state molecule is:

flame photometry

A legacy methodology for measuring electrolytes based on the emission of a photon by an excited molecule is:

new test, new lot, calibration

A linearity study should be done at least every 6 months, or in any of these circumstances:

adrenal insufficiency

A low anion gap is almost always due to analytical error, but this condition will also cause it:

metabolic acidosis

A low bicarbonate level indicates:

linear

A method that produces absorbance results that follow Beer's law is said to be:

absorbance, concentration

A more practical application of Beer's law in the context of clinical chemistry is that _____ is proportional to ______.

<10 nm

A narrow bandpass spectrophotometer has a bandpass of:

metabolic alkalosis

A negative base excess indicates:

unauthorized

A person who does not need to know confidential information to do their job is said to be:

metabolic acidosis

A positive base excess indicates:

7

A potassium value of >___ mmol/L is typically considered to be incompatible with life and is unlikely to be a true value.

9

A potassium value of >____ mmol/L is almost certainly caused by contamination.

control

A solution which has been assayed many times and has an established mean and acceptable range is a(n):

standard

A solution with a known concentration of an analyte used as a reference point is a(n):

standard

A solution with a known concentration which can be used for the purposes of instrument calibration, quality control analysis, etc, is a(n):

colorimeter

A spectrophotometer which uses an interference filter rather than a true monochromator is more accurately referred to as a(n):

fluorescent

A substance which absorbs light and releases it at a different λ is said to be:

shift

A sudden, major change in QC values is termed a(n):

bias error

A technical error which causes a loss of accuracy is known as a(n):

kidney disease

A urine-serum osmolality ratio of <2 is indicative of:

Haptin

A very small foreign compound, does not elicit an immune response until it covalently binds to other proteins and becomes a --- Carrier Complex, produced antibodies are also specific for the complex.

simple

A(n) _____ dilution is performed by adding sample to a diluent.

complex

A(n) _____ dilution is performed by diluting more than once with different dilution ratios.

serial

A(n) _____ dilution is performed by diluting more than once with the same dilution ratio.

linearity study

A(n) _____ is performed at least every 6 months to assess the reportable range of an assay.

certified reference

A(n) ______ standard is a biological solution that does not need to be reconstituted because it is a pre-made liquid.

primary

A(n) ______ standard is of known purity and can be used to make solutions of exactly known concentrations.

secondary

A(n) ______ standard is of known purity but because of its properties cannot be precisely measured out to produce solutions of exact concentration.

standard

A(n) _______ is not necessarily biological in origin, but may be.

Which of the following is the formula for MCHC? A. (Hgb x 100)/Hct B. Hgb/RBC C. RBC/Hct D. (Hct x 1000)/RBC

A. (Hgb x 100)/Hct

Which of the following is the formula for manual white cell count? A. (number of cells counted x dilution x 10)/number of squares counted B. (number of cells counted x dilution)/10 x number of squares counted C. number of cells counted x dilution D. number of cells counted x number of squares counted

A. (number of cells counted x dilution x 10)/number of squares counted

An automated luekocyte count is 22.5 x 10^96,250/µL (22.5 x 10^9/L). The differential reveals 200 normoblasts/100 leukocytes. What is the actual leukocyte count per microliter? A. 7,500/µL (7.5 x 10^9/L) B. 11,500/µL (11.5 x 10^9/L) C. 14,400/µL (14.4 x 10^9/L) D. 22,300/µL (22.3 x 10^9/L)

A. 7,500/µL (7.5 x 10^9/L)

174. Which of the following organisms is characterized by two rows of phialides on the conidia? A. A. terreus B. A. niger C. A. fumigatus D. A. flavus E. P. marneffei

A. A. Just like the discrete gross appearance on plates, each of the Aspergillus spp can be speciated by the appearance of the conidia. A. terreus has 2 rows of phialides. A. niger has black circumferential phialides. A. fumigatus has a single row of phialides. A. flavus has a circumferential row of phialides (a "flavorful lollipop"). P. marneffei looks like A. fumigatus but lacks the swollen vesicle at the base of the phialides that all the members of Aspergillus have. QCCP2, Aspergillus

137. Extraerythrocytic ring forms are characteristic of which organisms? A. Babesia microti B. Plasmodium falciparum C. Plasmodium vivax D. Plasmodium ovale E. Plasmodium malariae A.

A. B . Only Babesia has extraerythrocytic forms. In addition, there aren't gametocytes or schizonts present in the peripheral blood with babesiosis. QCCP2, Babesia microti

123. What is the non-pathogenic flagellate that must be distinguished from Giardia lamblia? A. Chilomastix mesneli B. Dientamoeba fragilis C. Trichomonas vaginalis D. Trypanosoma cruzi E. Leishmania donovani

A. C . Unlike the "falling leaf" motility of Giardia, C. mesneli has a rotary motion and a cyst form with only one nucleus (Giardia cysts have 4. The other choices are all pathogenic organisms. , QCCP2, Chilomastix mesneli

Of the following, the disease most closely associated with cytoplasmic granule fusion is: A. Chédiak-Higashi syndrome B. Peger-Huët anomaly C. May-Hegglin anomaly D. Alder-Reilly anomaly

A. Chédiak-Higashi syndrome

155. B cell (humoral) immunodeficiency increases susceptibility to this organism: A. Giardia B. Trichomonas C. Toxoplasma D. Strongyloides E. Cryptosporidium

A. G. Cryptosporidium, Toxoplasma, and Strongyloides infections are affected by T-cell immunodeficiency, while Trichomonas is not significantly affected by immunodeficiency. QCCP2, Parasitic infections in immunodeficient patients

A patient has a congenital nonspherocytic hemolytic anemia. After exposure to anti-malarial drugs the patient experiences a severe hemolytic episode. This episode is characterized by red cell inclusions caused by hemoglobin denaturation. Which of the following conditions is MOST consistent with these findings? A. G6PD deficiency B. thalassemia major C. pyruvate kinase deficiency D. paroxysmal nocturnal hemoglobinuria

A. G6PD deficiency

69. In which of the following patient populations is the risk of CMV retinitis, encephalitis, or nephritis at its highest? A. HIV patients with a CD4 counts <100/mL B. HIV patients with CD4 counts between 500 and 1000/mL C. solid-organ transplant recipients D. older children E. elderly adults

A. HIV patients with CD4 counts <100/mL. In addition to HIV patients with very low CD4 counts, immunocompromised transplant recipients are at an increased risk, though not as great as that of HIV patients. Typically, primary CMV presents with a mononucleosis-like syndrome, which can progress to a pneumonia - especially in neonates and the immunocompromised. QCCP2,CMV

79. What's the best definition of heterophile antibodies as produced in EBV infections? A. IgM antibodies with an affinity for sheep and horse red blood cells B. IgM antibodies with an affinity for the capsule of all DNA viruses C. IgA antibodies directed against protein-antigens often consumed in a normal diet D. IgG antibodies secreted in tears with an affinity pigeon egg antigens E. IgG antibodies with an affinity for plant antigens

A. IgM antibodies with an affinity for sheep and horse red blood cells. Heterophile antibodies are a fairly specific, though not very sensitive indicator of EBV infection. They are also the basis of the Monospot EBV detection agglutination assay. QCCP2,EBV

57. All of the following organisms utilize the mosquito as a vector, except: A. Loa loa B. Wuchereria bancrofti C. Brugia malayi D. Dirofilaria immitis E. Plasmodium falciparum

A. L. Loa loa is spread by the mango fly (Chrysops). There are a few memorable and significant vector-organism pairs to remember. Lyme disease, ehrlichiosis, and babesiosis are spread by the Ixodes tick. Borrelia recurrentis is the only borrelial species spread by a louse (the human body louse) rather than a tick. Trypanosoma cruzi and the reduviid bug, Leishmania (cutaneous) and the sandfly (Phlebotomus), and Onchocerca and the black fly (Simulium) are all high-yield. QCCP2,T3.3, Vectors

111. All of the following are endemic areas for the human T cell lymphotropic virus - I, except: A. Norway B. Caribbean C. S. Japan D. Brazil E. S. Africa

A. Norway. HTLV-I is spread primarily through IV drug use and sexual contact. It is the causative agent of the demyelinating disease, tropical spastic paraparesis and adult T cell lymphoma through the infection of CD4+ T cells. QCCP2,HTLV-I

134. Individuals who lack the Duffy antigen on the surfaces of their red blood cells are protected against which species of Plasmodium? A. P. vivax B. P. falciparum C. P. malariae D. A & B E. A, B, C

A. P. . The Duffy antigen is the receptor for P. vivax; therefore, loss of the receptor affords some protection against infection. G6PD deficiency provides some protection against all species of Plasmodium, while sickle cell trait protects individuals predominantly against P. falciparum. QCCP2, Plasmodium

A differntial was performed on an asymptomatic patient. The differential included 60% neutrophils: 55 of which had 2 lobes and 5 had 3 lobes. There were no other abnormalities. This is consistent with which of the following anomalies? A. Pelger-Huët B. May-Hegglin C. Alder-Reilly D. Chédiak-Higashi

A. Pelger-Huët

149. The eggs of which species of Schistosoma can be isolated from urine? A. S. haematobium B. S. japonicum C. S. mekongii D. S. mansoni E. S. intercalatum

A. S. . S. intercalatum and S. haematobium are very similar, but while S. intercalatum infects the intestine, S. haematobium prefers the veins of the bladder. S. mansoni and S. japonicum primarily infect the liver and can lead to cirrhosis. S. mekongii is very similar to S. japonicum but in a more limited distribution in Laos and Cambodia. QCCP2,Schistosoma (bilharziasis)

Prothrombin is: A. a protein formed by the liver in the presence of Vitamin K B. an enzyme that converts fibrinogen into fibrin threads C. the end product of the reaction between fibrinogen and thrombin D. a protein released by plts during coagulation

A. a protein formed by the liver in the presence of Vitamin K

A bedside test that can be used to monitor heparin activity is the: A. activated clotting time B. stypven time C. reptilase time D. partial thromboplastin time

A. activated clotting time

The most common form of childhood leukemia is: A. acute lymphocytic B. acute granulocytic C. acute monocytic D. chronic granulocytic

A. acute lymphocytic

A 50 y/o woman who has been receiving busulfan for three years for chronic myelogenous leukemia becomes anemic. Lab tests revealed: -Thrombocytopenia -Many peroxidase-neg blast cells in the peripheral blood -Bone marrow hypercellular in blast transformation -Markedly increased bone marrow TdT Which of the following complications is the patient most likely have? A. acute lymphocytic leukemia B. acute myelocytic leukemia C. acute myelomonocytic leukemia D. busulfan toxicity

A. acute lymphocytic leukemia

The chamber counting method of plt enumeration: A. allows direct visualization of the particles being counted B. has a high degree of precision C. has a high degree of reproducibility D. is the method of choice for the performance of 50-60 counts per day

A. allows direct visualization of the particles being counted

The RDW-CV and RDW-SD performed by automated cells counters are calculations that provide: A. an index of the distribution of RBC volumes B. a calculated mean RBC hgb concentration C. a calculated mean cell hub D. the mean RBC volume

A. an index of the distribution of RBC volumes

Low molecular weight heparin is monitored by a: A. anit-Xa assay B. APTT C. PT D. anti-IIa assay

A. anit-Xa assay

Specific (secondary) granules of the neutrophilic granulocyte: A. appear first at the myelocyte state B. contain lysosomal enzymes C. are formed on the mitochondria D. are derived from azurophil (primary) granules

A. appear first at the myelocyte state

Evidence of active red cell regeneration may be indicative on a blood smear by: A. basophilic stippling, nucleated RBCs and polychromasia B. hypchromia, macrocytes and nucleated RBCs C. hypochromia, basophilic stippling and nucleated RBCs D. Howell-Jolly bodies, Cabot rings and basophilic stippling

A. basophilic stippling, nucleated RBCs and polychromasia

Which of the following detects or measures plt function? A. bleeding time B. prothrombin time C. thrombin time D. partial thromboplastin time

A. bleeding time

Plt aggregation is dependent in vitro on the presence of: A. calcium ions B. sodium citrate C. thrombin D. potassium

A. calcium ions

In synovial fluid, the most characteristic finding in pseudogout is: A. calcium pyrophosphate dihydrate crystals B. cartilage debris C. monosodium urate crystals D. hemosiderin-laden macrophages

A. calcium pyrophosphate dihydrate crystals

The photo-optical method of endpoint detection is described as: A. change in optical density as a result of a fibrin clot B. measurement of tumidity of antigen-antibody formation C. decreased motion of a mechanical ball D. color-producing chromophor

A. change in optical density as a result of a fibrin clot

An oncology patient has the following results: The most probable explanation is: A. chemotherapy B. cold antibody C. clotted specimen D. inadequate mixing

A. chemotherapy

All stages of neutrophils are most likely to be seen in the peripheral blood of a patient with: A. chronic myelocytic leukemia B. myelofibrosis with myeloid metaplasia C. erythroleukemia D. acute myelocytic leukemia

A. chronic myelocytic leukemia

The most characteristic morphologic features of atypical lymphocytes include: A. coarse nuclear chromatin and basophilic cytoplasm B. blue-grey cytoplasm, fine nuclear chromatin C. nucleoli and deep blue RNA-rich cytoplasm D. a stretched nucleus and cytoplasmic indentations

A. coarse nuclear chromatin and basophilic cytoplasm

100. Which of the following tests for influenza is considered the gold standard? A. culture B. direct fluorescent antibody C. detection of influenza RNA D. serology E. rapid Monospot

A. culture. Nasopharyngeal, sputum, or throat samples can be used to culture virus, either in cell culture or in the more rapid shell vial assay. All the other tests mentioned, with the exception of the Monospot test, have shown utility in the diagnosis of influenza. QCCP2,Orthomyxovirus, diagnosis

156. For what purpose is Niger seed agar used? A. detecting melanin pigment in Cryptococcus B. selective agar for Malassezia furfur C. Aspergillus speciation D. to visualize yeast forms E. to detect pigment in Trichophyton rubrum

A. detecting melanin pigment inC. Bird seed (Niger) is used to selectively demonstrate Cryptococcus neoformans, where it will form brown/black colonies due to enzymatic (phenol oxidase) conversion of caffeic acid to melanin, within a week. QCCP2,T3.21

66. How can herpes simplex virus definitively be identified in shell vial assay? A. direct fluorescent antibody stain B. cytopathic effect viewed with light microscopy C. reculture (shell vial assay as a starter culture) D. A & B E. A, B, C

A. direct fluorescent antibody stain. The shell vial technique is very popular in virus labs due to its small size, low volume, ease of use, and ability to run multiple samples in parallel. The assay involves centrifugation of the patient sample onto a cover slip coated with a culture monolayer. In a shell vial assay for HSV, cytopathic effect is NOT used as indication of a positive assay, but rather the detection of viral antigens by direct fluorescent antibody staining. QCCP2,Human herpes virus

Cells for the transport of O2 and CO2 are: A. erythrocytes B. granulocytes C. lymphocytes D. thormbocytes

A. erythrocytes

On a smear made directly from a finger stick, no plts were found in the counting area. The first thing to do is: A. examine the slide for clumping B. obtain another smear C. perform a total plt count D. request another finger stick

A. examine the slide for clumping

A hemophiliac male and a normal female can produce a: A. female carrier B. male carrier C. male hemophiliac D. normal femal

A. female carrier

A patient diagnosed as having bacterial septicemia. Which of the following would best describe the expected change in his peripheral blood? A. granulocytic leukemoid reaction B. lymphocytic leukemoid reaction C. neutropenia D. eosinophilia

A. granulocytic leukemoid reaction

The majority of the iron in an adult is found as a constituent of: A. hemoglobin B. hemosiderin C. myglobin D. transferrin

A. hemoglobin

Acute DIC is characterized by: A. hypfibrinogenemia B. thrombocytosis C. negative D-dimer D. shortened thrombin time

A. hypfibrinogenemia

Blood collected in EDTA underuse which of the following changes if kept at room temp for 6-24 hours? A. increased Hct and MCV B. increased ESR and MCV C. increased MCHC and MCV D. decreased retic count and Hct

A. increased Hct and MCV

When using an electronic cell counter, which of the following results can occur in the presence of a cold agglutinin? A. increased MCV and decreased RBC B. increased MCV and normal RBC C. decreased MCV and increased MCHC D. MCV and RBC

A. increased MCV and decreased RBC

Biochemical abnormalities characteristic of polycythemia vera include: A. increases serum B12 binding capacity B. hypuricemia C. hypohistaminemia D. decerased leukocyte alkaline phosphatase activity

A. increases serum B12 binding capacity

50. All of the following criteria are used in the diagnosis of prosthetic joint infection, except: A. joint pain with positive bacteremia in two successive blood cultures B. growth of the same microorganism in two or more synovial fluid or periprosthetic tissue cultures C. purulent synovial fluid or periprosthetic tissue D. acute inflammation in periprosthetic tissue E. presence of a sinus tract

A. joint pain with positive bacteremia in 2 successive blood cultures. Each of the applicable criteria is fairly specific for prosthetic joint infection. There are several etiologies of joint infection. The most common is the direct introduction of bacteria, though in a small percentage of late infection, hematogenous spread is the cause. QCCP2,Prosthetic joint infection and other clinical syndromes

The most potent plasminogen activator in the contact phase of coagulation is: A. kallikrein B. streptokinase C. HMWK D. fibrinogen

A. kallikrein

96. Which is the best test for determining the likelihood of progression to cirrhosis in a patient with chronic HCV? A. liver biopsy B. quantitative HCV RNA C. qualitative HCV RNA D. serial ALT levels E. HCV serotype determination

A. liver biopsy. All of the tests mentioned have a role in the diagnosis and management of HCV. However, only liver biopsy can provide the data required to grade inflammation and stage fibrosis of liver infection. QCCP2,HCV

133. Where do Plasmodium sporozoites proliferate? A. liver B. red blood cells C. bone marrow D. within nucleated erythrocyte precursors E. freely within the blood

A. liver. After sporozoites are introduced into the blood stream by an anopheline mosquito, they travel to the liver to proliferate. The infected hepatocytes rupture and release the merozoites, which infect RBCs. Hypnozoites are the forms that maintain a latent infection in the liver. QCCP2, Plasmodium

The philadelphia chromosome is found by the translocation between the: A. long arm of chromosome 22 and long arm of chromosome 9 B. long arm of chromosome 21 and long arm of chromosome 9 C. long arm of chromosome 21 and short arm of chromosome 6 D. long arm of chromosome 22 and short arm of chromosome 6

A. long arm of chromosome 22 and long arm of chromosome 9

43. Which of the following viruses is the most common cause of winter viral encephalitis? A. lymphocytic choriomeningitis virus B. Coxsackie A virus C. Coxsackie B virus D. human herpes virus 6 E. West Nile virus

A. lymphocytic choriomeningitis virus. LCMV is the most common cause of winter/spring viral encephalitis, while enteroviruses such as Coxsackie, are the main causes of summer/fall epidemics. LCMV is spread to humans through contact with infected mouse feces. QCCP2,Meningitis, specific agents

The characteristic morphologic feature in folic acid deficiency is: A. macrocytosis B. target cells C. serum B12 and folate D. leukocyte alkaline phosphatase

A. macrocytosis

A plt determination was performed on an automated instrument and a very low value was obtained. The plts appeared adequate when estimated from the stained blood film. The best explanation for this discrepancy is: A. many plts are abnormally large B. blood sample is hemolyzed C. white cell fragments are present in the blood D. red cell fragments are present in the blood

A. many plts are abnormally large

Which of the following cells is the largest cell in the bone marrow: A. megakaryocyte B. histiocyte C. osteoclast D. mast cell

A. megakaryocyte

Which of the following is associated with Chédiak-Higashi syndrome? A. membrane defect of lysosomes B. Döhle bodies and giant plts C. two-lobed neutrophils D. mucopolysacchridosis

A. membrane defect of lysosomes

The large nucleated cell in the lower right-hand side of the image below is a: A. myelocyte B. metamyelocyte C. basophil D. plasma cell

A. myelocyte

Which of the following is a characteristic of Factor XII deficiency? A. negative bleeding history B. normal clotting times C. decreased risk of thrombosis D. epistaxis

A. negative bleeding history

The following results were obtained on a 45 y/o man complaining of chills and fever: -WBC: 23.0 x 10^3/µL (23.0 x 10^9/L) -LAP: 200 -Philadelphia choromosome: neg Differential: -Segs: 60% -Bands: 21% -Lymphs: 11% -Monos: 3% -Metamyelocytes: 2% -Meylos: 3% -Toxic granulation, Döhle bodies and vacuoles These results are consistent with: A. neutrophilic leukemoid reaction B. polycythemia vera C. chronic meylocytic leukemia D. leukoerythroblastosis in myelofibrosis

A. neutrophilic leukemoid reaction

The principle confirmatory test for hereditary speherocytosis is: A. osmotic fragility B. sucrose hemolysis C. heat instability test D. Kaihauer-Betke

A. osmotic fragility

101. What's the most common secondary complication that can arise with measles infection? A. otitis media B. pneumonia C. myocarditis D. appendicitis E. subacute sclerosing panencephalitis

A. otitis media. Superinfection can cause otitis media or pneumonia, especially in immunocompromised patients. All of the other choices can happen with measles infection but for the most part are rare, especially SSPE. QCCP2,Measles

Peripheral blood smears from patients with untreated pernicious anemia are characterized by: A. pancytopenia and macrocytosis B. leukocytosis and elliptocytosis C. leukocytosis and ovalocytosis D. pancytopenia and microcytosis

A. pancytopenia and macrocytosis

Refer to the following pattern: Which electrophoresis pattern is consistent with sickle cell trait? A. pattern A B. pattern B C. pattern C D. pattern D

A. pattern A

A patient diagnosed with polycythemia vera 5 years ago now has a normal Hct, decreased Hgb and microcytic, hypochromic red cells. What is the most probable cause of the current blood situation? A. phlebotomy B. myelofibrosis C. preleukemia D. aplastic anemia

A. phlebotomy

A bleeding time is used to evaluate the activity of: A. plts B. prothrombin C. labile factor D. Factor XIII

A. plts

71. All of the following factors affect the rate of CMV seropositivity? A. pregnancy status B. locale C. age D. socioeconomic status E. all of the above affect the seropositivity rate

A. pregnancy status. While the effects of primary and reactivated CMV infection in pregnant women are potentially dire, there is no statistically significant effect on the rate of seropositivity. The rate of seropositivity in Africa is much greater than most of the rest of the world. The risk of CMV infection increases with age (presumably due to increased risk of exposure). Also, lower socioeconomic status is associated with increased CMV seropositivity. QCCP2,CMV

A patient is taking 10 mg per day of Coumadin (warfarin). The results of which of the following lab tests will be most impacted? A. protein C level B. antithrombin III level C. Factor V Leiden mutation D. Factor VIII level

A. protein C level

A phase-platelet count was performed and the total plt count was 356 x 10^3/µL (356 x 10^9/L). 10 fields on the stained blood smear were examined for plts and the results per field were: -16, 18, 15, 20, 19, 17, 19, 18, 20, 16 The next step would be to: A. report the phase-plt count since it correlated well with the slide B. repeat the phase-plt count on a recollected specimen and check for clumping C. check ten additional fields on the blood smear D. repeat the plt count using a different method

A. report the phase-plt count since it correlated well with the slide

Which of the following it closely associated with erythroleukopenia? A. ringed sideroblasts, nuclear budding and Howell-Jolly bodies B. DIC C. micromegakaryocytes D. lysozymuria

A. ringed sideroblasts, nuclear budding and Howell-Jolly bodies

73. What's the most common means of transmission of EBV? A. saliva B. blood C. fecal oral D. respiratory droplet E. solid organ transplantation

A. saliva. They don't call it the kissing disease for nothing! A less common means of transmission is through blood transfusion or solid organ transplantation. Nearly 90% of people in the world have been infected with EBV. QCCP2,EBV

113. Which test is the primary screening test for HIV? A. serum enzyme-linked immunosorbent assay B. Western blot C. quantitative HIV RNA D. CD4 count E. p24 antigen detection

A. serum enzyme-linked immunosorbent assay. The extremely high sensitivity of the serum ELISA test makes it the screening test of choice. There can be a window period between the infection and seroconversion, usually 6-8 weeks, leading to possible false negative results. The confirmation of a positive result is usually done with Western blot, which is read positive if at least two bands (p24, gp41, and gp120/160) are positive. QCCP2,HTLV-3 [HIV-1 and -2]

119. What is the principal means of distinguishing Entamoeba histolytica from Entamoeba hartmanii by light microscopy? A. size of trophozoite B. appearance of karyosome C. appearance of nuclear chromatin D. number of nuclei in cyst form E. appearance of chromatoidal bars in cyst

A. size of trophozoite. The only selection that is different between the related organisms is the size of the trophozoite. Entamoeba histolytica is roughly twice the size at 20-30 microns diameter than E. hartmanii, which is only 5-10 microns in diameter. Also, though non-specific, E. histolytica tends to be the most common trophozoite to contain ingested RBCs. QCCP2,T3.16, Amoeba that resemble E. histolytica

106. What does hantavirus and the Crimean-Congo hemorrhagic fever viruses have in common? A. they are members of the bunyavirus family B. they are arthropod-borne C. they are found in geographically overlapping areas D. they have extremely long latency E. they have DNA-based genomes

A. they are member of the bunyavirus family. Hantavirus and the Congo-Crimean hemorrhagic fever virus are both members of the bunyavirus family, along with other hemorrhagic fever viruses, such as Rift Valley fever and La Crosse viruses. Most other hemorrhagic fever viruses belong to other virus families, such as the filoviruses Marburg and Ebola. QCCP2, Family Bunyaviridae

The ESR can be falsely elevated by: A. tilting the tube B. refrigerated blood C. air bubbles in the column D. specimen being too old

A. tilting the tube

Conjugate

Ab or Ag covalently bound to a label, may be an enzyme or fluorescing structure.

conjugate bases

Abnormal anions are frequently ________ of acids produced by the body, taken as medication, or consumed accidentally / for the purpose of harming oneself.

partially-compensated condition

Abnormal pH and bicarb and pCO2 are both abnormal indicates:

uncompensated condition

Abnormal pH and only one abnormal value out of bicarb and pCO2 indicates:

heart

Abnormal potassium most critically affects the:

2

About __% of conjugated bilirubin, since it is water soluble, makes it back into the bloodstream and is excreted renally.

Macrophage Function

Act to kill and clear dead cells/pathogens, delayed response to invasion (several days). Effective against resistant pathogens, may be only mechanism of immunity. Can also secrete cytokines as initial defense while inactive, and can perform antigen presenting to T-Cells.

Dendritic Cell

Activate naive TH Cells, can present using both MHC types at once if needed.

Mature Dendritic Cell

Activated by P/DAMP, cytokines, etc. Exits tissues to lymph nodes to present to TH Cells, low FcR, high MHC and cytokine levels.

Primary Lymphoid Response

Activation of naive lymphocytes, performed by Dendritic Cells in the lymph nodes. Results in clonal expansion of T Cells and IgM-type B Cells.

Neutrophil Regulation

Activity/adhesion/NET depressed with glucocorticoids (stress/drugs), some viruses, certain bacterial virulence factors, neonatals, and before/after parturition. Upregulated with Native Defense (tissue response to danger signals) and T-Helper Cell Cytokines.

Dysmorphic RBC

Acute glomerulonephritis DDx. IgA nephropathy Lupus nephritis, post infectious glomerulonephritis

Mannose-Binding Protein

Acute phase protein produced by liver, sticks to pathogens for opsonization and complement cascade.

Classical Pathway

Adaptive mechanism, C1 protein (held together by calcium) circulates inactive until it binds two IgG or one IgM (more efficient) in close proximity on an antigen surface, becomes an active enzyme C1b while remaining peptide C1a diffuses away, similarly clips other covalently bound Complement proteins into enzymes until Terminal Pathway is reached.

Which of the following methods can be used to overcome a delay in clot formation when a patient has a prolonged clotting time

Add liquid thrombin to the tube of blood Add dry thrombin to the tube of blood Add glass beads to the tube of blood

diabetic coma

Administering a glucose load to a patient with a very high fasting blood glucose level can lead to:

low

Adrenal insufficiency or Addison's disease will cause sodium levels to be:

OGTT

Adult - 75 g load Child - 1.75 g/kg Pregnant - 50g test, 100g confirm

Potassium (mEq/L = mMol/L)

Adult 3.5-5.1 mMol/L Newborn 3.7-5.9 mMol/L

Serum Inorganic Phosphorous

Adults 3.0 to 4.5 mg/dL Children 4.5 to 6.5 mg/dL

High Endothelial Vessels

Afferent blood vessels to a lymph node, site where immature B and T Cells enter via diapedesis (use Addressins).

lot #, expiration date, calibration

After a QC failure, a few things to check are:

spleen

Aged or deformed erythrocytes are culled from circulation by the:

NH4Cl

Ammonia ISEs contain a solution of:

base

Ammonia is the strongest naturally occurring:

7-27

Ammonia normal range is ___-___ μMol/L nitrogen.

ammonium ion and hydroxide ion

Ammonia reacts with water to form:

What is antihuman globulin

An IgG antibody against human globulin

oxygen binding

An O2 Sat which is abnormal in the context of the pO2 represents a defect in:

2

An SDI of greater than _____ indicates likely failure of next PT challenge cycle.

ionized

An advantage of ISE over earlier methods is that it is capable of measuring only _____ levels of a molecule.

If the antibody screen is positive after the addition of AHG and all crossmatched units are compatible after AHG, what is a probable cause

An alloantibody is present

Anti-immunoglobulin

An antibody that is specific for another antibody.

pheochromocytoma

An epinephrine secreting tumor, usually of the adrenal medulla, is termed a(n):

20

An osmolal gap >____ mOsm/kg indicates the presence of an unknown substance.

52. What is the causative agent of visceral larva migrans? A. Ancylostoma brazilensis B. Loa loa C. Toxicara canis D. Francisella tularensis E. Chlamydia trachomatis

Ancylostoma is the causative agent of cutaneous larval migrans, while Loa loa inhabits the subcutis and conjunctiva. Francisella tularensis causes ulceroglandular fever. Chlamydia trachomatis causes lymphogranuloma venereum and trachoma. QCCP2,T3.2

glomerular filtration rate

Another term used to describe the results of a creatinine clearance test is:

Quiz Question: A Gram negative rod is described as having a "sea gull wing" appearance and grows best in a microaerophilic atmosphere best describes: A) Vibrio sp. B) Campylobacter sp. C) Aeromonas sp. D) HACEK organisms

Answer: B) Campylobacter sp.

Quiz Question: An oxidizer gives what reaction in OF medium? A) Open (-) Closed (-) B) Open (+) Closed (-) C) Open (+) Closed (+) D) None of the above

Answer: B) Open (+) Closed (-)

Quiz Question: What can you conclude from the following OF medium :open - (green), closed - (green) A) Fermenter B) oxidizer C) Non -ultilizer D) Oxidizer and Fermenter

Answer: C) Non -ultilizer

Quiz Question: A Patient comes to the ER after complaining of severe gastroenteritis and "rice-water like stools following a trip outside of the US. Stool samples and blood work were collected. Colonies that grew on TCBS were yellow, oxidase positive. What do you suspect? A) Vibrio vulnificus B) Vibrio parahaemolyticus C) Vibrio cholerae D) Campylobacter jejuni

Answer: C) Vibrio cholerae

Quiz Question: TCBS agar is used to isolate: A) Pseudomonas sp. B) Legionella sp. C) Vibrio sp. D) Campylobacter sp

Answer: C) Vibrio sp.

Quiz Question: An organism that is K/K on TSI, Oxidase Negative, Motility Negative, OF Glucose (open) positive, and has a purple hue on MacConkey even though is is NOT a lactose fermenter is most likely: A) Acinetobacter iwoffi B) Eikenella corrodens C) Pseudomonas aeruginosa D) Acinetobacter baumannii

Answer: D) Acinetobacter baumannii

Quiz Question: Which two organisms are notorious for causing problems in cycstic fibrosis patients? A) vibrio cholerae and Staphylococcus aureus B) Pseudomas aeruginosa and Helicobacter pylori C)Acinetobacter pyogenes and Pseudomonas stutzeri D) Pseudomonas aeruginosa and Burkholderia cepacia

Answer: D) Pseudomonas aeruginosa and Burkholderia cepacia

If a Group A negative mom received antenatal Rh Immune Globulin at 28 weeks gestation, what is the specificity of the antibody that may be identified in her serum at delivery?

Anti-D

Which of the following antibodies reacts best in an acidic environment

Anti-M

Which of the following antibodies reacts with the same strength through all dilutions of the serum until it becomes negative

Anti-Rg

What antibodies in the Rh system can be made by a person whose Rh phenotype is R1R1

Anti-c and anti-E

Antibody Maturation

Antibodies increase affinity, transition from membrane-bound to soluble, and switch Fc class / Isotope to needed type (variable effects).

Linear Epitope

Antibodies produced to a denatured antibody, may be for both or only one hidden and/or exposed epitope in the intact antigen, which it might not bind.

Opsonization

Antibody effect, tags antigen and signals macrophages to phagocytose it.

Conformational Epitope

Antibody produced only recognizes an epitope on the folded antibody, which is lost when denatured.

Neoantigenic Epitope

Antibody produced to a lysed antibody, epitope is only present in the lysed version, antibody will not bind the intact antibody.

Dimeric IgA

Antibody secreted into and stays in mucosa, has a J-chain and Secretory Component, highest produced antibody, also used in milk, for neutralization, blockage of attachment, and neonatal intestinal immunity.

Antiglobulin

Antibody that targets another antibody.

IgD

Antibody, B-Cell surface receptor with a regulatory function for cell's development, not secreted.

IgM

Antibody, lasts for 5 days, pentamer with five Fab regions (ten binding sites), Heavy Chain has five domains, held together by a J-chain, many carbs, largest of the antibodies.

IgG

Antibody, smallest and most common in circulation with the longest lifespan (3 wks), Heavy Chain has four domains, different isotopes have varying Hinge Regions, binds cellular Fc receptors after binding its antigen.

ADCC

Antibody-Dependent Cell-Mediated Cytotoxicity, antibody effect that signals leukocytes to destroy the bound antigen/cell (includes Killer Cells).

elevated potassium, decreased calcium

Anticoagulation with EDTA/Potassium Oxalate, or Citrate, despite tube top color, should be suspected if this pattern is seen:

Defensin

Antimicrobial peptide, used in epithelia within GI, respiratory tracts, and within Phagocytes. Polymerizes into a transmembrane channel like C9.

bilirubin, creatinine, uric acid

Any chemical that competes for peroxidase activity will interfere with the glucose oxidase method, especially:

protected health information

Any information which can lead to identification of a patient is termed:

outlier

Any value which falls >3σ from the mean is a(n):

12

At 5 days, total bilirubin should be <___ mg/dL.

7

At 7 days, total bilirubin should be <___ mg/dL.

transferrin

At a pH of 6.0, _____ will dump all bound iron, allowing it to be assayed.

20

At a pH of 7.4, the molecular ratio of bicarbonate to carbonic acid is:

4.5

At birth, total bilirubin should be <___ mg/dL.

1

At one month, total bilirubin should be <___ mg/dL.

A person who donates blood with the intent that he will receive his own blood during a future surgery is called what kind of donor

Autologous

Bursa Fabricius

Avian primary lymphoid tissue, for B Cell maturation.

If a WBC count is performed on a 1:10 dilution and the number of cells counted in 8 squares is 120, the total WBC count is: A. 1,200/µL (1.2 x 10^9/L) B. 1,500/µL (1.5 x 10^9/L) C. 12,000/µL (12.0 x 10^9/L) D. 15,000/µL (15.0 x 10^9/L)

B. 1,500/µL (1.5 x 10^9/L)

What is the MCHC if the Hct is 20%, the RBC is 1.5 x 10^6/µL (1.5 x 10^12/L) and the Hgb is 6 g/dL (60 g/L)? A. 28% B. 30% C. 40% D. 75%

B. 30%

63. What magnitude elevation of virus-specific IgG titer is usually considered the minimum for the diagnosis of an acute viral infection? A. 2 fold B. 4 fold C. 10 fold D. 100 fold E. 1 million billion fold

B. 4 fold. Paired sera collected 7-10 days apart is used to measure the elevation of IgG titers between convalescence and acute illness. Elevation of 4-fold is considered strong evidence for an acute infection, while a single

Given the following data: -WBC: 8.5 x 10^3/µL (8.5 x 10^9/L) Differential: -Segs: 56% -Bands: 2% -Lymphs: 30% -Monos: 6% -Eos: 6% What is the absolute eosinophil count? A. 170/µL (0.17 x 10^9/L) B. 510/µL (0.51 x 10^9/L) C. 2,550/µL (2.55 x 10^9/L) D. 4,760/µL (4.76 x 10^9/L)

B. 510/µL (0.51 x 10^9/L)

If a WBC count is performed on a 1:100 dilution and the number of cells counted in 8 squares is 50, the total WBC count is: A. 5,000/µL (5.0 x 10^9/L) B. 6,250/µL (6.25 x 10^9/L) C. 50,000/µL (50.0 x 10^9/L) D. 62,500/µL (62.5 x 10^9/L)

B. 6,250/µL (6.25 x 10^9/L)

A total leukocyte count is 10.0 x 10^3/µL (10.0 x 10^9/L) and 25 NRBCs are seen per 100 leukocytes on the differential. What is the corrected leukocyte count? A. 2,000/µL (2.0 x 10^9/L) B. 8,000/µL (8.0 x 10^9/L) C. 10,000/µL (10.0 x 10^9/L) D. 12,000/µL (12.0 x 10^9/L)

B. 8,000/µL (8.0 x 10^9/L)

What is the if the Hct is 20%, the RBC is 2.4 x 10^6/µL (2.4 x 10^12/L), and the Hgb is 5 g/dL (50 g/L)? A. 68 pg B. 83 pg C. 100 pg D. 120 pg

B. 83 pg

122. Which organism causes granulomatous amebic encephalitis? A. Naegleria B. Acanthamoeba C. Entamoeba histolytica D. Entamoeba hartmanii E. Iodamoeba

B. A . Acanthamoeba is known for causing GAE as well as a severe keratitis in contact lens users who make their own contact lens cleaning solution or use tap water. QCCP2, Acanthamoeba

175. Which species is responsible for the majority of cases of aspergilloma and allergic bronchopulmonary aspergillosis? A. A. flavus B. A. fumigatus C. A. niger D. A. terreus E. P. marneffei

B. A. There are 3 principle pulmonary diseases caused by Aspergillus spp, primarily A. fumigatus: Aspergilloma, a fungal ball that grows in the site of a pre-existing cavitary lesion; Allergic bronchopulmonary aspergillosis, an exaggerated allergic response to noninvasive Aspergillus colonization that's seen mostly in patients with cystic fibrosis; and finally, invasive aspergillosis, a vascular-invasive disease seen mostly in the immunocompromised. QCCP2, Aspergillus

141. Which nematode has a characteristic mammilated bile-stained egg? A. Trichuris B. Ascaris C. Necator D. Ancylostoma E. Strongyloides

B. A. Without the unique outer shell, Ascaris eggs resemble those of hookworms or Strongyloides. Ascaris is also significant for its lifecycle - eggs are ingested, hatch in the intestine, migrate to the lungs, and, after being expectorated, are in turn swallowed and take up residence in the duodenum. During the lung stage, they can cause the hypereosinophilic Loeffler syndrome. QCCP2, Ascaris

Refer to the following illustration: Which area of the automated cell counter histogram indicates lymphocyte curve? A. A B. B C. C D. D

B. B

Which curve represents the production of alpha polypeptide chains of hemoglobin? A. A B. B C. C D. D

B. B

128. The only medically significant ciliate organism is: A. Entamoeba histolytica B. Balantidium coli C. Cryptosporidium parvum D. Chilomastix mesneli E. Cyclospora cayetenensis

B. B . A 50-70 micron organism with circumferential ciliation and a lenticulate nucleus, B. coli is the most medically significant ciliate (and the only one you probably have to be aware of). QCCP2,Ciliates

The atypical lymphocyte seen in the peripheral smear of patients with infectious mono is probably derived from which of the following? A. T lymphocytes B. B lymphocytes C. monocytes D. mast cells

B. B lymphocytes

169. The mold form of Paracoccidioides is identical to that of which other fungus? A. Histoplasma B. Blastomyces C. Coccidioides D. Sporothrix E. Chrysosporium

B. B. Paracoccidioides has been called the "South American Blastomyces" because it looks and behaves very similarly to Blastomyces. QCCP2, Paracoccidioides

157. All of the following are dimorphic fungi, except: A. Histoplasma B. Cryptococcus C. Coccidioides D. Blastomyces E. Paracoccidioides

B. C. Cryptococcus does not have a mold form, only yeast. Therefore, unlike Histoplasma, Coccidioides, Blastomyces, Paracoccidioides, and Sporothrix, it is not one of the dimorphic yeasts. Other medically relevant fungi of the coccoid yeast-type, like Cryptococcus, include Torulopsis and Malassezia. QCCP2, T3.22, Classification of fungi

74. Which cell surface antigen is the receptor for the Epstein-Barr virus? A. P antigen B. CD21 C. insulin-degrading enzyme D. CD4 E. Cd81

B. CD21. The C3d receptor, or CD21, functions as the target for EBV cell entry. P antigen (associated with Donath-Landsteiner ab), paroxysmal nocturnal hemoglobinuria, neutralized by pigeon egg and hydatid cyst fluid) is the parvovirus receptor. Insulin-degrading enzyme is the putative receptor for varicella-zoster virus, while CD4 is the receptor for HIV. Recent studies suggest that CD81 may have a role as a receptor for HCV. QCCP2,EBV

70. Which of the following viruses is responsible for the most common congenital infection in the United States? A. VZV B. CMV C. parvovirus B19 D. EBV E. adenovirus

B. CMV. Passed transplacentally, the risk of in utero infection is greatest when the mother acquires a primary CMV infection while pregnant. The effects from in utero infection can range from severe to mild, with the most common manifestation of sensorineural hearing loss. QCCP2,CMV

If a patient presents with a prolonged APTT that does not correct upon mixing, the next performed should be: A. Factor II B. DRVVT C. Factor VIII D. plt count

B. DRVVT

Which of the following is associated with May-Hegglin anomaly? A. membrane defect of lysosomes B. Döhle bodies and giant plts C. two-lobed neutrophils D. mucopolysacchridosis

B. Döhle bodies and giant plts

117. Which of the following organisms can be detected with the "adhesive tape" test? A. Giardia lamblia B. Enterobius vermicularis C. Strongyloides stercoralis D. Onchocerca volvulus E. Brugia malayi

B. E. The adult female worm migrates to the anal verge at night to lay eggs. This accounts for the nocturnal anal pruritus that is a symptom of enterobial infection. Some of the eggs that are laid by the female worm can be collected with adhesive tape applied to the anus at night. QCCP2,Cellophane tape

Which one of the following factors typically shows an increase in liver disease? A. Factor VII B. Factor VIII C. Factor IX D. Factor X

B. Factor VIII

Patient presents with bleeding 48 hours post tooth extraction. Results are as follows: (see picture) Possible causes are a deficiency in: A. plasminogen B. Factor XIII C. alpha2 anti-plasmin D. Factor XII

B. Factor XIII

An enzyme deficiency associated with a moderate to sever hemolytic anemia after the patient is exposed to certain drugs and characterized by red cell inclusions from by denatured hemoglobin is: A. lactate dehydrogenase deficiency B. G-6-PD deficiency C. pyruvate kinase deficiency D. hexokinase deficiency

B. G-6-PD deficiency

162. Which of the following dimorphic fungi needs to be plated on brain-heart infusion media with blood in order to grow as a yeast? A. Blastomyces dermatidis B. Histoplasma capsulatum C. Coccidioides immitis D. Sporothrix schenkii E. Paracoccidioides brazilensis

B. H. Blastomyces grows as a yeast on cottonseed agar while Coccidioides requires specialized media to grow as a yeast. The rest of the dimorphic fungi readily grow as yeast. QCCP2,Laboratory methods

90. The persistence of which marker is the best evidence of chronic HBV infection: A. HBeAg B. HBsAg C. anti-HBe D. anti-HBs E. anti-HBc

B. HBsAg. The persistence of surface antigen is an excellent indicator of chronicity. With acute infection and resolution, surface antigen peaks 2-3 months post-infection and before symptoms, but is completely gone before 6 months, to be replaced by anti-surface antibody (HBsAb) and the clinical symptoms of acute hepatitis. In chronic infection, the surface antibody does not appear, while the surface antigen persists. QCCP2,F3.2 - 3.3

Neutropenia is NOT usually associated with: A. viral infections B. Hodgkin disease C. select antibiotics D. chemotherapy

B. Hodgkin disease

80. Which of the following antibodies is most helpful in distinguishing an acute EBV infection from a remote EBV infection? A. IgM anti-EA B. IgM anti-VCA C. IgG anti-EA D. IgM anti-VCC E. IgG anti-EE

B. IgM anti-VCA. Anti-viral capsid antibodies (VCA) are the only EBV antibodies with a high specificity for the acute phase of infection. This IgM antibody is the first antibody to appear with acute infection but quickly decreases in titer with time. IgM anti-VCA recedes to undetectable titers with convalescence but quickly reappears with virus reactivation. QCCP2,EBV

177. All of the following hyalinohyphomyces have conidia that occur in clusters, except: A. Acremonium B. Penicillium C. Gliocladium D. Fusarium E. all of the above occur in clusters

B. P . If anything called for a mnemonic, it would be all these genera of hyaline molds - a "GAF" is a tool to grab things and hold them together in a cluster. The organisms with clustered conidia are Gliocladium, Acremonium, and Fusarium. The branching chain conidia are a further reach - imagine looking through a pay telescope to see a pen hanging from a chain on a tree branch. Interpretation - the branching chain conidia are Penicillium, Paecilomyces, and Scopulariopsis. All the rest occur singly - Scedosporium, Beuveria, Sepedonium, and Chrysosporium. QCCP2, Hyalinohyphomyces

139. Which of the following is the most common AIDS-defining illness? A. cryptosporidiosis B. Pneumocystis pneumonia C. esophageal candidiasis D. Kaposi sarcoma E. toxoplasmosis

B. P pneumonia. PCP, or Pneumocystis carinii pneumonia, is the most common AIDS-defining illness. Now renamed Pneumocystis jiroveckii, the organism presents with a characteristic bilateral "bat-wing" distribution of lung opacity with frothy alveolar exudate. Within the exudate, the organisms can be seen either in relief (Wright-Giemsa) or as positivelystaining "crushed ping-pong balls" (GMS). QCCP2, Pneumocystis

104. What is the gold standard assay for the diagnosis of enteroviral meningitis in CSF samples? A. cell culture B. RT-PCR C. direct fluorescent antibody D. EIA E. suckling mouse paralysis assay

B. RT-PCR. As of now, RT-PCR is the most rapid and sensitive assay available for the detection of enterovirus in the CSF. However, backup culture should be done in tandem as well. DFA and EIA are not commonly used, and I just wanted an excuse to use the suckling mouse assay again, which you may recall can be used in the diagnosis of Coxsackie A, an enterovirus. QCCP2,Picornaviruses

51. What is/are the most common bacterium found in prosthetic joint infections? A. Staphylococcus aureus B. Staphylococcus epidermidis C. Streptococci spp D. gram negative bacilli E. Enterococcus

B. S. Coagulase-negative staph account for almost 1/2 of the cases of prosthetic joint infections, most often presenting months after surgery. This makes sense since coagnegative staph is a normal skin flora microbe and the majority of joint infections are due to the introduction of bacteria during surgery. On the other hand, since it is normal skin flora, coag-negative staph is a common cause of contamination of cultures. QCCP2,Prosthetic joint infections and other clinical syndromes

147. Which organism is predominantly responsible for visceral larva migrans? A. Onchocerca volvulus B. Toxocara canis C. Toxocara cati D. Ancylostoma brazilensis E. Dirofilaria immitis

B. T. Most cases of visceral larva migrans (and ocular larva migrans) are due to Toxocara canis. A small percentage is due to T. cati. Onchocerca causes corneal opacities, A. brazilensis causes cutaneous larva migrans, and Dirofilaria causes lung infection in humans and heart infections in dogs and cats. QCCP2, Toxocara canis & cati

171. Which of the following dermatophytes can be identified with its "birds on a wire" microconidia? A. Microsporidium canis B. Trichophyton rubrum C. Epidermophyton floccosum D. Trichophyton mentagrophytes E. Trichophyton tonsurans

B. T. The Trichophyton spp have microconidia and, of them, each has unique morphological features. T. rubrum has "birds on a wire" microconidia spaced along hyphae; T. tonsurans has widely variable microconidia, while T. mentagrophytes has grape-like clusters and occasional spiral hyphae. QCCP2,Dermatophytes

172. What organism is the most common cause of onychomycosis? A. Epidermophyton floccosum B. Trichophyton rubrum C. Trichophyton mentagrophytes D. Microsporum canis E. Microsporum gypseum

B. T. The Trichophyton spp, especially T. rubrum, cause the majority of cases of onychomycosis. Individuals with HIV and diabetes are at an increased risk for onychomycosis. In addition, it is important to note that other fungi, such as Candida, can cause onychomycosis. QCCP2,Dermatophytes

Evidence indicates the genetic defect in thalassemia usually results in: A. the production of abnormal glob in chains B. a quantitative deficiency in RNA resulting in decreased globin chain production C. a structural change in the heme portion of the hemoglobin D. an abnormality in the alpha- or beta-chain binding or affinity

B. a quantitative deficiency in RNA resulting in decreased globin chain production

125. What feature of Leishmania spp helps to distinguish them from Histoplasmaor Toxoplasma? A. small intracellular amastigotes B. bar-like kinetoplast C. central axostyle D. external flagellum E. extracellular forms

B. bar-like kinetoplast. Especially in the intracellular amastigote forms of the three organisms, the one feature of Leishmania that helps to distinguish it from the others is the bar-like kinetoplast adjacent to the nucleus. Specific culture media, such as Novy-MacNeal-Nicolle medium, can be used to isolate Leishmania where the flagellated promastigote form can be identified. QCCP2, Leishmania

44. Which feature separates the typeable from non-typeable strains of Haemophilus influenzae? A. HA antigen B. capsule C. mecA gene D. penicillin binding protein E. growth at 42°C

B. capsule. The presence of a capsule separates the typeable (capsulated) from the non-typeable (unencapsulated). Furthermore, the typeable strains are categorized into serotypes according to the type of capsule protein present - the most prevalent serotype being type B. QCCP2,Meningitis, specific agents

56. MRC-5 cells are an example of which kind of cell culture? A. primary cell culture B. cell line (secondary cell culture) C. established cell line D. human diploid fibroblast E. malignant transformed cell line

B. cell line (secondary cell culture). MRC-5, named for the Medical Research Council where they were first propagated, are a secondary cell line with limited viability. Cells don't survive after a number of transfers, unlike a primary cell culture, which can't be transferred at all, or an established cell line, which can be transferred nearly illimitably. Human diploid fibroblasts are another type of secondary cell culture. QCCP2,Viral lab methods

Increased numbers of basophils are often seen in: A. acute infections B. chronic myelocytic leukemia C. chronic lymphocytic leukemia D. erythroblastosis fetalis (HDN)

B. chronic myelocytic leukemia

Which of the following lab findings is associated with Factor XIII deficiency? A. prolonged activated partial thromboplastin time B. clot solubility in a 5 molar urea solution C. prolonged thrombin time D. prolonged prothrombin time

B. clot solubility in a 5 molar urea solution

160. Which of the following media is most helpful in distinguishing the morphology of yeasts? A. cottonseed agar B. cornmeal agar with Tween 80 C. urea agar D. brain-heart infusion medium E. potato dextrose agar

B. cornmeal agar with Tween 80. Cottonseed agar is used specifically to convert the mold phase of Blastomyces to the yeast form. Cornmeal with Tween 80 stimulates conidiation and chlamydospore production, aiding in speciation. Urea agar is helpful in detecting urease which is produced by Cryptococcus neoformans. Brain-heart infusion is a non-selective medium which will support the growth of saprophytic and pathogenic fungi. Potato dextrose agar is useful in demonstrating the production of pigment by Trichophyton rubrum. QCCP2,T3.21, Special fungal culture techniques

If a blood smear is dried too slowly, the RBCs are often: A. clumped B. crenated C. lysed D. destroyed

B. crenated

Which of the following will not cause erroneous results when using a phase optical system for enumerating plts? A. incipient clotting B. decreased Hct C. Howell-Jolly bodes D. leukocyte cytoplasmic fragments

B. decreased Hct

An increased amount of cytoplasmic basophilia in a blood cell indicates: A. increased cytoplasmic maturation B. decreased cytoplasmic maturation C. reduction in size of the cell D. deceased nuclear maturation

B. decreased cytoplasmic maturation

The most appropriate screening test for hemoglobin S is: A. Kleihauer-Betke B. dithionite solubility C. osmotic fragility D. sucrose hemolysis

B. dithionite solubility

86. Benign HPV-associated lesions are associated with this form of the virus: A. integrated B. episomal C. mitochondrial D. extracellular E. intralysosomal

B. episomal. In most benign HPV lesions, the virus DNA is maintained in circular extrachromosomal episomes. The process of integration into the host genome by HPV is more often associated with malignant disease. QCCP2,HPV

Aliquots of plasma with a prolonged PT and prolonged APTT are mixed using various ratios of patient plasma and normal plasma. All samples are incubated at 37ºC and tested at 10-,30-, and 60-minute intervals. The PT and APTT results on all of the mixtures are corrected. These results would indicate the presence of: A. circulating anticoagulant B. factor deficiency C. contaminated reagent D. antibodies

B. factor deficiency

150. What is the primary vector for Schistosoma? A. Aedes mosquitoes B. freshwater snails C. freshwater fish D. freshwater plants E. pork

B. fresh water snails. The cercariae of Schistosoma infest fresh water snails in water where they can freely swim. The cercariae penetrate the skin and migrate to the blood vessels of the bladder (S. haematobium), liver (S. mansoni, japonicum, mekongii) or intestine (S. intercalatum). QCCP2,Schistosoma

Phagocytosis is a function of: A. erythrocytes B. granulocytes C. lymphocytes D.thrombocytes

B. granulocytes

53. What is the overall most common cause of bacterial cellulitis? A. coagulase-negative Staphylococcus B. group A Streptococcus C. Pasteurella multocida D. Aeromonas hydrophila E. Vibrio vulnificus

B. group A S Coag-negative strep causes many cases of post-traumatic cellulitis; Pasteurella is associated with animal bites, Aeromonas with fresh water and V. vulnificus with salt water exposure. QCCP2,T3.2

The M:E ration in acute myelocytic leukemia is usually: A. normal B. high C. low D. variable

B. high

The M:E ration in chronic myelocytic leukemia is usually: A. normal B. high C. low D. variable

B. high

The most common cause of error when using automated cell counters is: A. contamination of the diluent B. inadequate mixing of the sample prior to testing C. variation in voltage of the current supply D. a calibrating error

B. inadequate mixing of the sample prior to testing

After the removal of RBCs from the circulation hemoglobin is broken down into: A. iron, porphyrin, and amino acids B. iron, protoporphyrin, and glob in C. heme, protoporphyrin, and amino acids D. heme, hemosiderin, and globin

B. iron, protoporphyrin, and globin

145. What is the most characteristic feature of Enterobius worms in histological tissue sections? A. prominent genital groove B. lateral alae C. short buccal opening D. flattened uterine branches E. sheathed tail

B. lateral alae. The lateral alae should be searched for especially in cases of appendicitis, which is a common location to find the adult worm. Eggs can be identified via the cellophane tape test. QCCP2, Enterobius

120. What is the most common site of extraintestinal amebic abscess? A. lungs B. liver C. brain D. spleen E. bladder

B. liver. The gray-white purulence or anchovy paste amebic abscess is most often in the liver, though the brain and spleen are not uncommon sites for abscess formation. QCCP2, E. histolytica

In an uncomplicated case of infectious mononucleosis, which of the following cells are affected? A. erythrocytes B. lymphocytes C. monocytes D. thrombocytes

B. lymphocytes

All of the following condition are myeloproliferative disorders EXCEPT: A. myelocytic leukemia B. lymphocytic leukemia C. polycythemia vera D. idiopathic thrombocythemia

B. lymphocytic leukemia

The cytoplasmic abnormality of the WBC of Alder-Reilly anomaly is found in the: A. endoplasmic reticulum B. lysosomes C. mitochondria D. ribosomes

B. lysosomes

In an automated cell counter, the WBC printed result is "+++". The next step is to: A. repeat after warming the sample to 37ºC B. make an appropriate dilution of the sample C. recalibrate the machine from pooled samples D. request a new sample immediately

B. make an appropriate dilution of the sample

In the image below, the small nucleated cell seen in the lower left corner is a: A. polychormatophilic normoblast (rubricyte) B. mature lymphocyte C. plasma cell D. lymphoblast

B. mature lymphocyte

94. Which of the following scenarios might explain a conflicting set of results in a patient with a positive anti-HCV antibody result and negative HCV RNA PCR? A. no infection B. no infection or recovery from acute HCV C. early HCV D. chronic HCV E. infection

B. no infection or recovery from acute HCV. A positive anti-HCV could be either a false positive, in the case of non-infection, or it could indicate the patient was one of the 15% of cases where HCV did not develop chronic infection. QCCP2,HCV

81. All of the following EBV-associated lesions have demonstrable EBER (EBV-encoded RNA) by in situ hybridization, except: A. Hodgkin lymphoma B. oral hairy leukoplakia C. primary effusion lymphoma D. post-transplant lymphoproliferative disorder E. nasopharyngeal carcinoma

B. oral hairy leukoplakia. In addition to tumors and pre-tumoral lesions, EBER can be shown in the reactive hyperplastic lymphoid tissue of infectious mononucleosis. In some tumors, it is not strongly expressed (~1/2 of Hodgkin disease cases are positive) versus other tumors where it is very strongly expressed (nasopharyngeal carcinoma, NK-T cell lymphoma -nasal type, and endemic-type Burkitt lymphoma). QCCP2,EBV

Which of the following cell types is characteristic of Pelger-Huët anomaly is the: A. band form B. pince-nez form C. normal neutrophil D. myelocyte

B. pince-nez form

The following results were obtained on a 55 y/o man complaining of headaches and blurred vision: -WBC: 19.0 x 10^3/µL (19.0 x 10^9/L) -RBC: 7.2 x 10^6/µL (7.2 x 10^12/L) -Plt: 1,056 x 10^3/µL (1056 x 10^9/L) -uric acid: 13.0 mg/dL (0.76 mmol/L) -O2 saturation: 93% -Rh^1: negative -Red cell volume: 3,911 mL (normal: 1,600) Differntial: -Segs: 84% -Bands: 10% -Lymphs: 3% -Monos: 2% -Eos: 1% The results are consistent with: A. neutrophilic leukemoid reaction B. polycythemia vera C. chronic myelocytic leukemia D. leukoerythroblastosis in myelofibrosis

B. polycythemia vera

Which of the following types of polycythemia is most often associated with emphysema? A. polycythemia vera B. polycythemia, secondary to hypoxia C. relative polycythemia associated with dehydration D. polycythemia associated with renal disease

B. polycythemia, secondary to hypoxia

The main function of the hexose monophosphate shunt in the erythrocyte is to: A. regulate the level of 2,3-DPG B. provide reduced glutathione to prevent oxidation of hemoglobin C. prevent the reduction of heme iron D. provide energy for membrane maintainance

B. provide reduced glutathione to prevent oxidation of hemoglobin

Which of the following characteristics are common to hereditary spherocytosis, hereditary elliptocytosis, hereditary stomatocytosis, and paroxysmal nocturnal hemoglobinuria? A. autosomal dominant inheritance B. red cell membrane defects C. positive direct antiglobulin test D. meausre PLT count

B. red cell membrane defects

A citrated blood specimen for coag studies is to be collected from a polycythemic patient. The anticoagulant should be: A. the standard volume B. reduced in volume C. changed to EDTA D. changed to oxalate

B. reduced in volume

A phase-platelet count is performed using a plt Unopette (dilution=1:100). 155 plts are counted on one side of the hemacytometer in the center square mm, and 145 counted on the other side in the same area. After making the appropriate calculations, the next step would be to: A. repeat the procedure, using a 1:20 dilution with acetic acid B. report the calculated value C. collect a new specimen D. repeat the procedure, using a 1:200 dilution with saline

B. report the calculated value

Using the supra vital stain, the polychromatic RBCs below would probably be: A. rubricytes (polychromatophilic normoblasts) B. retics C. sickle cells D. target cells

B. retics

The characterisc peripheral blood morphologic feature in multiple myeloma is: A. cytotoxic T cells B. rouleaux formation C. spherocytosis D. macrocytosis

B. rouleaux formation

135. Which form of Plasmodium is described as intraerythrocytic collections of numerous organisms? A. merozoites B. schizonts C. hypnozoites D. trophozoites E. bradyzoites

B. schizonts. The schizont is the form that contains numerous merozoites within the erythrocyte. With the rupture of the schizont, merozoites are released to infect other RBCs. The hemolysis corresponds to the clinical fever spikes. QCCP2, Plasmodium

75. All of the following clinical syndromes are caused by EBV, except: A. infectious mononucleosis B. shingles C. primary effusion lymphoma D. post-transplant lymphoproliferative disease E. oral hairy leukoplakia

B. shingles. Shingles is associated with VZV infection. There are myriad clinical syndromes associated with EBV infection, especially long-term latent effects. In addition to the conditions presented above, primary CNS lymphoma, Burkitt lymphoma, Hodgkin lymphoma, lymphomatoid granulomatosis, and nasopharyngeal carcinoma are all associated with latent EBV infection. QCCP2,T3.9

59. What's the most common cause of false positive hemadsorption in viral culture? A. Mycoplasma spp B. simian virus C. mycobacterium D. Acanthamoeba E. Candida

B. simian virus. SV is a very common cause of false positive hemabsorption results. An uninoculated control should be run concurrent with the patient sample as a negative control. Another common contaminant is Mycoplasma, which can cause poor growth in cell lines and decreased infective potential of the viruses. QCCP2,Virology, lab methods

Which of the following technical factors will cause a decrease erythrocyte sedimentation rate (ESR)? A. gross hemolysis B. small fibrin clots in the sample C. increased room temp D. tilting of the tube

B. small fibrin clots in the sample

Erythropoieten acts to: A. shorten the replication of the granulocytes B. stimulate RNA synthesis of erythroid cells C. increase colony-stimulating factors produced by the B-lymphocytes D. decrease the release of marrow retics

B. stimulate RNA synthesis of erythroid cells

A patient has pancytopenia, decreased total serum iron, decreased serum iron binding capacity, and shows a homogenous fluorescence pattern with a high titer on a fluorescent anti-nuclear antibody test. This is suggestive of: A. polycythemia vera B. systemic lupus erythematosus C. iron deficiency anemia D. Hgb SC disease

B. systemic lupus erythematosus

A patient has been taking aspirin regularly for arthritic pain. Which of the following tests is most likely to be abnormal in this patient? A. plt count B. template bleeding time C. protheombin time D. activated partial thromboplastin time

B. template bleeding time

91. To what does the "window period" of HBV infection refer? A. the period between the appearance of HBV DNA and infection B. the period between the disappearance of HBsAg and appearance of HBsAb C. the period between the disappearance of HBcAg and appearance of HBcAb D. the period between HBeAg and HBcAg E. the period between infection and IgM anti-HBcAb

B. the period between the disappearance of HBsAg and appearance of HBcAb. Less sensitive assays of the past may have missed the diagnosis of HBV if performed in the brief period around 5-6 months post-infection. Serologically, this corresponds to the period between the disappearance of the surface antigen and the appearance of the surface antibody. Nowadays, we can measure IgM anti-HBc during this time period and expect it to be positive. QCCP2,HBV

The best test to determine if a sample is contaminated with heparin is: A. fibrinogen B. thrombin time C. prothrombin time D. stypven time

B. thrombin time

151. Which of the following features of Taenia saginata helps to distinguish it from Taenia solium? A. pork tapeworm B. unarmed rostellum C. egg with a radially striated wall D. proglottid with less than 13 uterine branches E. cystercercosis

B. unarmed rostellum. The Taenia species have several features to distinguish them from each other. T. saginata is the beef tapeworm; T. solium is the pork tapeworm. T. saginata has an unarmed rostellum, unlike the armed rostellum of the pork tapeworm. The proglottid of the beef tapeworm has more than 13 uterine branches (as visualized by India ink injection), where the pork tapeworm has less than 13 branches. In addition, the pork tapeworm can cause cystercercosis, which the beef tapeworm doesn't. The eggs of both tapeworms are identical, with thick radially-striated walls. QCCP2,Cestodes

Lab tests preformed on a patient indicate macrocytosis, anemia, leukopenia and throbocytopenia. Which of the following disorders is the patient most likely to have? A. anemia of chronic disorder B. vitamin B12 deficiency C. iron deficiency D. acute hemorrhage

B. vitamin B12 deficiency

129. Where can Cryptosporidium be found in an infected host? A. adherent to small intestinal brush border B. within an intracellular apical vacuole C. interdigitated between enterocytes D. in foveolar gland crypts of the stomach E. intracytoplasmically within enterocytes

B. within an intracellular apical vacuole. On light microscopy, the Cryptosporidia appear to be small (8-15 micron), round organisms attached to the extracellular brush border. However, ultrastructural studies have determined that the organism is located in a unique position within an intracellular, yet extracytoplasmic apical vacuole. Very strange. Isospora interdigitates, Microsporidium is intracellular, and Strongyloides is found within the crypt epithelium. QCCP2,Cryptosporidiosis

renal function markers

BUN and Creatinine are used as:

hepatic disease

BUN is decreased in:

renal disease

BUN is increased in:

BUN to Urea conversion

BUN mg/dL X 2.14

7-18

BUN normal range is ___-___ mg/dL.

pre-renal azotemia

BUN rises faster than creatinine in:

Listeria

Bacteria, activates virulence genes when defending WBCs are detected and proliferates in Macrophage cytoplasm, must be killed quickly by M1 Macrophages, vaccines ineffective.

Gram Negative

Bacterial type, cell wall has two membranes, outer is Endotoxin-filled, strong PAMP signal (vaccine risk).

didymium

Bandpass in broad bandpass instruments should be checked using:

holmium oxide

Bandpass in narrow bandpass instruments should be checked using:

hemolytic anemia

Because HbA1C depends on a normal erythrocyte lifespan, the result is meaningless if the patient also has:

competitive

Because only the antibodies which bind to the probe-analyte appear as fluorescent, and not the antibodies bound to specimen-analyte, FPIA is this type of assay:

creation, destruction

Because uric acid is the end result of breakdown and creation of nucleotides, both the ______ and ______ of cells will lead to its elevation:

humidified

Before calibrator gases can be used, they must be:

Capillaria Plica

Bladder Worm

capillary blood

Blood gas analysis on infants is most often performed on:

electrodes

Blood gas analyzers use _____ to detect pH, pO2, and pCO2.

acid-base balance, oxygenation, metabolic control of bicarbonate

Blood gas testing evaluates these physiological functions:

dietary intake, renal excretion, bone metabolism

Blood levels of calcium are determined by:

dietary intake, renal excretion, bone metabolism

Blood levels of phosphates are determined by:

dietary intake, renal excretion, blood pH

Blood levels of potassium are determined by:

dietary intake, renal excretion, osmotic pressure

Blood levels of sodium are determined by:

Urine color = red is indicative of

Blood or food pigment

Arterial Blood Gas Values

Blood pH 7.35 to 7.45 pCO2 35 to 45 mmHg pO2 83 to 108 mmHg TCO2 22 to 29 mMol/L Bicarb 21 to 28 mMol/L BE +/- 3 mMol/L O2 Sat. 95% - 98%

Factor H

Body cell surface protein, acts to bind C3 for cleavage and inactivation, protects against the Alternative Pathway Complement Cascade.

Facor I

Body cell surface protein, acts to cleave and inactivate C3, protects against the Alternative Pathway Complement Cascade.

B/T Cell Zone

Border between the Follicular and Parafollicular zones in lymph nodes, site where activated IgM B-Cells and TH Cells are attracted via cytokines for binding and class switch.

growth hormone

Both gigantism and acromegaly are caused by overproduction of:

Bone Marrow Cytokines

Both innate and adaptive, includes IL3 and GM-CSF, upregulate WBC production (may act on stem cells or maturing lineage). Released by TH Cells in response to activated presenting B Cells. Part of Secondary Response, good target for drug therapy.

diazo reagents

Both the Evelyn-Malloy and Jendrassic-Grof methodologies use:

inversely proportional

Boyle's Law states that a volume of gas is ________ to the pressure applied to it.

hyperkalemia

Bradycardia progressing to a flatline EKG and cardiac arrest are indicative of:

respiratory system

Broadly, carbon dioxide increases in disorders of the:

silver ions

Buchler-Cotlove chloridometers measure chloride by titrating the sample with _______, which produce a precipitate when combined with chloride.

If a patient has a weakly expressed D antigen, how is it detected

By the indirect antiglobulin test (IAT)

A blood smear shows 80 nucleated red cells per 100 leukocytes. The total leukocyte count is 18 x 10^3/µL (18 x 10^9/L). The true white cell count expressed in SI units is: A. 17.2 x 10^3/µL (17.2 x 10^9/L) B. 9.0 x 10^3/µL (9.0 x 10^9/L) C. 10.0 x 10^3/µL (10.0 x 10^9/L) D. 13.4 x 10^3/µL (13.4 x 10^9/L)

C. 10.0 x 10^3/µL (10.0 x 10^9/L)

A patient has the following lab results: -RBC: 2.00x10^6/µL (2.00x10^9/L) -Hct: 24% -Hgb: 6.8 g/dL (68 g/L) -retic: 0.8% The mean corpuscular volume (MCV) of the patient is: A. 35 µm^3 (35 fL) B. 83 µm^3 (83fL) C. 120 µm^3 (120 fL) D. 150 µm^3 (150 fL)

C. 120 µm^3 (120 fL)

If the leukocyte count is 20.0 x 10^3/µL (20.0 x 10^9/L) and 50 NRBCs are seen per 100 leukocytes on the differential, what is the corrected leukocyte count? A. 6,666/µL (6.666 x 10^9/L) B. 10,000/µL (10.0 x 10^9/L) C. 13,333/µL (13.333 x 10^9/L) D. 26,666/µL (26.666 x 10^9/L)

C. 13,333/µL (13.333 x 10^9/L)

Given the following data: -WBC: 8.5 x 10^3/µL (8.5 x 10^9/L) Differential: -Segs: 56% -Bands: 2% -Lymphs: 30% -Monos: 6% -Eos: 65 What is the absolute lymphocyte count? A. 170/µL (0.17 x 10^9/L) B. 510/µL (0.51 x 10^9/L) C. 2,550/µL (2.55 x 10^9/L) D. 4,760/µL (4.76 x 10^9/L)

C. 2,550/µL (2.55 x 10^9/L)

Blood is diluted 1:200, and a plt count is performed. 180 plts were counted in the center square mm on one side of the hemacytometer and 186 on the other side. The total plt count is: A. 146 x 10^3/µL (146 x 10^9/L) B. 183 x 10^3/µL (183 x 10^9/L) C. 366 x 10^3/µL (366 x 10^9/L) D. 732 x 10^3/µL (732 x 10^9/L)

C. 366 x 10^3/µL (366 x 10^9/L)

What is the MCH if the Hct is 20%, the RBC is 1.5 x 10^6/µL (1.5 x 10^12/L) and the Hgb is 6 g/dL (60 g/L)? A. 28 µm^3 (28 fL) B. 30 µm^3 (30 fL) C. 40 µm^3 (40 fL) D. 75 µm^3 (75 fL)

C. 40 µm^3 (40 fL)

The mean value of a retic count on specimens of cord blood from healthy, full-term newborns is approximately: A. 0.5% B. 2.0% C. 5.0% D. 8.0%

C. 5.0%

173. Which of the following organisms is characterized by blue-green colonies with a white apron? A. Aspergillus terreus B. A. niger C. A. fumigatus D. A. flavus E. Penicillium marneffei

C. A. Each of the Aspergillus spp has a distinct appearance on plates. A. fumigatus is blue-green, A. terreus has a cinnamon-buff colony, A. niger has a black colony, and A. flavus has a brown colony with lateral striations. QCCP2, Aspergillus

176. Which species of Aspergillus is most commonly seen as otitis externa? A. A. flavus B. A. fumigatus C. A. niger D. A. terreus E. P. marneffei

C. A. QCCP2, Aspergillus

In an uncomplicated case of sever iron deficiency anemia, which of the following sets represents the typical patter of results A. A B. B C. C D. D

C. C

Which curve represents the production of gamma polypeptide chains of hemoglobin? A. A B. B C. C D. D

C. C

166. Which of the following dimorphic fungi presents the highest risk to laboratory personnel during culture? A. Histoplasma B. Blastomyces C. Coccidioides D. Sporothrix E. Paracoccidioides

C. C. The mold form of Coccidioides forms barrel-shaped arthroconidia of live cells alternating with empty shells. The shell portions break easily, releasing the infective arthroconidia into the air where they can be freely inhaled unless proper precautions are taken. QCCP2,Dimorphic fungi and molds

Patient results are as follows: (see picture) This workup suggests: A. blood clot B. hemorrhage C. DIC D. HUS

C. DIC

170. Which of the following dermatophytes is identified by its macroconidia? A. Trichophyton tonsurans B. Trichophyton rubrum C. Epidermophyton floccosum D. A & B E. A, B, C

C. E. Trichophyton species (T. rubrum, T. tonsurans, T. mentagrophytes) are identified by their microconidia, while E. floccosum and the Microsporum spp are identified by their macroconidia. E. floccosum has "beaver tail" macroconidia with transverse septae; Microsporum canis has transverse septa also, but with serrated edges ("dog teeth") and pointed ends. Microsporum gypseum is very similar to M. canis but without the serrations. QCCP2,Dermatophyte

148. Which organism has the largest egg? A. Clonorchis B. Diphyllobothrium C. Fasciola hepatica D. Paragonimus E. Trichuris

C. F. Fasciola hepatica (and buskii) have the largest human parasite eggs known (~150 microns in diameter). The eggs have a non-shouldered operculum and lack an abopercular knob. QCCP2,T3.20

99. Which of the following influenza subtypes is associated with pandemic avian influenza? A. H1N1 B. H5N2 C. H5N1 D. H2N1 E. H2N2

C. H5N1. There are currently over 15 H and 9 N subtypes, with the numbers growing steadily due to antigenic shifts. Birds are the more common reservoir for influenza, for example, the chickens in S.E. Asian H5N1 avian influenza. The virus can cause an ARDS clinical appearance with a 50% mortality, predominantly affecting teenagers. QCCP2,Orthomyxoviruses

92. Which HBV antigen has been used to characterize chronic HBV carriers as either replicative or non-replicative? A. HBc B. HBs C. HBe D. HBx E. HBp

C. HBe. In the past, HBe antigen was used to classify patients. However, there are issues with the consistency of HBe serology, so now the measurement of HBV DNA copy number has replaced HBe. A copy number of >105/mL is needed to classify a chronic HBV carrier as "replicative." QCCP2,HBV molecular assays

97. Which of the following hepatitis viruses requires coinfection or chronic infection with HBV in order to infect the liver? A. HAV B. HCV C. HDV D. HEV E. HGV

C. HDV. HDV can only infect hepatocytes that are infected with HBV. Coinfection of patients with HBV and HDV results in a more severe disease and significantly increases the risk of cirrhosis and hepatic failure. QCCP2,HBV

Coagulation factors affected by warfarin (Coumadin) drugs are: A. VIII, IX and X B. I, II, V, and VII C. II, VII, IX and X D. II, V and VII

C. II, VII, IX and X

The most appropriate screen test for detecting hemoglobin F is: A. osmotic fragility B. dithionite solubility C. Kleihauer-Betke D. Heat instability test

C. Kleihauer-Betke

126. Which species of Leishmania is most commonly associated with mucocutaneous leishmaniasis? A. L. major B. L. tropica C. L. brazilensis D. L. donovani E. L. mexicana

C. L. A number of species of Leishmania are pathogenic, often causing distinct disease entities. L. tropica and L. major are associated with solitary cutaneous lesions, while L. donovani is associated with systemic disease (Kala-azar). L. mexicana is the causative agent of the self-limiting Chiclero ulcer of the ear lobe, while L. brazilensis causes mucosal and cutaneous lesions. QCCP2, Leishmania

46. Which of the following organisms typically causes meningitis in a disproportionate number of both very young and very old patients? A. group B Streptococcus B. Staphylococcus aureus C. Listeria monocytogenes D. Streptococcus pneumoniae E. Haemophilus influenzae

C. L. Besides age (<1 month and older than 70 years), predisposing factors toward the development of Listeria meningitis include diabetes and immunosuppression (steroids, HIV, and transplantation). QCCP2,Meningitis, specific agents

Of the following, the disease most closely associated with pale blue inclusions in granulocytes and giant plts is: A. Gaucher diease B. Alder-Reilly anomaly C. May-Hegglin anomaly D. Pelger-Huët anomaly

C. May-Hegglin anomaly

The values below were obtained on an automated blood count system performed on a blood sample from a 25 y/o man: These results are most consistent with which of the following? A. megoblastic anemia B. hereditary sphericytosis C. a high tiger of cold agglutinins D. an elevated retic count

C. a high tiger of cold agglutinins

A plt count done by phase microscopy is 200 x 10^3/µL (200 x 10^9/L) (reference range 150-450 x 10^9/L). A standardized template bleeding time on the same person is 15 minutes (reference range 4.5 ± 1.5 minutes). This indicates that: A. the Duke method should have been used for the bleeding time B. the manual plt count is in error C. abnormal plt function should be suspected D. the results are expected

C. abnormal plt function should be suspected

A leukocyte count and differential on a 40 y/o Caucasian man revealed: -WBC: 5.4 x 10^3/µL (5.4 x 10^9/L) Differential: -Segs: 20% -Lymphs: 58% -Monos: 20% -Eos: 2% This data represents: A. absolute lymphocytosis B. relative neutrophilia C. absolute neutropenia D. leukopenia

C. absolute neutropenia

142. What's the best way to distinguish Necator americanus from Ancylostoma duodenale? A. appearance of eggs B. appearance of long buccal cavity C. appearance of mouth parts D. absence of genital groove E. length of worm

C. appearance of mouth parts. Necator has cutting plates (the "Necator grater") while Ancylostoma has teeth. Otherwise, the other features presented in the question are remarkably similar in both organisms. Other difference not mentioned include the geographic distribution - A duodenale is not present in the U.S. QCCP2,Hookworms

Which of the following is most likely to be seen in lead poisoning? A. iron overload in tissue B. codocytes C. basophilic stippling D. ringed sideroblasts

C. basophilic stippling

The following results were obtained on a 35 y/o woman complaining of fatigue and weight loss: -WBC: 1.8 x 10^3/µL (1.8 x 10^9/L) -RBC: 4.6 x 10^6/µL (4.6 x 10^12/L) -Plt: 903 x 10^3/µL (903 x 10^9/L) -uric acid: 6.4 x mg/dL (380 µmol/L) -LAP: 0 -Philadelphia chromosome: positive Differential: -Segs: 30% -Bands: 17% -Lymphs: 13% -Monos: 3% -Eos: 4% -Basos: 6% -Metamyelos: 3% -Myelos: 20% -Promyelos: 3% -Blasts: 1% These results are consistent with: A. neutrophilic leukemoid reaction B. idopathic thrombocythemia C. chronic meylocytic leukemia D. leukoerythroblastosis in myelofibrosis

C. chronic meylocytic leukemia

136. All of the following features of Plasmodium falciparum are helpful in distinguishing it from P. vivax or P. ovale, except: A. multiple ring forms within an erythrocyte B. applique trophozoite forms C. enlarged infected red blood cells D. banana-shaped gametocytes E. double chromatin dots in ring forms

C. enlarged infected RBCs. Unlike P. ovale and P. vivax, P. falciparum-infected RBCs tend not to be enlarged. Schuffner dots are also a characteristic of P. vivax and P. ovale. QCCP2, Plasmodium

72. All of the following techniques are routinely used to diagnose CMV, except: A. culture B. PCR C. flaccid paralysis in suckling mice injected with patient serum D. direct fluorescent antibody E. serology

C. flaccid paralysis in suckling mice after injection. In addition to tissue histology looking for the characteristic nuclear owl eye, inclusions can be performed if diagnostic tissue is available. Observation of flaccid paralysis in suckling mice is used to diagnose Coxsackie A virus. QCCP2,CMV

Excess D-dimer indicate that clots have been: A. converted to fibrin monomers B. released into circulation C. formed and are being excessively lysed D. stimulated to activate plts

C. formed and are being excessively lysed

In flow cytometry analysis, right angle (90 degrees) or side scatter of a laster light beam provides information that pertains to a cell's: A. volume B. viability C. granularity D. lineage

C. granularity

Which one of the following statements concerning Vitamin K is NOT true? A. there are 2 sources of Vitamin K: vegetable and bacterial B. Vitamin K converts precursor molecules into functional coagulation factors C. heparin inhibits the action of Vitamin K D. Vitamin K is fat soluble

C. heparin inhibits the action of Vitamin K

Which of the following cells contain hemosiderin? A. megakaryocyte B. osteoclast C. histiocyte D. mast cell

C. histiocyte

The white cell feature most characteristic of pernicious anemia is: A. eosinophilia B. toxic granulation C. hypersegmentation D. atypical lymphocytes

C. hypersegmentation

The absence of intermediate maturing cells between the blast and mature neutrophil commonly seen in acute myelocytic leukemia and myelodysplastic syndromes is called: A. subleukemia B. aleukemic leukemia C. leukemic hiatus D. leukemoid reaction

C. leukemic hiatus

Elevation of the total white count above 12 x 10^3/µL (12 x 10^9/L) is termed: A. relative lymphocytosis B. absolute lymphocytosis C. leukocytosis D. relative neutrophilic leukocytosis

C. leukocytosis

A term that means carrying degrees of leukocytosis with a shift to the left and occasional nucleated red cells in the peripheral blood is: A. polycythemia vera B. erythroleukemia C. leukoerythroblastosis D. megalobastoid

C. leukoerythroblastosis

Which of the following is true of acute lymphoblastic leukemia (ALL)? A. occurs most commonly in children 1-2 years of age B. patient is asymptomatic C. massive accumulation of primitive lymphoid-appearing cells in bone marrow occurs D. children under 1 years of age have a good prognosis

C. massive accumulation of primitive lymphoid-appearing cells in bone marrow occurs

Abnormalities found in erythroleukemia include: A. rapid DNA synthesis B. marrow fibrosis C. megaloblastic development D. increased erythrocyte survival

C. megaloblastic development

In synovial fluid, the most characteristic microscopic findings in gout is: A. calcium pyrophosphate crystals B. cartilage debris C. monosodium urate crystals D. hemosiderin-laden macrophages

C. monosodium urate crystals

Which of the following is characteristic of plt disorders? A. deep muscle hemorrhages B. retroperitoneal hemorrhages C. mucous membrane hemorrhages D. severely prolonged clotting times

C. mucous membrane hemorrhages

The large cell indicated by the arrow in the image below is a: A. meyloblast B. promyelocyte C. myelocyte D. metamyelocyte

C. myelocyte

Which of the following is associated with pseuod-Pelger-Huët anomaly? A. aplastic anemia B. iron deficiency C. myelogenous leukemia D. Chédiak-Higashi syndrome

C. myelogenous leukemia

Which of the following is used for staining retics? A. Giemsa stain B. Wright stain C. new methylene blue D. Prussian blue

C. new methylene blue

The flooding results were obtained on a patient's blood: -Hgb: 11.5 g/dl (115 g/L) -Hct: 40% -MCV: 89 µm^3 (89 fL) -MCH: 26 pg -MCHC: 29% Examination of a Wright-stained smear of the same sample would most likely show: A. macrocytic, normochromic erythrocytes B. microcytic, hypochromic erythrocytes C. normocytic, hypochromic erythrocytes D. normocytic, normochromic erythrocytes

C. normocytic, hypochromic erythrocytes

Which of the following is characteristic of cellular changes as megakaryoblasts mature into megakaryocytes within the bone marrow? A. progressive decrease in overall cell size B. increasing basophilia of cytoplasm C. nuclear division without cytoplasmic division D. fusion of the nuclear bodies

C. nuclear division without cytoplasmic division

Refer to the following pattern: Which pattern is consistent with beta-thalassemia major? A. patter A B. pattern B C. pattern C D. pattern D

C. pattern C

In the APTT test, the patient's plasma is mixed with: A. ADP and Ca B. tissue thromboplastin and collagen C. phospholipid and Ca D. tissue thromboplastin and Ca

C. phospholipid and Ca

The ideal capillary blood collection site on a new born site is: A. tip of the thumb B. ear lobe C. plantar surface of the heel D. the great toe

C. plantar surface of the heel

114. Which of the following HIV tests is the assay for determining response to anti-retrovirals? A. serum ELISA B. Western blot C. quantitative HIV RNA D. CD4 count E. p24 antigen detection

C. quantitative HIV RNA. Quantitative HIV RNA is an assessment of viral load. Quantitation of RNA levels is an excellent prognostic tool to predict progression, especially long-term. QCCP2,HIV

115. Which test is the best choice for the detection of HIV infection shortly after an infection? A. serum ELISA B. Western blot C. quantitative HIV RNA D. CD4 count E. p24 antigen detection

C. quantitative HIV RNA. The sensitivity of quantitative HIV RNA approaches 100%; however, the specificity is less. In a confirmatory test, you would like optimal specificity, unlike screening tests where you would like the sensitivity to be high. QCCP2,HIV

Elevation of the granulocyte percentage above 75% is termed: A. absolute lymphocytosis B. leukocytosis C. relative neutrophilic leukocystosis D. absolute neutrophili leukocytosis

C. relative neutrophilic leukocystosis

Which of the following types of polycythemia is a severely burned patient MOST likely to have? A. polycythemia vera B. polycythemia, secondary to hypoxia C. relative polycythemia associated with dehydration D. polycythemia associated with renal disease

C. relative polycythemia associated with dehydration

A patient is admitted with a history of chronic bleeding secondary to peptic ulcer. Hematology workup reveals a severe microcytic, hypochromic anemia. Iron studies were requested. Which of the following would be expected in this case? (chart on picture) A. result A B. result B C. result C D. results D

C. results C

A specimen run on an automatic cell counter has a plt count of 19 x 10^3/µL (19 x 10^9/L). The first thing the tech should do is: A. report the count after the batch run is complete B. request a new specimen C. review the stained blood smear D. notify the lab manager

C. review the stained blood smear

110. Negri bodies in Purkinje cells are associated with which of the following viruses? A. paramyxoviruses B. Eastern equine encephalitis C. rhabdovirus D. lymphocytic choriomeningitis virus E. Ebola virus

C. rhabdovirus. Rabies virus is transmitted through bites by contaminated animals. In most of the world, that means dogs and cats. In areas where the domestic causes are controlled, non -domestic animals, such as bats, skunks, and foxes are responsible for the transmission of the majority of cases. QCCP2, Family Rhabdoviridae,

65. Why is caesarean section delivery of children from mothers with prodromal or active genital herpes recommended? A. increased risk of vaginal trauma to the mother B. indication of immunodeficiency in mother and increased risk of secondary infection C. to decrease the risk of neonatal herpes D. to decrease the risk of spreading infection to health care workers E. to decrease risk of puerperal coinfection

C. to decrease the risk of neonatal herpes. Vertical transmission of HSV from genital lesions to an infant can cause neonatal HSV infection, which can manifest as skin lesions to encephalitis, retinitis, even sepsis. There is a very high transmission rate with vaginal birth. QCCP2,Human herpes viruses

The lab tests performed on a patient indicated macrocytosis, anemia, leukopenia, and thrombocytopenia. Which of the following disorders is the patient most likely to have? A. iron deficiency B. hereditary spherocytosis C. vitamin B12 deficiency D. acute hemorrhage

C. vitamin B12 deficiency

If a person's most probable Rh phenotype is R1R1, what is that in Fisher-Race nomenclature and in Rosenfield

CDe/CDe Rh: 1, 2, -3, -4, 5

viral and bacterial meningitis

CSF glucose and lactic acid are used to distinguish between:

cerebrospinal fluid

CSF is short for:

dye binding

Calcium and magnesium are most frequently measured using _____ assays.

Envelope-shaped crystals

Calcium oxalate stones

gd T-Cell Function

Can recognize cell surface proteins without MHC and Stress Proteins on MHC1B (uses a surface CD8 variant) to induce apoptosis of presenting cell. Can secrete cytokines or become cytotoxic.

Reciprocal TH Inhibition

Can reduce effectiveness of vaccine if given while animal has a parasitic infection. May also develop wrong antibody type.

Polysaccharide

Can't be presented on MHC2, may bypass by attaching a foreign protein for TH activation and B Cell switch/proliferation.

1, 2, 1

Carbohydrates usually follow the pattern of ____ carbon(s) to ____ hydrogen(s) to ____ oxygen(s).

lungs

Carbon dioxide is eliminated by the:

decarboxylation of organic compounds

Carbon dioxide is primary formed by:

B-Cell

Carry a single type of antigen receptor as a surface protein, requires intact antigen (3-D) for activation, proliferate into Memory and/or Plasma (effector) Cells once activated.

C5b67

Causes Leukocyte chemotaxis to site.

Secondary Response

Causes a strong, rapid spike of IgG/A/E, followed by a normal IgM spike, faster with a higher antibody titer and binding affinity.

Initial Response

Causes equal IgM and IgG/A/E spikes.

Killer Cell Binding

Causes rearrangement of organelles to proximal side of cell to release granules -> extrinsic apoptosis.

Neutrophil Adhesion

Cell uses L-Selection, LFA-1, and Leukosalin (?). Binds to P-Selectin, ICAM-1, and Elastase (?).

muscle

Cells in this tissue lack the enzyme glucose-6-phosphate phosphotase, trapping glucose in the cells:

Cellular Cast

Cellular casts may be red cell, white cell, or epithelial cell in composition. Red cell casts indicate renal hemorrhage or inflammation, white cell casts indicate renal inflammation, and epithelial cell casts indicate acute tubular degeneration

low

Cereuloplasmin in Wilson's disease is typically:

liver

Cereuloplasmin is manufactured in the:

barometric pressure changes

Cheap or old analyzers may need to be recalibrated if:

light source

Chemiluminescence is 2000x more sensitive than spectrophotometry because it doesn't use a(n):

AgCl crystals in a polymer membrane

Chloride ISEs contain:

detect abnormal anions

Chloride itself is relatively inert, but its presence in the blood is measured to:

concentration is very low

Chloridometers are most useful for measuring chloride when:

halogen

Chloridometers will detect any chemical that is a(n):

Mature Lymphocytes

Circulate from blood -> lymph, do not express receptors for High Endothelial addressins.

Killer Cell CD8

Circulates to any site of infection, moves along cell regions, can kill many in a row, but selective for a specific MHC1 antigen.

transferrin

Circulating iron is bound to:

Eosinophil

Circulatory life is 30 min, somewhat rare, live ~ 2wks, very high granular content.

What is the first procedure to be done in the laboratory investigation of a transfusion reaction

Clerical check on all the paper work.

Cells from which of the following tubes may give a false positive DAT due to C3 sensitizing the cells in vitro?

Clotted—red stoppered

raising pH

Clotting causes issues with blood gas analysis by:

proteins

Colloid osmotic pressure osmometry is used specifically to measure the osmotic pressure exerted by:

Neutrophil

Common granulocyte phagocytotic cell, polymorphonuclear (lobed and continuous), condensed chromatin (little transcription, end-stage cell), excess membrane, short life.

1

Competency testing must be performed at least ____ time(s) per year.

C3a

Complement protein acting as an anaphylatoxin (Mast Cell degranulator, causes vasodilation and increased permeability), and microbial killer.

C2a

Complement protein causing increased vascular permeability.

C5a

Complement protein, causes Neutrophil chemotaxis to site, increases their activity and lysosomal enzyme secretion, acts as an anaphylatoxin, increases vascular permeability and smooth muscle contraction.

Complement Fixation

Complement proteins expose a highly reactive carbonyl group when cleaved into active form, must immediately bind (covalently) to nearby surface, or get deactivated by water.

Substrate Modulation

Complement proteins involved in a cascade must covalently bind to target before cleavage occurs by preceding enzyme complex, regulatory mechanism.

bone tumors, hypervitaminosis D, calcium overdose

Conditions in which calcium is high and phosphates are high are:

hypoparathyroid disease, renal disease

Conditions in which calcium is low and phosphates are high are:

uric acid

Conditions such as gout and kidney stones are associated with high:

When a clinically significant alloantibody is identified in the patient's serum, what is done prior to transfusing the unit

Confirm that the donor cells are negative for the antigen specific for the patient's antibody by testing the donor cells with commercial antisera. Perform a crossmatch using AHG

Competitive ELISA

Conjugate added competes with bound serum sample, more quantitatively sensitive, less color = higher serum levels. Can use to confirm pregnancy via ProgE levels.

high

Conn's disease or Cushing's syndrome will cause sodium levels to be:

Hybridoma

Contains two nuclei, one from B-Cell producing desired antibody, the other from a Myeloma for indefinite expansion.

biological

Control materials must be ______ in origin.

accuracy, validity

Controls are used to asses these characteristics of an assay:

atomic absorption spectroscopy

Copper can be directly measured using:

cereuloplasmin

Copper is stored and transported by binding to the protein:

Neonatal Bilirubin

Cord Blood Less than 4.5 mg/dL 3-Day Less than 7.0 mg/dL 5-Day Less than 12.0 mg/dL 7-Day Less than 7.0 mg/dL

renally

Creatinine is excreted:

0.5-1.1

Creatinine normal range for females is ___-___ mg/dL.

0.6-1.2

Creatinine normal range for males is ___-___ mg/dL.

intra-renal azotemia

Creatinine rises faster than BUN in:

<40 mg/dL, >450 mg/dL

Critical values for venous serum/plasma glucose on children and adults is:

<30 mg/dL, >300 mg/dL

Critical values for venous serum/plasma glucose on newborns is:

Struvite stone

Crystal Proteus spp

hyperglycemia

Cushing's disease and Cushing's syndrome cause:

square

Cuvettes should be _____ in shape or they will reflect light.

Hexagonal-shaped crystals

Cystine stones

HMGB1

Cytokine released by damaged tissues (new route for pathogen entry).

Interferons

Cytokine released in response to viral infection (more with hit-and-run virus types), promotes apoptosis of infected cells and exerts antiviral effects and resistance on adjacent uninfected cells, but also impedes normal cell functions. Also acts as chemotactic for NKs and Lymphocytes.

Chemokine

Cytokine that acts to attract immune cells to a site of infection.

Interferon

Cytokine that interferes with virus reproduction.

Acute Phase Protein

Cytokines and factors produced by the liver when activated by innate immunity. Includes Mannose-binding factors, clotting factors, Fe/Cu chelators, protease inhibitors, and complement proteins. Indicated by downregulation of Albumin production, check Albumin/Globulin ratio.

What is the MCV if the Hct is 20%, the RBC is 1.5 x 10^6/µL (1.5 x 10^12/L) and the Hgb is 6 g/dL (60 g/L)? A. 68 pg B. 75 pg C. 115 pg D. 133 pg

D. 133 pg

Thrombocytosis would be indicated by a plt count of: A. 100 x 10^3/µL (100 x 10^9/L) B. 200 x 10^3/µL (200 x 10^9/L) C. 300 x 10^3/µL (300 x 10^9/L) D. 600 x 10^3/µL (600 x 10^9/L)

D. 600 x 10^3/µL (600 x 10^9/L)

Given the following data: -Hgb: 8 g/dL (80 g/L) -Hct: 28% -RBC: 3.6 x 10^6/µL (3.6 x 10^12/L) The MCV is: A. 28 µm^3 (28 fL) B. 35 µm^3 (35 fL) C. 40 µm^3 (40 fL) D. 77 µm^3 (77 fL)

D. 77 µm^3 (77 fL)

93. What percentage of people infected with hepatitis C virus will go on to chronic infection? A. <5% B. 25% C. 50% D. 85% E. >99%

D. 85%. The vast majority of people infected with HCV develop chronic infections. Of these, 10-15% become cirrhotic, and of the cirrhotics, 5% (if non-drinkers) will develop hepatocellular carcinoma. Alcohol consumption greatly increases the risk of HCC, especially in the cirrhotic with chronic infection. QCCP2,HCV

167. Which of the following organisms grows as a mold with smooth, "lollipop"-shaped conidia? A. Chrysosporium B. Blastomyces C. Sporothrix D. A & B E. A, B, C

D. A & B. A number of other molds, such as Paracoccidioides and Scedosporium can produce "lollipop" conidia. Sporothrix, however, produces a very characteristic "daisy-head" conidia that is not easily confused with Blastomyces. QCCP2, Blastomyces

62. Which of the following viruses can have both nuclear and cytoplasmic inclusions? A. CMV B. measles C. rabies D. A & B E. A, B, C

D. A & B. Both CMV and measles are known for having nuclear and cytoplasmic inclusions. HSV and adenovirus have nuclear inclusions only, while rabies is the only major human viral pathogen with cytoplasmic inclusions only (Negri bodies). QCCP2,T3.6, Viral histology

83. Which of the following subtypes of herpes virus is a possible cause of roseola infantum (exanthem subitum)? A. HHV6 B. HHV7 C. HHV8 D. A & B E. A, B, C

D. A & B. Both HHV6 and HHV7 cause overlapping and similar clinical syndromes. A small percentage of those infected develop the characteristic roseola infantum rash. In addition, they are both responsible for a significant number of cases of juvenile febrile seizures, presumably due to their tropism for the CNS. QCCP2,HHV6

138. Which of the following organisms is/are commonly seen coinfecting patients with babesiosis? A. Borrelia burgdorfei B. Ehrlichia chaffeensis C. microfilariae D. A & B E. A, B, C

D. A & B. Lyme disease, babesiosis, ehrlichiosis, and even some flaviviruses are transmitted by the deer tick, Ixodes scapularis, on the east coast and the western black-legged tick, Ixodes pacificus, on the west coast. QCCP2, Babesia microti

88. Oral squamous papillomas, laryngeal papillomas, condyloma acuminatum, and low-grade cervical squamous intraepithelial lesions are associated with this/these type/s of HPV: A. HPV6 B. HPV11 C. HPV16 D. A & B E. A, B, C

D. A & B. Subtypes 6 and 11 are often associated with low-grade lesions, while the more oncogenic subtypes, such as 16 and 18, are associated with the higher-grade lesions. HPV18 is most commonly associated with cervical adenocarcinoma. QCCP2,HPV, T3.11

109. Which of the following mosquitoes transmit both yellow fever and Dengue fever viruses? A. Aedes aegypti B. Aedes albopictus C. Culex pipiens D. A & B E. A, B, C

D. A & B. The Aedes mosquitoes are responsible for transmission of the viruses that cause yellow fever and Dengue fever, as well as some of the causative agents of filariasis and viral encephalitis. Culex pipiens is also a vector for many of the Arboviridae, such as Eastern and Western equine encephalitis, but it is not been shown transmit yellow or Dengue fever viruses. QCCP2,Dengue and yellow fever

143. Which aspects of Strongyloides larva help distinguish it from hookworm larva? A. short buccal groove B. prominent genital primordium C. appearance of eggs D. A & B E. A, B, C

D. A & B. The short buccal groove and prominent genital primordium help distinguish Strongyloides larva. Again recall that the eggs of the hookworms, a decorticate Ascaris egg, and the eggs of Strongyloides are, for all intents and purposes, identical. QCCP2, Strongyloides

If a person has the Se gene, the A gene and the B gene, what blood group substances will be found in his body fluids

D. A, B and H blood group substance.

The prothrombin time test requires that the patient's citrated plasma be combined with: A. plt lipids B. thromboplastin C. Ca++ and plt lipids D. Ca++ and thromboplastic

D. Ca++ and thromboplastic

60. Which of the following viruses is assayed by injection into suckling mice and observation for flaccid paralysis? A. Clostridium botulinum B. adenovirus C. parvovirus D. Coxsackie A virus E. respiratory syncytial virus

D. Coxsackie A virus. Clostridium botulinum can cause flaccid paralysis in infants, but it's not a virus, and it's not assayed in suckling mice. Adenovirus causes cytopathic effect in Hep2 cells, parvovirus needs erythroid precursors and RSV causes syncytia formation in Hep2 cells. QCCP2,T3.5

54. Which of the following organisms is the most common cause viral myocarditis? A. adenovirus B. JC virus C. Coxsackie A D. Coxsackie B E. human herpes virus 6

D. Coxsackie B. Coxsackie A and B are frequently confused for each other when it comes to the diseases that they each cause. Coxsackie A causes hand-foot-mouth disease, so named for the most common locations of the lesions. Coxsackie B, on the other hand, is responsible for the majority of cases of viral myocarditis. QCCP2,T3.2

Refer to the following illustration: Which area in the automated cell counter histogram represents the RBC distribution curve? A. A B. B C. C D. D

D. D

124. Which amoeba is most often seen as a coinfection with Enterobius vermicularis? A. Naegleria B. Acanthamoeba C. Entamoeba histolytica D. Dientamoeba E. Iodamoeba

D. D. With two nuclei and a fractured central karyosome, Dientamoeba is often seen coinfecting with Enterobius. Like Enterobius, D. fragilis causes diarrhea and pruritus ani. QCCP2, Dientamoeb

152. Infection by this organism can be a cause of B12 deficiency: A. Taenia solium B. Taenia saginata C. Schistosoma mansoni D. Diphyllobothrium latum E. Echinococcus granulosus

D. D. . Diphyllobothrium latum is noted for its ability to cause vitamin B12 deficiency by competing for binding the vitamin. It occurs primarily in Scandinavia and E. Europe. QCCP2, Diphyllobothrium

Inclusions in the cytoplasm of neutrophils as shown in the figure below are known as: A. Auer bodies B. Howell-Jolly bodies C. Heinz bodies D. Döhle bodies

D. Döhle bodies

153. This organism is responsible for hydatid cysts of the liver: A. Diphyllobothrium B. Taenia C. Hymenolepis D. Echinococcus E. Dypylidium

D. E. The definitive host is the dog. Ingestion of infected dog stool (oh, boy!) through intermediate hosts, such as sheep or cattle, causes the disease. The primary manifestation is multilocular hepatic cysts, and there are several species. QCCP2, Echinococcus spp

Which of the following is Vitamin-K dependent? A. Factor XII B. fibrinogen C. antithrombin III D. Factor VII

D. Factor VII

Morphologic variants of plasma cells do NOT include: A. flame cells B. morula cells C. grape cells D. Gaucher cells

D. Gaucher cells

The light-colored zone adjacent to the nucleus in a plasmacyte is the: A. ribosome B. chormatin C. mitochondira D. Golgi area

D. Golgi area

154. All of the following organisms can show respective dual or triple infection, except: A. Ascaris and Trichuria B. Enterobius and Dientamoeba C. Babesia, B. burgdorfei, and Ehrlichia D. Hymenolepis nana and Echinococcus E. lepromatous leprosy and Strongyloides

D. H andE. There is no known association between Hymenolepis and Echinococcus. All the other groups have been shown to coinfect, some due to common vectors (Babesia, Lyme, Ehrlichia), some due to host immune status (leprosy and Strongyloides hyperinfection). QCCP2,Take-home points

89. In a person vaccinated against hepatitis B virus several years prior, which serological marker would be expected? A. HBsAg B. HBeAg C. IgG anti-HBc D. HBsAb E. anti-HBc

D. HBsAb. All of the other choices, with perhaps the early appearance post-vaccination of HBsAg, should not be seen in a vaccinated individual. HBeAg is present during active viral replication, while anti-HBc is present only in someone who has resolved an acute infection. It is the same with anti-HBe, though this is less associated with resolution than with someone who has just had the HBeAg turn negative in testing. QCCP2,HBV

49. Latex agglutination tests on CSF are commonly used for the diagnosis of all of the following causes of meningitis, except: A. Haemophilus influenzae, type B B. Neisseria meningitidis C. group B Streptococcus D. Listeria monocytogenes E. Streptococcus pneumoniae

D. L. Listeria is a very difficult organism to identify in CSF. Gram stain sensitivity is less than 50% and there is no commercially available latex agglutination test at this time. Culture is the main source of positive identification. QCCP2,Laboratory evaluation

Which of the RBC indices is a mausre of the amount of hemoglobin in individual RBCs? A. MCHC B. MCV C. Hct D. MCH

D. MCH

A blood sample from a patient with a high-titer cold agglutinin, analyzed at room temp, with an electronic particle counter would cause an error in the: A. Hgb and MCV B. MCHC and WBC C. WBC and RBC D. MCV and MCHC

D. MCV and MCHC

Which of the following anomalies is an autosomal dominant disorder characterized by irregularly-sized inclusions in polymorphonuclear neutrophils, abnormal giant plt and often thrombocytopenia? A. Pelger-Huët B. Chédiak-Higashi C. Alder-Reilly D. May-Hegglin

D. May-Hegglin

Of the following, the disease most closely associated with mucopolysaccharidosis is: A. Pelger-Huët anomaly B. Chédiak-Higashi syndrome C. Gaucher disease D. Alder-Reilly anomaly

D. May-Hegglin anomaly

Which of the following is most closely associated with chronic myelogenous leukemia? A. ringed sideroblasts B. DIC C. micromegakaryocytes D. Philadelphia chromosome

D. Philadelphia chromosome

Which of the following stains is used to demonstrate iron, ferrite and hemosiderin? A. peroxidase B. Sudan black B C. periodic acid-Shiff (PAS) D. Prussian blue

D. Prussian blue

The 2 factors that differentiate liver diease from Vitamin K deficiency are: A. II and VII B. IX and VII C. VII and IX D. V and VII

D. V and VII

68. Ramsay-Hunt syndrome is an infection of the facial nerve as caused by which of the following viruses? A. SV40 B. CMV C. HSV D. VZV E. HIV

D. VZV. VZV reactivation in a dermatomal pattern is the cause of the painful shingles rash. When reactivation occurs in the facial nerve the Ramsay-Hunt syndrome can result in facial paralysis, vertigo, tinnitus, and otalgia. QCCP2,VZV

102. What name is given to the characteristic measles-infected cell in the lungs or appendix? A. Anitschow cell B. Touton giant cell C. floret cell D. Warthin-Finkeldey E. Hallmark cell

D. Warthin-Finkeldey cell. The Anitschow cell is seen in rheumatic fever. The Touton giant cell with its wreath-like arrangement of nuclei can be seen with several lesions, including juvenile xanthogranuloma and atypical fibroxanthoma. The floret cell is seen with pleomorphic liposarcoma and the hallmark cell is usually associated with anaplastic large cell lymphoma. QCCP2,Measles

Hemorrhage is polycythemia vera is the results of: A. increased plasma viscosity B. persistent thrombocytosis C. splenic sequestration of plts D. abnormal plt fuction

D. abnormal plt fuction

Elevation of the total granulocyte count above 7.7 x 10^3/µL (7.7 x 10^9/L) is termed: A. relative lymphocytosis B. leukocytosis C. relative neurtophilic leukocytosis D. absolute neutrophilic lekocytosis

D. absolute neutrophilic lekocytosis

Chronic lymphocytic leukemia is defined as a(n): A. malignancy of the thymus B. accumulation of pro lymphocytes C. accumulation of hairy cells in the spleen D. accumulation of monoclonal B cells with a block in cell maturation

D. accumulation of monoclonal B cells with a block in cell maturation

50-90% myeloblasts in peripheral blood is typical of which of the following? A. chronic meylocytic leukemia B. myelofibrosis with myeloid metaplasia C. erythroleukemia D. acute myelocytic leukemia

D. acute myelocytic leukemia

Auer rods are MOST likely present in which of the following? A. chronic meyloblastic leukemia B. myelofibrosis with myeloid metaplasia C. erythroleukemia D. acute myelocytic leukemia

D. acute myelocytic leukemia

A block in the differentiation of maturation of, and an accretion of immature hmatopoietic progenitors is a hallmark of: A. chronic lymphocytic leukemia B. myeloproliferative diseases C. polycythemia vera D. acute myelogenous leukemia

D. acute myelogenous leukemia

58. Which of the following viruses grows best in Hep2 cells? A. enterovirus B. Coxsackie A C. Coxsackie B D. adenovirus E. cytomegalovirus

D. adenovirus. Hep2 cells are derived from the upper respiratory tract, a region for which adenovirus has a high tropism (same with RSV). QCCP2,T3.4

A mean cellular hemoglobin concentration (MCHC) over 36/dL (36 g/L) is frequently found in: A. hereditary spherocytosis B. lipemia (abnormally high plasma lipid) C. active cold agglutinin D. all of the above

D. all of the above

131. What test best helps to differentiate pregnant women at a lower risk for intrauterine Toxoplasma infection? A. anti-Toxoplasma IgM B. PCR of CSF C. Giemsa staining D. anti-Toxoplasma IgG E. amniotic fluid serology

D. anti-Toxoplasma IgG. Less than 1/5 of pregnant women have been previously infected by Toxoplasma, a fact which is confirmed with positive IgG anti-toxo serology. These previously infected women are at an extremely low risk of transplacental transmission of Toxoplasma compared to those who have not been previously exposed. QCCP2, Toxoplasma

In comparison to malignant lymphoma cells, reactive lympocytes: A. have a denser nuclear chromatin B. are know to be T cells C. have more cytoplasm and more mitochondria D. are morphologically more variable throughout the smear

D. are morphologically more variable throughout the smear

A patient has a tumor that concentrates erythropoietin. He is most likely to have which of the following types of plycythemia? A. polycythemia vera B. polycythemia, secondary to hypoxia C. benign familial polycythemia D. polycythemia associated with renal disease

D. associated with renal disease

48. Which of the following etiologies of meningitis is consistent with CSF findings of glucose <45 mg/dL, protein >500 mg/dL, and a white blood cell count >1000 WBC/mL? A. viral B. amebic C. aseptic D. bacterial E. chemical

D. bacterial. The cause of meningitis can be narrowed down with the CSF chemistry. Low glucose and high protein/WBC count is consistent with a bacterial meningitis. The WBC differential is typically left-shifted with a predominance of neutrophils. QCCP2,Laboratory evaluation

42. What is the animal reservoir of West Nile virus? A. mice B. rats C. domestic cats D. birds E. rabbits

D. birds. Remember to stay away from dead birds! The host organism is the bird, which passes on the virus through a mosquito vector to humans. Arboviruses in general (to include Eastern equine, St. Louis, California, etc.) are best diagnosed through serological means. QCCP2,Meningitis, specific agents

A patient has been treated for polycythemia vera for several years. His blood smears now shows: -Oval macrocytes -Howell-Jolly bodies -Hypersegmented neutrophils -Large, agranular plts The most probable cause of this picture is: A. iron deficiency B. alcoholism C. dietary B12 deficiency D. chemotherapy

D. chemotherapy

168. In what patient population has an unusual inhalational form of Sporothrix been identified? A. the very young B. elderly C. immunocompromised D. chronic alcoholics E. smokers

D. chronic alcoholics. Although Sporothrix is unique among the dimorphic because its primary means of infection is percutaneous, a less common means of inhalation (like the rest of the dimorphic fungi) has been documented in chronic alcoholics. QCCP2, Sporothrix

Which of the following is MOST closely associated with iron deficient anemia? A. iron overload in tissue B. target cells C. basophilic stippling D. chronic blood loss

D. chronic blood loss

158. All of the following entities are in the differential diagnosis of a fuzzy colony on a plate, except: A. hyaline septate mold B. dematiaceous mold C. dimorphic fungus D. coccoid yeast E. aseptate mold

D. coccoid yeast. The presence of a fuzzy colony implies the presence of a mold. Based on that, one would not expect a coccoid yeast to be in the differential diagnosis. It is important to also note that most dimorphic fungi will grossly appear as a mold by yeast conversion. QCCP2,Identification of a fungal isolate

Thalassemias are characterized by: A. structural abnormalities in the hemoglobin molecule B. absence of iron in hemoglobin C. decreased rate of heme synthesis D. decreased rate of glob in synthesis

D. decreased rate of glob in synthesis

Heinz bodies are: A. readily identified with polychrome stains B. rarely found in G6PD deficient erythrocytes C. closely associated with spherocytes D. denatured hemoglobin inclusions that are readily removed by the spleen

D. denatured hemoglobin inclusions that are readily removed by the spleen

The following results were obtained on an electronic particle counter: -WBC: ++++ -RBC: 2.01 x 10^6/µL (2.01 x 10^12/L) -Hgb: 7.7 g/dL (77 g/L) -Hct: 28.2% -MCV: 141 µm^3 (141 fL) -MCH: 38.5 pg -MCHC: 23.3% What step should be taken before recycling the sample? A. clean the apertures B. warm the specimen C. replace the lysing agent D. dilute the specimen

D. dilute the specimen

In infectious mono, lymphocytes tend to be: A. small with little cytoplasm B. normal C. decreased in number D. enlarged and indented by surrounding structures

D. enlarged and indented by surrounding structures

Many microspherocytes, schistocytes and budding off of spherocytes can be seen on peripheral blood smears of patients with: A. hereditary spherocytosis B. DIC C. acquired autoimmune hemolytic anemia D. extensive burns

D. extensive burns

A blue top tube is drawn for coag studies, the sample is a short draw results may be: A. falsely shortened B. correct C. unable to be obtained D. falsely prolonged

D. falsely prolonged

In synovial fluid, the most characteristic finding in traumatic arthritis is: A. monosodium urate crystals B. cartilage debris C. calcium pyrophosphate dihydrate crystals D. hemosiderin-laden macrophages

D. hemosiderin-laden macrophages

In order for hemoglobin to combine reversibly with oxygen, the iron must be: A. complexed with haptoglobin B. freely circulating in the cytoplasm C. attached to transferrin D. in the ferrous state

D. in the ferrous state

When using the turbidity (solubility) method for detecting the presence of hemoglobin S, and incorrect interpretation may be when there is: A. concentration of less than 7 g/dL (70 g/L) Hgb B. glucose concentration greater than 150 mg/dL (8.3 mmol/L) C. blood specimen graeater than 2 hours old D. increased Hgb

D. increased Hgb

Which of the following is NOT a characteristic usually associated with hairy cell leukemia? A. pancytopenia B. mononuclear cells with ruffled edges C. splenomegaly D. increased resistance to infection

D. increased resistance to infection

A 14 y/o boy is seen in the ER complaining of a sore throat, swollen glands and fatigue. The CBC results are: -WBC: 16.0x10^3/µL(16.0x10^9/L) -RBC: 4.37 x 10^6/µL(4.37x10^12/L) -Hgb: 12.8 g/dL (128 g/L) -Hct: 38.4% -Plt: 180x10^3/µL(180x10^9/L) Differential: -absolute neurtophils: 3.9x10^9/L -absolute lymphs: 6.0x10^9/L -absolute monos: 0.5x10^9/L -absolute atypical lymphs: 3.2x10^9/L What is the most likely dx? A. acute lymphoblastic leukemia B. chroni lymphocytic leukemia C. viral hepatitis D. infectious mononucleosis

D. infectious mononucleosis

Heparin acts by: A. precipitating fibrinogen B. binding Ca C. activating plasmin D. inhibiting thrombin

D. inhibiting thrombin

The following results were obtained from a post-surgical patient receiving total parenteral nutrition: The most consistent explanation for the above data is: A. acute surgical bleeder B. specimen on day 19 from wrong patient C. improperly mixed specimen on day 19 D. lipid interference on days 17 and 18

D. lipid interference on days 17 and 18

Therminal deoxynucleotidyl transferase (TdT) is a marker found on: A. hairy cell B. myeloblasts C. monoblasts D. lymphoblasts

D. lymphoblasts

Multipotent stem cells are capable of producing: A. daughter cells of only one cell line B. only T-lymphocytes and B-lymphocytes C. erythropoietin, thrombopoietin, and leukopoietin D. lymphoid and myeloid stem cells

D. lymphoid and myeloid stem cells

Which of the following is most closely associated with chronic myelomonocytic leukemia? A. Philadelphia chromosome B. DIC C. micromegakaryocytes D. lysozymuria

D. lysozymuria

vWF antigen can be found in which of the following? A. myeloblast B. momoblast C. lymphoblast D. megakaryoblast

D. megakaryoblast

Which of the following conditions is NOT associated with a high incidence of leukemia? A. paroxysmal nocturnal hemoglobinuria B. Fanconi anemia C. aplastic anemia D. megaloblastic anemia

D. megaloblastic anemia

In normal adult bone marrow, the most common granulocyte is the: A. basophil B. myeloblast C. eosinophil D. metamyelocyte

D. metamyelocyte

Cell description: -Size: 12 to 16 µm -nucleus: oval, notched, folded over to horseshoe shape -Chromatin: fine lacy, stains light purple-pink -Nucleoli: none present -Cytoplasm: abundant, slate gray, with many fine lilac-colored granules The cell is a: A. promyelocyte B. lymphocyte C. neutrophil D. monocute

D. monocute

In which of the following disease states are teardrop cells and abnormal PLTs most characteristically seen? A. hemolytic anemia B. multiple myeloma C. G-6-PD deficiency D. myeloid metaplasia

D. myeloid metaplasia

85. Members of which of the following families of viruses are responsible for progressive multifocal leukoencephalopathy and viral hemorrhagic cystitis? A. herpes viruses B. picornaviruses C. reoviruses D. papovaviruses E. bunyaviruses

D. papovavirus. The papovaviridae are a family of DNA viruses including the papilloma viruses, polyoma viruses, and vacuolating viruses (SV40) JC, a member of the polyoma virus family, is responsible for progressive multifocal leukoencephalopathy, while another polyoma virus, BK, can cause hemorrhagic cystitis. The BK-induced cystitis is notable for the presence of virus-infected "decoy" cells, so named because they can be easily confused for exfoliated high-grade urothelial carcinoma. QCCP2,Papovaviruses

77. All of the following are considered Hoagland criteria for the diagnosis of infectious mononucleosis, except: A. leukocytosis >50% lymphocytes, >10% atypical lymphocytes B. fever C. pharyngitis D. positive culture E. positive serological testing

D. positive culture. Adenopathy is the missing criterion. While positive culture can occur with EBV, it is neither routinely performed, nor required for the diagnosis. These criteria are very restrictive and may miss many cases. Partial fulfillment of criteria can be seen in many other diseases, such as strep pharyngitis, which can have both fever and adenopathy, while the leukocytosis can be seen with CMV or Toxoplasma infections. QCCP2,EBV

The automated plt count on an EDTA specimen is 58 x 10^3/µL (58 x 10^9/L). The plt estimate on the blood smear appears normal, but it was noted that the plts were surrounding the neutrophils. The next step should be to: A. reports the automated plt count since it is more accurate than a plt estimate B. warm the EDTA tube and repeat the automated plt count C. rerun the original specimen since the plt count and blood smear estimate do not match D. recollect a specimen for a plt count using a different anticoagulant

D. recollect a specimen for a plt count using a different anticoagulant

An automated plt count indicates plt clumping, which is confirmed by examining the smear. The tech should: A. repeat the count on the same sample B. report the automated cell count C. preform a manual diff D. recollect in sodium citrate

D. recollect in sodium citrate

In the normal adult, the spleen acts as a site for: A. storage of RBCs B. production of RBCs C. synthesis of erythropoietin D. removal of imperfect and aging cells

D. removal of imperfect and aging cells

On setting up the electronic particle counter in the morning, one of the controls is slightly below the range for the MCV. Which of the following is indicated? A. call for service B. adjust the MCV up slightly C. shut down the instrument D. repeat the control

D. repeat the control

A patient has a high cold agglutinin titer. Automated cell counter results reveal an elevated MCV, MCH and MCHC. Individual erythrocytes appear normal on a stained smear, but agglutinates are noted. The appropriate course of action would be to: A. perform the RBC, Hgb, and Hct determinations using manual methods B. perform the RBC determination by a manual method; use the automated results for the Hgb and Hct C. report the determinations using a microsample of diluted blood D. repeat the determination using a pre warmed microsample of diluted blood

D. repeat the determination using a pre warmed microsample of diluted blood

The ISI in the INR represents the reagents: A. activator B. specificity C. phospholipids D. sensitivity

D. sensitivity

Patients with A(-) type G6PD deficiency are LEAST likely to have hemolytic episodes in which of the following situations? A. following the administration of oxidizing drugs B. following the ingestion of fava beans C. during infections D. spontaneously

D. spontaneously

A characteristic morphologic feature in hemoglobin C disease is: A. macrocytosis B. spherocytosis C. rouleaux formation D. target cells

D. target cells

When evaluating a smear for a retic count, the tech observes the the RBCs are overlapping throughout the entire slide. The most likely explanation is: A. grease on the slide prevented even spreading B. improper proportions of blood and stain were used C. the slide was dried too quickly D. the drop used for the slide prep was too large

D. the drop used for the slide prep was too large

Which of the following statements about this field is TRUE? A. retics are demonstrable B. toxic granulation is present C. the cell in the center is a basophilic normoblast D. the large cell on the left is a monocyte

D. the large cell on the left is a monocyte

121. What's the fastest way to identify Iodamoeba butschlii? A. the presence of ingested red blood cells in the trophozoite form B. culture on a lawn of inactivated E coli C. its small (5-10 micron) size D. the prominent vacuole in the cyst form E. the presence of up to 8 nuclei in the cyst form

D. the prominent vacuole in the cyst form. Iodamoeba beutschlii, at risk of sounding indelicate, has a "butt" or a large clear vacuole in the cyst form. Naegleria is grown on inactivated E. coli, Entamoeba hartmanii is small, and Entamoeba coli has up to 8 nuclei in the cyst form. QCCP2,Protozoa

A prolonged thrombin time and a normal reptilase-R time are characteristic of: A. dysfibrinogenemia B. increased D-dimer C. fibrin monomer-split product complexes D. therapeutic heparinization

D. therapeutic heparinization

On Monday a patient's Hgb determination was 11.3 g/dL (113 g/L), and on Tuesday it measured 11.8 g/dL (118 g/L). The standard deviation of the method used is ±0.2 g/dL (2 g/L). Which of the following can be concluded about the Hgb values given? A. one value probably resulted from a lab error B. there is poor precision; daily QC charts should be checked C. the second value is out of range and should be repeated D. there is no significant change in the patient's Hgb concentration

D. there is no significant change in the patient's Hgb concentration

107. Why are yellow fever and rubella viruses different from the other members of the togavirus family? A. they have DNA genomes B. they have hybrid RNA-DNA genomes C. they require coinfection with another virus D. they do not cause arthropod-borne encephalitis E. they both primarily infect the liver

D. they do not cause arthropod-borne encephalitis. Yellow fever and rubella virus are both RNA virus members of the Togaviridae. Unlike all other togaviruses, such as alphaviruses and other flaviviruses, they are not predominantly arthropod-borne causes of encephalitis. Yellow fever favors the heart, GI tract, liver, and kidneys, while rubella causes lymphadenopathy and rash. QCCP2, Family Togaviridae

Cells invalid in hemostasis are: A. erythrocytes B. granulocytes C. lymphocytes D. thrombocytes

D. thrombocytes

Hageman Factor (XII) is involved in each of the following EXCEPT: A. activation of C1 to C1 esterase B. activation of plasminogen C. activation of Factor XI D. transformation of fibrinogen to fibrin

D. transformation of fibrinogen to fibrin

95. How is a sustained virological response or a positive HCV treatment outcome defined? A. complete absence of detectable HCV RNA for perpetuity B. disappearance of anti-HCV antibody C. no evidence of chronic HCV on liver biopsy D. undetectable HCV RNA for 24 weeks E. greater than 10-fold reduction in HCV RNA

D. undetectable HCV RNA for 24 weeks. A sustained virologic response (SVR) is used to define patients who have responded positively to treatment (IFN/ribavirin). The single most important prognostic predictor of response to IFNs is the genotype of HCV. Types 2 and 3 respond well while type 1, the more common type seen in the U.S., tends to respond poorly. QCCP2,HCV

the sum of all partial pressures

Dalton's Law states that the total pressure of a gas mix is equal to:

hemochromatosis

Darkening of the skin may occur in both iron and copper overload and is a major symptom of:

Helper T-Cell Types

Defined by secreted Cytokine profile, may overlap. No difference in surface molecules. Initially activated by Dendritic Cells, later can accept Macrophages, NK, Neutrophils, etc.

covalent bonds

Delta bilirubin results when unconjugated bilirubin forms _______ with albumin.

hepatitis, choleostasis

Delta-bilirubin is only present in the blood of patients who have one of these conditions:

TH Regulatory Stimulation

Dendritic Cell releases TFGB in the absence of danger signals.

TH17 Stimulation

Dendritic Cell releases high levels of TFGB and IL6,23.

TH1 Stimulation

Dendritic Cells respond to pathogens with IL12 to induce NK secretion of IFNg.

Cytokine Effects

Depend on levels, may be local at low (phagocytes, endothelia, complement system), systemic at moderate (malaise etc. and liver APP production), and lethal at high (Septic Shock), which can occur with G- bacteria.

-decrease in size -condensation of nuclear chromatin -loss of nucleus -hemoglobinization

Describe the erythroid maturation sequence

-multilobulation of nucleus -increased cytoplasm -development of platelet-specific granules -breaking off platelet fragments

Describe the megakaryocytic maturation sequence

myeloblast->promyelocyte->myelocyte->metamyelocyte->band cell->mature PMN -development of granules -decrease in size -loss of nucleolus -condensation and segmentation of nucleus

Describe the myeloid maturation sequence

shift

Describe this pattern:

trend

Describe this pattern:

low

Diarrhea and/or vomiting will cause sodium levels to be:

calcium-resistant ricketts

Dietary deficiency of magnesium causes:

nucleoproteins

Diets rich in ________, such as organ meats, may result in hyperuricemia.

If a patient is receiving saline by IV, where should the phlebotomist stick the patient

Draw the blood below the IV line

spreading, reagent, support

Dry slides have these three layers, sandwiched between slide mounts:

Deceptor

Dud receptor, binds ligand without causing its intended response (functional dilution), reduces ligand's effect on target.

Integrins

Due to damage or PAMP, upregulated when danger signals contact basal endothelium, causes Neutrophils to bind tightly and move into tissue.

infection

Due to hyperglycemia, diabetes patients are at a higher risk of:

buffers

Dye binding reagents for calcium and magnesium contain _____ to adjust the pH to around 6.0.

127. What characteristic feature of T. cruzi trypomastigotes helps distinguish it from T. brucei in peripheral blood smears? A. undulating flagellum B. central kinetoplast C. culture on Novy-MacNeal-Nicolle medium D. presence in lymph nodes E. "C" shape

E. "C" shape. The other features mentioned are shared to some extent by both species of trypanosomes. The characteristic "C" shape (remember - Cruzi, Chagas, "C"-shape) helps to identify Trypanosoma cruzi compared to the more randomly curved trypomastigotes of T. brucei. QCCP2,Trypanosoma spp

98. Which of the following viruses is hemagglutinin-positive? A. influenza A B. influenza B C. parainfluenza D. A & B E. A, B, C

E. A, B, C. A viral surface protein, hemagglutinin, binds to sialic acid receptors on the surface of respiratory epithelial cells and in turn are expressed on the surface of infected cells. The hemagglutinin has the ability to agglutinate red blood cells, which is the basis of the diagnostic hemadsorption test. QCCP2,Orthomyxoviruses

146. Which feature(s) of microfilariae are most helpful in categorizing individual species? A. sheath/unsheathed B. pattern of nuclei in tail C. presence in lymphatics D. A & B E. A, B, C

E. A, B, C. All are potential means of categorizing the microfilariae - the sheathed worms are Wuchereria, Brugia, and Loa loa. The unsheathed are Mansonella and Onchocerca. Both Onchocerca and Wuchereria lack terminal nuclei, while Loa loa & Mansonella have nuclei to the tip and Brugia has two isolated nuclei at the tip Finally, Wuchereria and Brugia can be found in the lymphatics, while the rest are not. In addition, the worms can be categorized according to their periodicity in the blood. QCCP2,T3.19, Filariae

116. Which of the following organisms is detectable with modified acid-fast stains? A. Cryptosporidium B. Cyclospora C. Isospora D. A & B E. A, B, C

E. A, B, C. All of the above parasites are potentially positive for acid-fast staining, improving the chances of detection notably. The related organism microsporidium is not acid-fast and may require more invasive means for diagnosis. QCCP2,Parasites, specimens

165. Which if the following organisms is in the differential diagnosis of Coccidioides spherules? A. Rhinosporidium seeberi B. Histoplasma capsulatum C. Prototheca wickerhamii D. A & B E. A, B, C

E. A, B, C. All resemble the 50-200 micron spherules that contain the small 2-5 micron endospores. Rhinosporidia is larger and found in the nasal sinus, Prototheca in the olecranon bursa. The wall of the spherule of Coccidioides is thick, but when the spherules are released. they can resemble those of Histoplasma. QCCP2,Dimorphic fungi and molds

159. Which of the following characteristics assist with the speciation of molds? A. rate of growth B. type of hyphae C. pigmentation D. A & B E. A, B, C

E. A, B, C. Each of these features - and also type of sporulation - will help to categorize molds into discrete families. QCCP2,Identification of a fungal isolate

84. Which of the following clinical presentations is associated with HHV8? A. Kaposi sarcoma B. primary effusion lymphoma C. multicentric Castleman disease D. A & B E. A, B, C

E. A, B, C. Especially in the immunocompromised patient, HHV8, or KSHV, is responsible for a number of disease processes. The etiologic agent in these cases is identified by FISH, PCR, IHC, or serological means. QCCP2,HHV8

61. Which of the following viruses causes syncytia formation in culture? A. RSV B. measles C. HSV D. A & B E. A, B, C

E. A, B, C. If you think about the histopathology of these viruses, it helps. RSV of course causes syncytia formation - it's in the name! Measles infection can cause the formation of Warthin-Finkeldey giant cells, and then there's the 3 Ms describing the nuclei of cells infected with HSV - multinucleation, margination, and molding. QCCP2,T3.6, viral histology

78. Which of the following antibodies can be produced in response to EBV infection? A. anti-i B. ANA C. Paul-Bunnell heterophile antibodies D. A & B E. A, B, C

E. A, B, C. In addition, rheumatoid factor levels may be increased. For this reason and because the culture of EBV is difficult, routine diagnosis is usually made serologically. QCCP2,EBV

161. Which of the following fungi are inhibited by cyclohexamide? A. Zygomyces B. Aspergillus C. Cryptococcus D. A & B E. A, B, C

E. A, B, C. Most fungal pathogens, especially dimorphic fungi or dermatophytes will grow, albeit slowly, in cyclohexamide. Zygomyces, Aspergillus, and Cryptococcus won't grow. QCCP2,Identification of a fungal isolate

112. Which of the following is/are characteristic of adult T cell lymphoma? A. peripheral CD4+/CD25+ flower cells B. hypercalcemia C. high serum IL-2 receptor D. A & B E. A, B, C

E. A, B, C. Patients can also have skin rash and extreme thirst. Interestingly, the neoplastic cells express CD25, which is the IL-2 receptor, as well as high levels of free IL-2 receptor in the serum. The usual incubation time for ATCL is 20-30 years. QCCP2,HTLV-I

64. Which of the following herpes virus family members lie latent in the dorsal root ganglia? A. HSV-1 B. HSV-2 C. VZV D. A & B E. A, B, C

E. A, B, C. The HSV family of viruses is made up of the prototypical enveloped DNA viruses. Three of the members lie dormant within dorsal root ganglia until they are reactivated. QCCP2,Human herpes viruses, T3.8

130. Which of the following organisms is in the differential diagnosis of the bradyzoite of Toxoplasma gondii? A. Leishmania B. Histoplasma C. Trypanosoma D. A & B E. A, B, C

E. A, B, C. The bradyzoite forms of Toxoplasma are small intracellular organisms seen within the cytoplasm of histiocytes, while the tachyzoites are curved extracellular organisms. A histiocyte filled with Toxoplasma bradyzoites resembles Leishmania, Histoplasma, Trypanosoma, and Coccidioides, and must be distinguished. QCCP2, Toxoplasma

82. LMP1 and EBER staining show high concordance in which of the following lesions? A. Hodgkin lymphoma B. post-transplant lymphoproliferative disorder C. infectious mononucleosis D. A & B E. A, B, C

E. A, B, C. The positive correlation between the two stains is highest when observed in the cells that are of interest. LMP1 has a modest false positive rate which can be overcome by narrowing the observation to the cells of interest. QCCP2,EBV

67. All of the following features are used to diagnose congenital varicella, except: A. maternal VZV during pregnancy B. skin lesions in newborn in a dermatomal distribution C. serological evidence of newborn infection with elevated specific IgG persisting beyond 7 months D. serological evidence of newborn infection with elevated specific IgM E. all of the above are potentially utilized in the diagnosis of congenital varicella

E. All of the above are potentially utilized in the diagnosis of congenital varicella. The incidence of perinatal varicella is highest when the mother contracts VZV within a few days of delivery, with the incidence declining inversely with gestational age. Because of the extremely high infectivity of VZV contracted perinatally (50-60%), VZ immunoglobulin is recommended for prophylaxis, perhaps along with acyclovir. QCCP2,VZV

55. All of the following viruses are most commonly detected by culture, except: A. HSV-1 B. HSV-2 C. adenovirus D. HPV E. EBV

E. EBV. EBV, along with rubella and the arboviridae, are not routinely cultured, but rather diagnosed based on the positive serology. Similarly, rota- and rhabdoviruses are not cultured; their diagnosis depends on virus-specific antigen detection. QCCP2,Virology Laboratory Methods

179. Which of the following fungal organisms is a major concern as an opportunistic infection in burn victims? A. Gliosporium B. Paecilomyces C. Scopulariopsis D. Scedosporium E. Fusarium

E. F. In addition to the myriad of infections including a disseminated infection in immunocompromised hosts, Fusarium can cause infections in burn victims, pulmonary infections, skin infections, and fungemia. QCCP2, Fusarium

45. Patients with complement deficiencies are at an increased risk for meningitis caused by this infectious agent: A. Streptococcus pneumoniae B. Haemophilus influenzae C. E. coli D. Listeria monocytogenes E. Neisseria meningitidis

E. N. N. meningitidis is associated with outbreaks of meningitis in children and adults in close living conditions, such as schools, dorms, barracks, or nursing homes (though less prevalent in the elderly). Systemic meningococcemia is associated with a petechial rash and hemorrhagic adrenal infarction (Waterhouse-Friedrichsen syndrome), both of which have a poor prognosis. QCCP2,Meningitis, specific agents

47. Which of the following amoebae is most commonly the cause of primary amebic encephalitis? A. Entamoeba histolytica B. Acanthamoeba C. Entamoeba coli D. Endolimax nana E. Naegleria fowleri

E. N. Naegleria infection is associated with fresh water exposure. The organism migrates through the nasopharynx and invades the brain through the cribriform sinus. Often there is a history of trauma in fresh water (water skiing accident, diving accident). The infection is especially virulent, often causing death within days. Acanthamoeba can also cause encephalitis, but with a granulomatous appearance. Acanthamoeba is also responsible for the majority of cases of amebic keratitis. QCCP2,Meningitis, specific agents

163. Which of the dimorphic yeasts is often confused histologically with the mold form of another dimorphic fungus? A. Histoplasma B. Blastomyces C. Coccidioides D. Sporothrix E. Paracoccidioides

E. P. The yeast form of Paracoccidioides contains the characteristic "Mariner's wheel" configuration of irregular circumferential cytoplasmic blebbing. Similarly, the mold form of Histoplasma has circumferential cytoplasmic buds which are more frequent and regular than those of the Paracoccidioides yeast. QCCP2,F3.14, Dimorphic fungi

118. Which parasite can be specifically detected serologically after a recent infection? A. Paragonimus westermanii B. Strongyloides stercoralis C. Brugia malayi D. Taenia solium E. Toxoplasma gondii

E. T. For the most part, specific serology beyond elevated IgE following parasitic infection is not available. The exception is Toxoplasma, which is a potentially catastrophic cause of transplacental (the "T" in TORCH) infection if it is primarily acquired by a woman during pregnancy. QCCP2,Laboratory methods

76. Which of the following disorders, also known as Duncan disease, is characterized by hepatic necrosis with a profound NK/T cell infiltrate? A. Reye syndrome B. immunoreactive cirrhosis C. hepatitis mononucleosis D. primary hepatic lymphoma E. X-linked lymphoproliferative disorder

E. X-linked lymphoproliferative disorder. As an X-linked disorder, primarily men are affected. The range that disease can manifest extends from the previously mentioned hepatic necrosis and death to less severe agammaglobulinemia or B-cell lymphoma. The disorder is due to a defect in the SAP gene, which leads to uncontrolled NK/T cell activation. QCCP2,EBV

87. Which disease is associated with an inherited defect in the ability to defend against several HPV subtypes? A. Li-Fraumeni B. recurrent respiratory papillomatosis C. progressive multifocal leukoencephalopathy D. post-transplant lymphoproliferative disorder E. epidermodysplasia verruciformis

E. epidermodysplasia verruciformis. An autosomal recessive condition mapped to a gene on chromosome 17, EV manifests with upper extremity lesions within the first 10 years, which can progress to invasive squamous cell carcinoma. QCCP2,HPV

105. What is notable about the culture of rhinovirus? A. several weeks of incubation B. culture must contain support virus C. high salt media is needed D. requires addition of nasal mucus to culture E. must be incubated at 32°C

E. must be incubated at 32°C. The only unique property of rhinovirus is that it should be cultured in a temperature slightly lower than body temperature. The thought of adding nasal mucus to a culture is a little more than I can bear. QCCP2,Rhinovirus

calcium, magnesium

EDTA will chelate these analytes, grossly deflating their value:

2

Each ferritin protein can hold ___ atoms of iron.

cal, slope

Each sensor in the blood gas analyzer requires two calibrators:

Binding Forces

Electrostatic (ionic), Hydrogen (H to two eneg atoms), Van der Waals (random), and Hydrophobic (clumping), all noncovalent, binding is reversible.

uremia

Elevated blood levels of non-protein nitrogenous waste products PLUS other signs and symptoms of renal failure is a condition known as:

respiratory acidosis

Elevated pCO2 corresponds with this condition:

Addressin

Endothelial surface molecules used to direct WBC migration via adhesion, specific by location and cell type needed, may be up or downregulated based on need.

Mannose Binding Lectin Associated Protease

Enzyme of the --- Pathway Complement Cascade, analogous to C1.

Physical Barriers

Epithelia, mucosa, pH levels, natural flora, Defensins, peristalsis, coughing and sneezing, etc.

Stress Proteins

Expressed on the surface of ---/infected cells, bound by gd T and NK Cells to induce apoptosis (via NK receptors), via MIC-A and MIC-B receptors on target cell.

kernicterus

Extremely elevated levels of unconjugated bilirubin can form inclusions in the brains of neonates, whose blood-brain barrier is not yet intact. This condition is known as:

liver

Ferritin and hemosiderin are found mostly in the:

diurnal variation

Ferritin is a better marker of iron treatment than serum iron because it does not exhibit:

Terminal Pathway

Final self-assembly process of the Complement System, C8 pokes initial hole in pathogen membrane, C9 polymerizes into 12-18 molecules and inserts as an open pore (hydrophobic outside, hydrophilic inside) allowing passive water and solute diffusion down their gradients, causes cell lysis or virus disruption.

Kupffer Cell

Fixed liver sinusoid macrophage, filters portal venous blood from GI of pathogens from within the vessel lumen, failure = Septicemia.

Intraglomerular Mesangial Cell

Fixed macrophage in the kidney, filters pathogens from blood flowing through the glomeruli.

Microglia

Fixed macrophages in the brain, regulatory, help maintain the BBB, but with limited killing ability, poor control of successful invasion.

Follicular Dendritic Cell

Fixed within lymphoid ---, trap antigens long term on their surface via C3b and Fc receptors, forms Lccosomes for B Cell binding.

anaerobically

For accurate results, collection, processing, and analysis of arterial specimens must be done:

loss

For calcium and magnesium, renal disease favors:

Neutrophil Receptors

For cell adhesion, complement system, antibodies, and cytokines.

75g

For glucose tolerance tests done on non-pregnant patients older than 12 years of age, the dosage of glucose should be:

1.75 g/kg

For glucose tolerance tests done on patients younger than 12 years of age, the dosage of glucose should be:

Neutrophil Function

For initial/acute response to invasion/injury, contains lysosomes for killing small phagocytosed pathogens and digesting tissue, numbers are vital for health.

TH1 Function

For intracellular pathogens, releases IFNg to stimulate Macrophages and IgG production. Also stimulates WBC production with TNFa, GMCSF, and IL3.

TH2 Function

For parasites and allergens, releases IL-4,5,13 to stimulate Mast Cells and Eosinophils, and IgE production. Also stimulates B-Cells via synapse and direct cytokine release.

retention

For phosphates, renal disease favors:

4

For proficiency testing, 5 specimens are sent to a laboratory and at least ____ of them must return correct results in order to pass.

TH17 Function

For rapid Neutrophil recruitment in response to rapid infection, releases IL-17A,17F,22. Also stimulates Monocytes.

50g

For screening glucose tolerance tests done pregnant patients, the dosage of glucose should be:

TH Regulatory Function

For suppression of immune response and maintenance of self tolerance.

Strong Antigen

Foreign, large (> 10k, may bind small ones (Haptins) to a carrier to form a complex), chemically complex, rigid, and mildly degradable, proteins best.

Glycogen Granules

Found in Neutrophils, used as source of glucose for anaerobic glycolysis as source of energy when away from blood supply.

IgG Function

Found in blood, interstitium, placenta, and colostrum. Does complement activation, toxin/virus neutralization, opsonization, and passive transfer.

Sentinel Cells

Found under epithelia, carry PAMP receptors, secrete Cytokines and pro-inflammatory factors upon activation to trigger Innate Immunity. Includes Dendritic Cells, Macrophages, and Mast Cells.

Lccosome

Free membrane blebs produced by Follicular Dendritic Cells, contains trapped antigens for B Cell binding and phagocytosis.

toxicology

Freezing point depression is the preferred osmolality methodology for this application:

Which of the following components has ALL of the coagulation factors

Fresh frozen plasma

7-10 days

Fructosamine can be used to extrapolate the average blood glucose over the last:

Which of the following carbohydrates are needed to produce blood type B

Fucose and Galactose

Which of the following antigens are destroyed by enzymes such as ficin or bromelin

Fy(a), Fy(b), S

Select the cell below that should be used for the positive control when testing commercially prepared anti-Fy(a)

Fy(a+b+)

VDJ Segments

Germline Heavy Chain exons divided into three loci that are permanently mixed and matched for future Fab region as lymphocyte matures (ends adjacently), paired with desired Constant Fc region before transcription.

Red Blood Cell casts in urine is indicative of

Glomerulonephritis

alpha cells

Glucagon is produced in the ______ of the pancreas.

close to normal

Glucose level should be ______ by the 2nd hour of the 5HGTT.

165

Glucose will typically begin to show up in urine when blood glucose is >____ mg/dL.

hexose

Glucose, galactose, fructose, and mannose are ______ monosaccharides.

Resp. Alkalosis Compensate

Go into Metabolic Acidosis

Resp. Acidosis Compensate

Go into Metabolic Alkalosis

Metabolic Alkalosis Compensate

Go into Resp. Acidosis (hypoventillate)

Metabolic Acidosis Compensate

Go into Resp. Alkalosis (hyperventillate)

Granular Cast

Granular casts are simply older epithelial cell casts in which the epithelial cells have degenerated to the point that they can no longer be identified as individual cells. Granular casts are of two forms: coarsely granular (early stage) and finely granular (late stage). Both forms are interpreted as evidence of tubular degeneration.

We find ABO hemolytic disease most often in babies born to moms who are which ABO type

Group O

If there is no time to determine a patient's ABO and Rh type because he is profusely bleeding, what ABO/Rh type should be transfused

Group O, Rh negative

A mom is group O positive and has anti-Fy(a), anti-K and anti-E. The baby is group A neg and has a positive DAT. The eluate from the baby's cells shows anti-Fy(a). Knowing that any blood transfused to this newborn must be compatible with the mom's serum, select the correct blood type to transfuse this infant.

Group O, Rh negative and negative for E, K and Fy(a) antigens

If a patient is blood type O, which of the following antigens are on his cells

H antigen only

If a person has only H blood group substance, what antigens will be found on his red cells

H antigens only

-a defective Hb that alone is not associated with any severe clinical manifestation -when combined with sickle cells (Hb SC), increases risk of thrombosis -Results from AA sub. at pos. 6 in beta chain Lysine substituted for glutamic acid

Hb C

-Normal fetal hemoglobin from 3rd to 9th month. -Has two alpha & two gamma subunits (α2ɣ2) -higher affinity for O2, shifts ox-diss. curve left -comprises <1% Hb in normal adult -elevated in β-thalassemia

Hb F

-4 beta chains (β4) -present in α-thalassemia -biologically useless -precipitates, shortening RBC life

Hb H

-Sickle Cell Hb (α2β2s) -Results from AA sub. at pos. 6 in beta chain Valine substituted for glutamic acid -when deoxygenated, hydrophobic valines aggregate, causing crystal polymer -crystal polymer causes sickle shape

Hb S

charge, affinity to dihydroxyboryl groups

HbA1C is differentiated from normal hemoglobin by:

glucose control

HbA1C is most useful in determining ______ in already-diagnosed diabetics.

4-6 weeks

HbA1c can be used to extrapolate the average blood glucose over the last:

Complement System Inactivation

Heat to 56 degrees celsius for 30 min (deactivates Complement but not antibodies) or remove/chelate calcium (also deactivates coagulation, but reversible, use EDTA or Acid Citrate Dextrose). Adding Heparin only deactivates coagulation.

heavy metals poisoning

Heme synthesis blockage is usually caused by:

400, 600

Hemoglobin absorbs light between _____ and _____ nm.

bilirubin, iron, globin

Hemoglobin is broken down into three chemicals:

carbaminohemoglobin

Hemoglobin which has carbon dioxide bound to it may be called:

carboxyhemoglobin

Hemoglobin which has carbon monoxide bound to it may be called:

methemoglobin

Hemoglobin which has had its iron atom oxidized may be called:

potassium

Hemolysis may cause artificial elevation of any of the analytes found within the erythrocyte, especially:

potassium, magnesium, phosphate, iron

Hemolysis will result in a huge increase in these analytes:

Which of the following clinical applications uses the DAT

Hemolytic Transfusion Reactions (HTR)

store glycogen

Hepatic disease can cause hypoglycemia by rendering the liver unable to functionally:

hemoglobin F

High amounts of _____ will cause a falsely elevated HbA1C.

Eosinophilia

High blood --- count, indicates parasitic infection or allergies.

calcium phosphate

High blood calcium may lead to the formation of ______ crystals and renal calculi.

calcification

High blood calcium may result in _____ of soft tissues.

weakness

High blood calcium will have this effect on muscles:

Contraction

High cell count due to clonal expansion decreases after infection for homeostasis.

tissue, vascular system

High osmotic pressure forces fluid from the ______ to the ______.

Which of the following Rh typing reagents must ALWAYS be used with a control

High protein anti-D

hypertension

High serum osmolality is associated with this cardiovascular effect:

higher

Higher resistance correlates with _____ purity.

hyperglycemia

Higher than normal blood glucose level is a condition known as:

Hyaline Cast

Hyaline casts are composed of mucoprotein and are seen with mild renal injury and glomerular leakage. Febrile animals with normal kidneys may have occasional hyaline casts in the urine.

oxygen from the sample

Hydrogen gas within a pO2 electrode will combine with ______ and form water.

Neutrophil Membrane

Hydrophobic, may be repelled by bacteria unless they're coated in antibodies/C3b and opsonized.

diabetes mellitus

Hyperglycemia due to problems with insulin production or functionality is diagnosed as:

pernicious anemia, trauma, heavy metal poisoning:

Hyperuricemia may occur when cells are dying more than usual, for example in these conditions:

malignancy, leukemia, leukocytosis, polycythemia

Hyperuricemia may occur when cells are proliferating more than usual, for example in these conditions:

Resp. Alkalosis Caused by

Hyperventillation

hypotension, edema, weakness, neurological failure

Hyponatremia can cause:

Resp. Acidosis Caused by

Hypoventilation (asthma, emphysema, pneumonia, throat obstruction)

membrane potential

ISEs work by measuring the _______ between the unknown sample and a fixed standard.

Metabolic Alkalosis Caused by

Iatrogenic (treatment with drugs, chemicals that bring up bicarb) Antacids (Tums, rolaids) and Cushings dz.

WBC cast

Identify

Calcium oxalate crystal

Identify envelopes-shaped crystal

unconjugated bilirubin

Identify this molecule.

glutamic dehydrogenase

Identify this reaction: 2-oxoglutarate + NH₄+ NADH + H -> glutamate + NAD + H₂O

nessler

Identify this reaction: NH₃ + HgI + KI -> NH₄ + Dimercuric Iodate

berthelot

Identify this reaction: NH₃ + NaClO + phenol -> indophenol + NaCl + H₂O

jaffe reaction

Identify this reaction: creatinine + tri-nitro-phenol + NaOH -> red color change

henry-caraway phosphotungstate reaction

Identify this reaction: uric acid + phosphotungstic acid + O₂ -> allantoin + CO₂ + tungsten blue

uricase

Identify this reaction: uric acid -> allantoin + H₂O₂

Dysmorphic RBC

Identify urinary sediment

myxedema

Idiopathic hypothyroid disease that typically presents in middle age and may be genetic is:

rejected

If 10 consecutive QC values are on one side of the mean, it should be:

not reported, recorded

If QC fails, patient results should be _______ and QC values should be _____.

failed

If QC values fall outside the acceptable range, the QC is said to have:

% T

If X is the amount of light which strikes a specimen and Y is the amount of light that exits the specimen, Y/X =

stray light

If a 680nm cutoff filter is placed in a spectrophotometer and it is dialed to 700nm, any light hitting the detector is:

xy

If a dilution of 1:x is made, then further diluted by 1:y, the overall dilution factor is:

contamination with iv dextrose

If a glucose result is >1500 mg/dL, this should be strongly suspected:

2

If a lab fails proficiency testing for an analyte, they must pass the next ____ challenge cycles or they will be unable to perform that assay.

calibration

If both levels of QC fail and they have both moved in the same direction, this is most likely the cause:

chylomicrons

If lipemia is caused by not fasting, the fat particles are known as:

rejected

If one level of QC exceeds 2σ from the mean on two consecutive runs, it should be:

examined

If one level of QC exceeds 2σ from the mean, it should be:

rejected

If one or both levels of QC exceed 3σ from the mean, it should be:

hemolysis

If the assay method uses a λ <600, _____ will act as an interfering factor.

report as <x mmol/L

If the bottom limit of linearity for a given assay is x mmol/L and the value given by the instrument is lower, the tech must:

iron2

If the first iron run is designated iron1, and the second (force loaded) iron is designated iron2, TIBC=

iron2 - iron1

If the first iron run is designated iron1, and the second (force loaded) iron is designated iron2, UIBC =

protein buildup on membrane

If the response time of an ISE increases or it becomes less reproducible/stable, this may be the culprit:

dilute and rerun

If the upper limit of linearity for a given assay is x mmol/L and the value given by the instrument is higher, the tech must:

Which immunoglobulin class is the blood banker most interested in

IgG

Germline Constant Region

IgM and IgD are first in series, permanently looped out for Class Switching until desired region is adjacent to VDJ, may switch again if next desired region is still present (IgA last).

Band Cell

Immature Neutrophil released before nucleus is lobed, occurs when demand is unusually high.

Innate Immunity

Immediate and nonspecific, triggered by "danger signals" from pathogens or tissue damage, activates Adaptive Immunity. Includes physical barriers, Natural Killer Cells, Phagocytes, etc.

Tolerance

Immunity ideally does not attack body proteins, all antigens present during fetal development (including pathogens) are tolerated, can be reversed later in life.

Monoclonal Selection

Immunized animal's B-Cells are fused with Myeloma Cells for indefinite expansion (Hybridoma), correct antibody/B-Cell type is selected out of result.

indirect bilirubin, urine urobilinogen

In Dubin-Johnson Syndrome, all bilirubin results are high except:

deposited in tissues

In Wilson's disease, excess copper cannot be excreted and is:

alkalosis

In ______, ionized calcium will decrease without a change in total calcium.

acidosis

In _______, ionized calcium will increase without a change in total calcium.

potential interference AND analyte

In a bichromatic analysis, the primary λ should be one which is absorbed by:

7.4-7.45

In a fully compensated condition, a pH of ____-____ indicates alkalosis.

ionic

In a healthy patient, the bond between unconjugated bilirubin and albumin is:

98, 2

In a healthy person, ___% of hemoglobin is hemoglobin A1 and ___% is hemoglobin A2.

increase

In a healthy person, bicarbonate should ______ as chloride decreases.

reaction rate

In a kinetic assay, the _____ is proportional to analyte concentration.

dietary

In a non-fasting patient, many different sugars of _____ origin may be found in the bloodstream.

metabolic ketoacidosis

In a non-starvation state, ketone bodies produced will build up in the bloodstream and result in this condition:

ketone bodies

In a patient with a very high glucose, an abnormally elevated osmolal gap likely signals the presence of:

high, low

In a patient with hepatic disease, ammonia will be _____ while urea will be ______.

incident

In a spectrophotometer, the light that hits the specimen is the ____ light.

central nervous system

In a starvation state, ketone bodies will be used as an energy source by the:

neurotoxin

In addition to being a very strong base, ammonia is a rather potent:

pH, temperature

In addition to contaminants, these can also cause false values on fluorescence assays:

explosive

In addition to lack of specificity, the Jaffe reaction was problematic because picric acid is extremely:

sweat

In addition to urine and serum, osomolality may be measured on this body fluid:

decreases

In alkalosis, plasma potassium:

low

In an arterial specimen which is exposed to air, pCO2 will be falsely:

high

In an arterial specimen which is exposed to air, pH will be falsely:

high

In an arterial specimen which is exposed to air, pO2 will be falsely:

water

In both water and reagent blanks, the volume of what would be a specimen should be replaced with:

ketone bodies

In diabetics with high anion gaps, the mystery anion is usually:

sulfanillic acid, sodium nitrate

In diazo reactions, ______ is combined with _____ in HCl to produce a diazo compound which is red in color when bilirubin is present.

half

In hepatocellular jaundice, dBili should make up _____ of tBili.

liver

In humans, glycogen for use by the whole body is stored in the:

iatrogenic

In most cases, metabolic alkalosis is:

at an angle

In nephelometry, the light source is ______ to the photomultiplier.

all

In obstructive jaundice, dBili should make up _____ of tBili.

35 mg/dL

In order to diagnose VonGierke's, neither of the glucose levels can be >_____ above the baseline FBS.

validate 24 hour urine assays

In patients with healthy kidneys, urine creatinine may be used to:

ferritin

In patients with known iron deficiency, treatment may be monitored by evaluating:

exchange transfusion

In serious cases of neonatal jaundice, ________ may be required to prevent brain damage from kernicterus.

water

In the Evelyn-Malloy methodology, direct bilirubin is measured in:

ethanol

In the Evelyn-Malloy methodology, total bilirubin is measured in:

pH

In the Jendrassic-Grof methodology, bilirubin is fractionated by differences in solubility at different:

a given wavelength

In the equation Aλ = abc, λ is:

urobilinogen

In the intestines, conjugated bilirubin is converted to:

oxidation of the chromagen

In the traditional glucose oxidase methodology, peroxidase catalyzes the transfer of an oxygen molecule to a chromagen, leading to:

ammonia

In the urease reaction, the actual analyte being measured is:

urease

In this reaction, an enzyme converts urea to a different chemical which can be measured by an existing assay:

multi-point kinetic

In this type assay, a reagent is added to the specimen and the absorbance is taken two or more times during the reaction:

end point

In this type of assay, a reagent is added to the specimen and the absorbance is taken once the reagent is complete:

Phagocytes

Include Neutrophils and Macrophages, part of Innate Immunity.

Immunologic Memory

Includes B and T-Cells, takes ~1 wk to respond to initial exposure, subsequent exposures trigger Anamnestic Response.

Killer Cells

Includes CD8 and NK Cells, large and granular, use similar mechanisms to induce apoptosis via membrane contact.

Immune Dysfunction

Includes Hypersensitivity, Allergies, Autoimmunity, and Immunodeficiency.

Cell-Mediated Immunity

Includes T-Cells ab TCR and gd TCR.

Leukocytes

Increased numbers with normal urinary protein indicate inflammation probably in the u. bladder. Increased numbers with elevated protein indicate inflammation in the kidneys.

Viral Resistance

Increases degradation of viral mRNA, inhibits viral protein assembly, and resists viral IC replication.

Alternative Pathway

Innate mechanism, C3 (most common) tends to spontaneously degrade and activate while in circulation, normal body cells express protective surface proteins Factor H (bound to membrane Salic Acid) to bind C3b and Factor I to cleave/inactivate it. Otherwise Factors B, D, and P stabilize it into a C3-cleaving enzyme, causing rapid amplification leading to Terminal Pathway.

Lectin Pathway

Innate mechanism, uses Mannose-binding Lectin to begin Complement cascade by binding to Mannose sugars in pathogen membranes (unique to bacteria), activates Mannose-binding Lectin-associated Protease, which acts as C1s to begin cascade, leading to Terminal Pathway.

Proinflammatory Cytokines

Innate type, includes IL1, IL6, and TNF.

IL12

Innate type, released by Sentinel Cells, activates NK and TH1 for bacteria and viruses.

Type 1 Interferons

Innate type, released by viral infected cells via IC PAMP-Rs, , includes IFNa/b. Also increases MHC1 expression of infected cell.

beta cells

Insulin is produced in the ______ of the pancreas.

DM1, DM2

Insulin, or more typically C-peptide, is used to differentiate between:

quenching

Interference in fluorescent assays is termed:

Acute Inflammation

Invasion of tissues/damage causes Sentinel Cell activation via D/PAMPs, which secrete Cytokines and vasoactives (permeability), antibacterials (Defensins), recruit Neutrophils before Macrophages. Includes dolor, rubor, calor, and tumor.

dioctylphenyl phosphate in a polymer membrane

Ionized calcium ISEs contain:

pushes the dioctylphenyl phosphate out

Ionized calcium ISEs work because ionized calcium:

ISE

Ionized calcium is measured by:

ISE

Ionized magnesium is measured by:

contamination

Iron assays are extremely sensitive to:

500

Iron becomes toxic at concentrations >____ μg/dL.

ferrous

Iron which is contained within cytochrome molecules such as hemoglobin is in this state:

ferric

Iron which is is bound to transferrin, ferritin, or hemosiderin, is in this state:

hemoglobin

Iron/TIBC values will be skewed from a hemolyzed specimen due to this interfering factor:

What must be done to prevent graft-versus-host disease

Irradiate the cellular products to be infused.

Granzymes

Killer cell granule enzyme, enter target cells through Perforin channels, activate Intrinsic apoptosis.

Perforins

Killer cell granule enzyme, forms a hollow complex in target cell membrane similar to complement C9 MAC, allows Granzymes through to cytoplasm.

Granulysin

Killer cell granule enzyme, induces apoptosis.

Tetanus

Kills a naive animal before it can produce IgG (~14 days), which is the only effective antibody type.

specimen chromophores

Kinetic assays are more useful than end-point assays in that they eliminate interference from:

prognosis

Lactate can help in triage of burn and trauma patients because it is a useful indicator of:

tissue hypoxia

Lactate is a measure of:

glucose, galactose

Lactose is a disaccharide composed of:

What is the systolic blood pressure limits for an allogeneic donor

Less than or equal to 180

What is the cause of febrile transfusion reactions

Leukoctye antibodies present in the patient's plasma that react with leukocytes in the blood

If a patient has febrile transfusion reactions, how are future transfusions handled

Leukoreduce the cellular products to be infused.

If there is no blood that is tested CMV negative, which of the following is considered CMV "safe"

Leukoreduced components

FIV

Levels of serum Ag are too low to test, uses Indirect ELISA as SNAP Test.

Which of the following blood group systems produces antibodies that do not cause hemolytic disease of the newborn

Lewis

Fab

Light and Heavy chain arms of the antibody, includes region where Epitopes bind, imparts physical effects (agglutination, neutralization, and blocking).

transmitted

Light which strikes an object and passes through it is said to be:

reflected

Light which strikes an object and then moves away from the object is said to be:

Avian Immunity

Like mammalian, use Heterophils, IgY instead of IgM/G, lymphoid nodules, absence of IgE, higher gd T Cell count.

all wavelengths

Lipemia increases absorbance at:

colorimetric

Lipemic, icteric, and hemolyzed specimens will often show artificially elevated results on this type of assay:

LiCl

Lithium ISEs contain a solution of:

vascular system, tissue

Low osmotic pressure forces fluid from the ______ to the ______.

dietary iron deficiency, chronic blood loss, pregnancy, malabsorption

Low serum iron is correlated with these conditions:

hypotension

Low serum osmolality is associated with this cardiovascular effect:

Cytokines

Low-weight protein messengers produced by activated immune cells, includes Pro-Inflammatories, Chemokines, Colony-Stimulating Factor, Growth Factors, and Interferons. Can be autocrine, paracrine, or endocrine. Regulate the strength and duration of immune responses, antibody type produced, and bone marrow activity.

Fish Antibodies

Lower affinity, less diverse, weaker in general. No hypermutation process (affinity maturation).

hypoglycemia

Lower than normal blood glucose level is a condition known as:

Recombinase

Lymphocyte enzyme, acts to recombine Germline DNA for antibody selection and class switching via looping out. Unique to the immune system.

Natural Killer Cells

Lymphocytes, nonspecific and innate, act to kill cells expressing abnormal proteins, works against infected cells and cancer cells.

Myeloperoxidase

Lysosomal enzyme, causes conversion of Hydrogen Peroxide and Chloride into Hypochlorous Acid (active ingredient of bleach), used by Neutrophils to kill phagocytosed bacteria.

Alternative Activation

Macrophage function is upregulated by Interleukin cytokines, becomes type M2 cell for healing and tissue repair, decreased killing ability, anti-inflammatory, late response.

Innate Activation

Macrophage function is upregulated by local D/PAMPs binding to TLR type receptors, later becomes type M1 cell with Interferon-g classical activation for further upregulation, kills a wider range of pathogens faster and more successfully, pro-inflammatory (release nitrous oxide), early response.

Uric Acid (Serum/Plasma)

Males 3.5 to 7.2 mg/dL Females 2.6 to 6.0 mg/dL

Serum Iron

Males 50 to 170 ug/dL Female 30 to 160 ug/dL

Plasma Cell

Mature B-Cell, produces large amounts of its specific antibody in free soluble form from the Spleen or bone marrow.

Lymphocyte

May be B, T, or NK (granular) Cells. Large peripheral nucleus. Are the only WBC that return to circulation once they leave, due to continuous transition btw blood and lymph. Circulatory life is 4 mo.

Epitope

May be Continuous (linear binding sequence) or Discontinuous (binding region only present when folded).

Fish Vaccination

May be injected, administered orally, or done via submersion (best) for mucosal exposure (including Lateral Line), but results in low serum levels. Injection results in high serum but low mucosal levels, match method to disease entry type.

B-Cell Class Switch

May be performed by TH Cells with recognition of any processed and presented peptides, stimulates production of cell's original Fab region as new antibody class, regardless of presented fragment.

Avian Vaccination

May be performed via injection into egg or sprayed onto body surface after birth for ingestion (both active immunity).

Abscess

May form after Neutrophil activity, contain pus and dead neutrophils.

Neutrophil Oxidative Metabolism

Mechanism by which Neutrophils generate lethal reactive oxygen species, only works near blood flow, produces Superoxide Anion --> Hydrogen Peroxide, Hydroxyl Radical (from Fe), causing highly reactive oxidation and breakage of covalent bonds (including Neutrophil and surrounding tissue), results in Aldehyde byproducts (also lethal).

Diapedesis

Mechanism by which Neutrophils move out of vasculature into surrounding tissue, digest tight junctions after binding to Integrins, which are later reestablished.

Chemotaxis

Mechanism by which Neutrophils move to a site of invasion/damage, follow a gradient as read by either side of the cell (may be C5a)

Hypermutation

Mechanism for increasing antibody affinity by altering VDJ region, occurs in Germinal Centers with activated B Cells at every division during clonal expansion, usually results in deficient binding -> apoptosis. Successfully increased binding causes higher stimulation by TH Cells.

Anergic Response

Mechanism of tolerance, T Cell becomes persistently unresponsive to a presented molecule when Costimulatory proteins are absent.

The commercially prepared anti-A and anti-B are

Monoclonal antibodies IgM antibodies Prepared with a colored dye added

IgE

Monomer antibody, has five Heavy Chain domains and many carbs, Fc has uniquely high affinity for Mast Cell Fc surface receptors, used to cover Sentinel Mast Cells for sensitization and recognition of antigens (Homocytotropic), also attracts Eosinophils.

Macrophage

Mononuclear phagocytotic cell, high membrane content, very long-lived once in tissues, presence at site indicates a chronic infection.

What is considered a significant difference in the titer of two samples when they are being compared

More than a two tube difference

cereuloplasmin

Most commonly, copper is measured indirectly by testing for:

biliary tract, intestines

Most conjugated bilirubin is transported via the _____ to the ______.

oral

Most glucose tolerance tests are _____, meaning the glucose solution is taken by mouth as a drink.

blood urea nitrogen

Most labs report urea as:

Acute tubular necrosis

Muddy brown cast

Cytokine Synergy

Multiple --- induce an increased, cumulative effect.

Cytokine Redundency

Multiple --- induce the same effect.

Cytokine Antagonism

Multiple --- with effects that oppose each other.

Complement System Regulation

Multiple soluble and membrane-bound factors in normal body tissues act to prevent activation of Complement System, works even if Factor H is overwhelmed, incompatibility results in Xenograph organ rejection.

hypoglycemia

Myxedema and Hashimoto's disease cause:

photomultiplier

Name this part of a spectrophotometer.

primary exit slit

Name this part of a spectrophotometer.

readout device

Name this part of a spectrophotometer.

respiratory acidosis

Nearly all disorders of the respiratory system will result in:

immunochemistry

Nephelometry is useful for this type of test because it can detect light scatter from immunocomplexes:

Waxy casts in the urine is indicative of

Nephrotic syndrome

hepatic encephalitis

Neurological problems caused by high ammonia levels are termed:

3

Newborns typically begin to produce the enzyme beta-d-glucuronotransferase at about ___ days old.

Fish Immunity

No bone marrow or lymph nodes, but similar to mammals. Complement system with wide temperature range (4-45 `C). +/- Eosinophils, heterophils, and basophils by species. Can undergo allograft rejection. Use IgM for secreted mucosal defense.

I, III

Normal QC procedures use these levels of QC:

Calcium oxalate dihydrate

Normal in urine seen in both acidic and alkaline

fully-compensated condition

Normal pH but abnormal bicarb and pCO2 indicates:

Class Switch

Occurs with an activated B-Cell via looping out of unneeded C regions, maintains permanent VDJ regions.

Tolerance Development

Occurs with antigens that are long-lived, entered via mucosa or circulation, presence in generative areas (bone marrow, lymph nodes) absent adjuvant (danger signal), low costimulators/cytokines.

Septic Shock

Occurs with high levels of cytokines released into circulation, causes clotting, thrombosis, and deficient cardiac output and vasodilation, leading to edema and hypoxia. May be lethal.

Urine color = black is indicative of

Ochronosis (The syndrome caused by the accumulation of homogentisic acid in connective tissues. The phenomenon was first described by Rudolf Virchow in 1865. The condition was named after the yellowish (ocher-like) discoloration of the tissue seen on microscopic examination. However, macroscopically the affected tissues appear bluish grey because of a light scattering phenomenon known as the Tyndall effect. The condition is most often associated with alkaptonuria but can occur from exogenous administration of phenol complexes like hydroquinone.)

original absorbance, specimen blank absorbance

Once a specimen blank is run, the equation for removing the interference is ______ - ______.

If the qualitative test for a feto-maternal bleed is negative, how many vials of Rh Immune Globulin must be injected into the mom

One because she is a candidate; that's why the test was done.

If a person has the Le, se and H genes, what substance(s) are in his body fluids

Only Le(a) blood group substance

T-Cell

Only capable of recognizing linear, processed peptides as presented to them on MHCs.

Squamous Epithelium

Originate from the distal 1/3 of the urethra, vagina, or prepuce Primary source is the urethra. Small nuclei. LARGEST epithelial cell.

Renal Tubular Epithelium

Originate from the kidney Larger than WBC, and smaller than transitional Increased numbers suggests tubular damage.

sodium, BUN, glucose

Osmolality may be calculated using these values:

oncotic pressure

Osmotic pressure that is the result of proteins is termed:

AML - acute myeloid leukemia

PE: BT 38, moderate pallor, no Jx, *ecchymosis all extremities* CBC: Hct 27% MCV 84 *WBC 3000* *LYMP 80%* *Plt 90,000* *RBC* : microcyte 1+, schistocyte few *WBC* : Matured Neutrophil not seen, Promyelocyte seen, Auer's rod seen, fagot cell seen *Plt* : decreased platelet smear *Diagnosis* _____

Iron deficiency anemia

PE: moderate pallor, no Jx, *SEM gr. III LUPSB* CBC: *Hct 15%* MCV 70 WBC 7,500 PMN 70% *Plt 560,000* *RBC* : microcyte 3+, hypochromic 2+, anisocytosis 2+, elliptocyte 1+ *WBC*: Neutrophil predominate, normal morphology *Plt* : increased platelet smear *Diagnosis* _______

Pernicious anemia

PE: moderate pallor, no Jx, *glossitis*, *impaired proprioceptive sensation* CBC: *Hct 21* *MCV 120* WBC 5,000 Plt 240,000 *RBC* : macroovalocyte 2+, microcyte 1+ *WBC* : hypersegmented Neutrophil seen *Plt* : adequate platelet smear *Diagnosis* ______

multiple myeloma

PE: pallor, no Jx, no point of tenderness *CBC* : Hb 9 Hct 26% MCV 82 WBC 8,800 Na 128 K 3.0 Ca 12.8 Albumin 3.0 Globulin 9.0 RBC: NCNC, Rouleaux formation seen

medical records

PHI is typically only confidential when attached to:

Natural Killer Cell

Part of innate immunity, not antigen-specific. Binds any cell not expressing normal MHC1 or normal MHC1-presented peptides. Also binds stress proteins. Must be deactivated by bound cell to prevent apoptosis.

PAMP

Pathogen-associated molecular patterns. Matching receptors carried by Sentinel Cells.

What is the minimal information that must be on a properly labeled tube of blood that will be used for compatibility testing

Patient's full name, unique identification number and date drawn

Feline Leukemia

Persistent retrovirus spread in saliva. ELISA uses viral/infected cell protein p27, vaccine does not yield a false positive.

TH Synapse

Physically connects to presenting cell and secretes cytokines onto surface if activated.

anaerobic

Plastic vacutainers are not idea for blood gas analysis because they are not:

negative exponential

Plotting x=concentration and y=%T of an analyte that follows Beer's law will give a(n) _____ line.

positive linear

Plotting x=concentration and y=absorbance of an analyte that follows Beer's law will give a(n) _____ line.

negative linear

Plotting x=concentration and y=log(%T) of an analyte that follows Beer's law will give a(n) _____ line.

Accessory Molecules

Produced by presenting cells in response to P/DAMPs, type of pathogen, location, etc., for recognition by T Cells.

B Cell Maturation

Produces IgM for selection maturation, later produces IgD after approval as B0.

3

Proficiency testing challenge cycles occur _____ time(s) a year.

patient samples

Proficiency testing samples must be run in exactly the same manner as:

Myeloid

Progenitor of Granulocytes, Macrophages, Megakaryocytes, Erythrocytes, and Dendritic Cells.

Apoptosis

Programmed cell death, appears as dark chromatin and nuclear blebs, with --- bodies.

IL1

Proinflammatory, danger signal released by activated sentinel cells, stimulates B Cells.

c-peptide

Proinsulin is cleaved into two parts, insulin and:

White Blood Cell casts in the urine is indicative of

Pyelonephritis

twice

QC may be run only _____, if it continues to fail it is illegal to keep trying.

2 levels of controls every run

QC procedures for batch assays typically require:

2 levels of controls every run

QC procedures for manual assays typically require:

2 levels of controls every shift

QC procedures for random access analyzers typically require:

3

QC which falls more than this many standard deviations from the mean always indicates a problem:

If a cell is positive for the antigens D, C, E, c and e, what is the most probable Rh phenotype

R1R2

Acute glomerulonephritis

RBC cast

echinocyte

RBCs with numerous surface regular protrusion with rounded ends, evenly distributed over the red cell surface

gaussian

Random error follows this type of distribution:

percentiles

Reference ranges are now typically calculated by excluding the top and bottom 2.5 ______.

standard deviations

Reference ranges were originally defined as ±2 _____ about the mean.

Secretory Component

Region on Dimeric IgA that allows it to travel to mucosal surfaces and protects it from resident proteolytic enzymes.

Antibody Combining Site

Region on the Fab tips where the Hypervariable Regions spatially come together, only present when folded, may bind multiple different epitopes in different subregions.

Transmembrane Anchor Segment

Region on the Fc tip of an antibody, clipped for class switching to Effector type.

Germinal Center

Region within a lymph node where activated Lymphocytes are undergoing clonal expansion.

Pro-inflammatory Factors

Released by activated Sentinel Cells, early and rapid, include IL-1, IL-6, TNF-a, and HMGB1. Travel to Hypothalamus to cause fever, malaise, anorexia. Increase liver synthesis of Acute Phase Proteins and WBC production by bone.

Metabolic Acidosis Caused by

Renal failure, shock, diabetic ketoacidosis, lactic acid buildup, asprin OD.

Phagolysosome

Result of a phagosome fused with lysosomes after Degranulation, fills with Defensins (bactericidal), Lactoferrin (iron chelator), digestive enzymes, Myeloperoxidase, and Lysozymes (dissolves g+ bacterial walls). Lethal components occur first under basic conditions, then digestive components take over under acidic conditions (protective).

on ice, STAT

Samples for lactate analysis should be collected ______ and processed/assayed _____.

Choose the correct statement below regarding screening cells and panel cells.

Screening cells include 3 separate vials and a panel includes 8-20 vials.

Thymic Nurse Cells

Secrete growth factors for T Cells.

Avian Passive Transfer

Secreted by oviduct epithelia onto egg albumen for ingestion, and IgY in yolk for absorption into blood.

Cytokine Secretion

Self-limiting, slow, brief, diverse cell sources and targets, may have multiple effects on target, can be local or systemic (by concentration), acts on specific Rs, regulated by multiple signals, can influence other ---s, usually induce transcription and protein synthesis, cell responses undergo negative feedback.

What is the first stage of agglutination called

Sensitization

Natural Killer Cell Receptors

Separate Activating and Inhibiting types. Activating R causes lysis if Inhibiting R is not bound by target cell. Can also bind Fc antibody regions for coated targets (ADCC).

Fasting Blood Glucose (FBS)

Serum 70 to 110 mg/dL

Chloride (mEq/L = mMol/L)

Serum 98 to 106 mMol/L Sweat Up to 35 mMol/L

diurnal variation

Serum iron levels exhibit this trait, making day-to-day comparison of iron levels difficult:

How long must the sample be stored following a transfusion of the patient

Seven days after the last transfusion

interpretation

Sexual harassment is defined not by intent, but:

hypoperfusion of the kidneys

Shock, hypotension, heart failure, blood loss, trauma, and surgery can result in pre-renal azotemia by causing:

IgM Structure

Similar to IgG but with one extra CH4, transmembrane bound on naive B-Cell surfaces and secreted type.

C-Reactive Protein

Similar to Serum Amyloids, pentamer acute phase protein produced by liver, sticks to pathogens like an antibody for opsonization and complement cascade.

bilirubinometer

Since bilirubin itself is a pigment, it is possible to directly measure it when levels are high using a special spectrophotometer known as a(n):

absorption

Since the purpose of the intestine is _______, some urobilinogen ends up in the bloodstream and is excreted renally.

Lymph Node Medulla

Site of exit for filtered lymph, to efferent duct.

Epigonal Organ

Site of hematopoiesis in cartilaginous fish (sharks).

Head Kidney

Site of hematopoiesis in teleosts.

Hypervariable Region

Site on Variable Regions where the AA sequence actually varies and binds the epitopes, small compared to the rest of the region. 4 in VH, 3 in VL.

Medullary Thymus

Site where approved T0 Cells lose either CD4 or CD8 before exiting to circulation.

Bronchial Pneumonia

Site-specific inflammation in the alveoli resulting from inhaled pathogens, may cause asphyxiation if Neutrophil response is too aggressive.

labile

Specimen collection and handling guidelines should be carefully adhered to with bilirubin samples because it is an extremely _____ analyte.

draw on ice, keep tightly capped

Specimen handling considerations for ionized magnesium and calcium are:

liquid

Specimens collected with this type of anticoagulant are most likely to be affected by short sampling:

buffered

Specimens for iron assay are _____ to 6.0.

heparinized plasma, serum

Specimens of choice for the electrolyte panel are:

photomultiplier

Spectrophotometric accuracy is a test of this part of the spectrophotometer:

neutral gray filters

Spectrophotometric accuracy is usually tested using:

ABO and Rh hemolytic disease of the newborn each have unique distinguishing characteristics. Which of the following red cell shapes is unique to ABO HDN

Spherocytosis

low

Stray light causes falsely _____ absorbance readings.

Endotoxin

Strong PAMP, consists of lipopolysaccharides.

fructose, glucose

Sucrose is a disaccharide composed of:

Affinity

Sum of an antibody-epitope's attraction and repulsion.

cystic fibrosis

Sweat osmolality is specifically used to diagnose this condition:

bacteria

Symptoms of meningitis with a low CSF glucose and elevated CSF lactate is probably caused by a(n):

virus

Symptoms of meningitis with a normal CSF glucose and lactate is probably caused by a(n):

gd TCR

T-Cell category cells on mucosa and within epithelia, which can directly recognize cell surface proteins.

ab TCR

T-Cell category including CD4+ T-Cells (Helpers) and CD8+ T Cells (Cytotoxics, granular).

Positive Selection

T0 must be capable of binding an endothelial MHC molecule in the thymus, otherwise undergoes apoptosis, occurs in the Cortex.

Negative Selection

T0 must not bind the presented contents of an endothelial MHC molecule in the thymus, otherwise undergoes apoptosis, occurs in the Cortex.

TRALI and TACO have similar symptoms, but they differ by which of the following

TACO patients suffer from hypervolemia and TRALI patients do not.

total binding capacity of transferrin

TIBC is a measure of:

force iron loaded

TIBC is calculated by running serum iron twice, once on a normal specimen and once on a(n) ________ specimen.

renal disease, nephrotic syndrome

TIBC is decreased in these conditions because of broad-spectrum protein loss:

chronic infection, chronic inflammation, mild hepatic disease, cirrhosis

TIBC is decreased in these conditions because of decreased production of transferrin:

pregnancy, iron deficiency, chronic blood loss

TIBC is elevated in these conditions to compensate for low serum iron:

hypokalemia

Tachycardia and arythmias, progressing to ventricular fibrillation and cardiac arrest are indicative of:

Direct ELISA

Tests for serum antigens. Ab fixed to sides, serum Ag added, conjugated Ab added (may be same as first Ab), then substrate added. Positive and negative controls also present.

Pemphigus

Tests for tissue autoantibodies. AAb are added to biopsy sample, then conjugated AAAb. Redundant layering is done for amplification of visible positive result.

taking two absorbance readings at different wavelengths

The Bu/Bc vitros slide fractionates bilirubin by:

kinetic reaction

The Jaffe reaction is not entirely specific for creatinine, but it is effective when used as a(n):

na - (bicarb + cl)

The Official Anion Gap Calculation of MLS 2211™ is:

unstandardized

The PPGTT is not as useful as an oral GTT because the carbohydrate load is:

refuse the glucola

The PPGTT is useful for patients who:

exposure control plan

The ____ is OSHA mandated and ensures that employees are protected against blood-borne pathogens and that medical waste is properly disposed of.

sensitivity

The ____ of a spectrophotometer is measured by bandpass.

higher, lower

The ____ the osmolality, the _____ the freezing point.

procedure manual

The _____ contains information on each test, including but not limited to purpose, principle, limitations, and reference/panic ranges.

higher, lower

The _____ the osmolality, the _____ the dew point.

higher

The _____ the pO2(50), the lower the binding affinity.

chemical hygiene plan

The ______ is OSHA mandated and must include a complete annual inventory and training for all employees handling chemicals:

hemolysis

The advantage of using Fehling II reagent during a Jendrassic-Grof assay is that it is not affected by interference from:

light absorbing molecules, color

The amount of _______ is proportional to the amount of _______ of a solution.

inversely proportional

The amount of fluorescence produced by an FPIA test is ______ to the concentration of analyte.

bilirubin

The analyte which causes the coloration of icteric specimens is:

chloride shift

The balance between bicarbonate and chloride is known as the:

hypoglycemic

The baseline FBS of an epinephrine tolerance test must be:

quartz glass

The best cuvettes are made of:

renal excretion of bicarbonate

The body can produce a state of metabolic acidosis as a compensatory mechanism by:

bicarbonate

The body uses this chemical to regulate blood pH by metabolic means:

carbon dioxide

The body uses this chemical to regulate blood pH by respiratory means:

bone salts

The body's store of phosphate is in:

Anti-Le(a) is usually IgM, but sometimes it can be IgG; however it will not cause hemolytic disease of the newborn. Why

The cells of newborns are Le(a-b-)

NH4

The chemical formula of ammonium is:

bilirubin diglucuronide

The chemical name of this molecule is:

abnormal anions

The chloride shift will not happen if ______ are present.

indophenol

The chromophore of the Berthelot reaction is:

dimercuric iodate

The chromophore of the Nessler reaction is:

direct bilirubin

The clinical name for conjugated bilirubin is:

renal calculi

The clinical term for kidney stones is:

kidney stones

The condition most commonly associated with post-renal azotemia is:

NADH, NAD

The conversion of _____ to _____ produces the absorbance change in the glutamic dehydrogenase reaction.

osmolal gap

The difference between a directly measured osmolality and a calculated osmolality is termed:

anion gap

The difference between the amount of anions and cations measured in the blood is the:

If a patient's serum sample is positive with all cells (screens, auto and donor cells) after the addition of AHG, what test will be your clue that there is something in the serum reacting with something in the entire test system

The direct antiglobulin test on the patient's cells

higher heat

The downside of the diacetyl- monoxime-thiemicarbazide reaction is that it requires ______ than most automated analyzers can provide.

18-70

The eGFR can only be used between these ages and is not validated for pregnant patients:

schiff base

The end product of the o-toluidine reaction is a variable compound known as a(n):

amperage increases

The endpoint of a chloridometer titration is identified as when:

glucono-lactone, NADH

The enzyme glucose dehydrogenase converts glucose and NAD to:

creatinine hydrolase, creatine kinase, pyruvate kinase, lactate dehydrogenase

The enzymes used in the creatinine hydrolase reaction are, in order:

concentration * total volume

The equation for calculating the total amount of an analyte from analyzing an aliquot is:

aldosterone

The excretion of sodium in the distal convoluted tubules is regulated by:

glucose-6-phosphate

The first step of glycolysis is the conversion of glucose to ______ by dephosphorylation of an ATP molecule.

phosphorylation of glucose

The first step of the hexokinase method involves:

run the sample normal

The first step of using a specimen blank is to:

scintillation

The flash of light released by some chemicals when they are oxidized is termed:

105, 190, 165, 145

The following are criteria for dx of GDM (pick two) : FBS > ____, 1hr >_____, 2hr >_____, 3 hr >______ (all in mg/dL).

24 hour urine creatinine, serum creatinine, 24 hour urine volume, patient height, patient weight

The following information is needed to calculate the creatinine clearance:

sodium silicate

The glass of a pH electrode is made from:

5, 10

The glucose level of arterial/capillary blood is about ___% higher than that of venous blood, typically a difference of about ____ mg/dL.

380

The glutamic dehydrogenase reaction will result in a drop in absorbance at ____ nm.

creatinine clearance

The gold-standard for diagnosing and monitoring renal function is:

pos, pos

The hepatocellular pattern for urine is _____ bilirubin and _____ urobilinogen.

stretched receptor theory

The idea that type II diabetes can be caused solely by obesity is known as the:

undetectable

The insulin level of a patient with type I diabetes mellitus should be:

normal

The insulin level of a patient with type II diabetes mellitus should be:

kalium

The latin term for potassium is:

natrium

The latin term for sodium is:

lack of specificity

The limitation of chloridometry is:

ammonia

The liver synthesizes urea from:

sodium

The main electrolyte which controls osmotic pressure is:

ferritin

The main storage form of iron is bound to:

lithium-specific ionophore

The membrane of a lithium ISE typically has this embedded in the glass:

sodium silicate, HCl

The membrane of a pH ISE is made from _______ and it contains ______.

valinomycin

The membrane of a potassium ISE is typically coated with:

monactin, nonactin

The membrane of an ammonia ISE is typically coated with:

Wilson's disease

The most common cause of copper overload is:

plastic syringe

The most common container for ABG specimens is:

ion specific electrode

The most common modern methodology for measuring electrolytes is:

hemoglobin S

The most common variant hemoglobin in the US is:

potassium

The most concentrated electrolyte within cells is:

glass syringe

The most ideal container for ABG specimens is:

OCPC

The most popular dye-binding calcium assay is:

phosphate, protein

The most significant anions in cells are:

chloride, CO2

The most significant anions in plasma are:

sodium

The most significant cation in plasma is:

potassium, magnesium

The most significant cations in cells are:

70-110

The normal range for fasting blood glucose is: ____-____mg/dL.

percentages of each gas, barometric pressure

The partial pressures of calibrator gases must be determined before it can be used, and the following information is required:

If a patient's sample is negative with the screen cells and the autocontrol, but one of the crossmatched units is incompatible after adding AHG, what may be the cause

The patient may have an alloantibody to a low frequency antigen present on that one donor's cells

If a patient's sample is negative with the screen cells and the autocontrol, but all the crossmatched units are incompatible at all phases, what may be the cause

The patient was incorrectly ABO typed.

If a patient's sample is negative with the screen cells and the autocontrol, but one of the crossmatched units is incompatible at immediate spin, what may be the cause

The patient's serum may have anti-M

freezing point

The point at which a super-cooled frozen liquid absorbs heat without an increase in temperature is the:

dew point

The point at which all condensation is reabsorbed is the:

glycogen

The polysaccharide made by animals from many glucose molecules is known as:

starch

The polysaccharide made by plants from many glucose molecules is known as:

0.1

The potassium value will be increased by ____ mmol/L for serum specimens as compared to heparinized plasma.

glutamic dehydrogenase

The preferred ammonia methodology is:

heparinized arterial whole blood

The preferred specimen for blood gas analysis is:

milli

The prefix meaning 10⁻³ is:

pico

The prefix meaning 10⁻¹² is:

micro

The prefix meaning 10⁻⁶ is:

nano

The prefix meaning 10⁻⁹ is:

high, normal

The prehepatic pattern for serum/plasma is _____ tBili and _____ dBili.

neg, pos

The prehepatic pattern for urine is _____ bilirubin and _____ urobilinogen.

polydipsia, polyuria, dehydration

The primary three symptoms of diabetes are:

ketone bodies

The production of ______ are a by-product of lipid metabolism.

gluconeogenesis

The production of glucose from non-carbohydrate molecules, such as amino acids and lipids, is:

increases

The production of hydrogen gas within a pO2 electrode ______ resistance.

lipogenesis

The production of lipids from glucose with Acetyl-CoA as the intermediary is:

decreases

The production of water within a pO2 electrode ______ resistance.

inhibit glycolysis

The purpose of fluoride in tubes used to collect glucose samples is to:

ionize protein-bound analyte

The purpose of the buffer in dye-binding reagents is to:

age, gender, race

The purpose of the eGFR is to standardize creatinine results against these factors:

coulimeteric titration chloridometer

The reference method for chloride analysis is:

atomic absorption spectroscopy

The reference method for serum iron assays is:

70-110 mg/dL

The reference range for blood glucose on venous serum or plasma is:

88-128

The reference range for creatinine clearance for females is ___-___ mL/min.

97-137

The reference range for creatinine clearance for males is ___-___ mL/min.

0.0-0.2

The reference range for dBili in adults is ____-____ mg/dL.

0.2-0.6

The reference range for iBili in adults is ____-____ mg/dL.

0.5-1.9 mmol/L

The reference range for lactate is:

289-308

The reference range for serum osmolality is _____ mOsm/kg.

0.2-1.0

The reference range for tBili in adults is ____-____ mg/dL.

300-900

The reference range for urine osmolality is _____mOsm/kg.

2-3

The reference range for urine-serum osmolality ratio is ___-___ mOsm/kg.

hemoglobin C

The second most common variant hemoglobin in the US is:

zero the instrument

The second step of using a specimen blank is to:

126

The simplest way to diagnose diabetes is to record at least two incidences of a fasting blood sugar >____ mg/dL.

hexiometer

The spectrophotographic instrument which is used to differentiate what is bound to hemoglobin is:

barometric pressure - 47

The sum of the partial pressures of CO2, O2, and N2 of a calibrator gas is:

take absorbance of specimen

The third step of using a specimen blank is to:

ionized, protein-bound, carbonates/phosphates

The three forms of calcium found in the blood are:

glucose oxidase, peroxidase

The traditional glucose oxidase methodology uses two enzymes:

triple distilled, deionized

The very purest water is:

400-700

The visible spectrum of light is about ___-___ nm.

25, 4

The xylose dose is ____ mg and at least ____mg of xylose should be excreted within 5 hours.

intestinal malabsorption

The xylose tolerance test is a measure of:

specimen

The λ of choice for reflectance photometry is the same color as the:

insulin stimulating drugs

Therapy for type II diabetes primarily consists of diet management and:

liquid

There is no _____ in a chloride ISE.

chronic infection, chronic inflammation

These conditions are correlated with low iron and low TIBC:

osteomalacia, ricketts

These conditions result in a decrease of calcium and no change in phosphate:

hemoglobinopathy, increased 2,3-dpg, anemia

These hematological conditions will cause a high pO2(50):

persistence of fetal hemoglobin, decreased 2,3-dpg

These hematological conditions will cause a low pO2(50):

T3, T4

These hormones are produced by the thyroid and raise blood sugar by stimulating both glycogenolysis and gluconeogenesis:

1/2, 1, 2

These hours of the 5HGTT are used to diagnose diabetes mellitus:

3, 4, 5

These hours of the 5HGTT are used to diagnose non-diabetic hyperglycemia and hypoglycemia:

high pressure liquid chromatography, hemoglobin electrophoresis

These methods may be used to identify variant hemoglobins:

parenchymal

These specific cells of the kidney are damaged by interstitial nephritis:

tubules

These specific structures of the kidneys are harmed in ATN:

sodium heparin, potassium heparin

These types of heparin should not be used for an electrolyte panel:

What is the protocol for transfusing neonates

They are given O negative cells of their own Rh type

Tumor Necrosis Factor Beta

Third mechanism to induce apoptosis, released by Killer Cells to bind their receptor, causes extrinsic apoptosis. May be used as a chemokine.

diabetes mellitus

This 5HGTT curve indicates:

glucose not administered

This 5HGTT curve indicates:

hyperglycemia

This 5HGTT curve indicates:

normal carbohydrate metabolism

This 5HGTT curve indicates:

technician error

This 5HGTT curve indicates:

pO2

This abbreviation refers to the amount of oxygen bound to hemoglobin:

pCO2

This abbreviation refers to the partial pressure of dissolved carbon dioxide gas:

heparin

This anticoagulant is acceptable for collection of glucose, but should be spun down immediately for best results:

Rh

This blood group antigen is responsible for most cases of HDN:

CSF

This body fluid does not require processing and should be run STAT:

cal

This calibrator is used for the low end of the linear range:

pyelonephritis

This cause of intra-renal azotemia is caused by infection of the kidney:

interstitial nephritis

This cause of intra-renal azotemia is caused by toxins, certain infectious diseases, or hypoxia:

glomerulonephritis

This cause of intra-renal azotemia results from inflammation of the renal glomeruli:

acute tubular necrosis

This cause of intra-renal azotemia usually results from toxins or hypoxic damage to the kidneys:

2,3-DPG

This chemical is produced by the Rapoport-Leubering shunt:

indirect

This clinical term is also used to describe unconjugated bilirubin:

cushing's syndrome

This condition is caused by a cortisol secreting tumor:

thyrotoxicosis

This condition is caused by a thyroid tumor that secretes thyroid hormones:

cushing's disease

This condition is caused by an ACTH secreting tumor:

Conn's disease

This condition is caused by the growth of an aldosterone secreting tumor:

addison's disease

This condition is caused by underproduction of adrenal hormones, particularly cortisol:

dubin-johnson syndrome

This condition is characterized by hepatocytes that are not efficient at secreting conjugated bilirubin into the biliary tract, leading to obstructive jaundice:

gouty arthritis

This condition is characterized by high levels of uric acid built up in joints, causing painful crystals to form:

primary gout

This condition is characterized by hyperuricemia associated with an inborn error of metabolism:

hypothyroid disease

This condition is characterized by underproduction of thyroid hormones:

pregnancy

This condition is considered normal and non-pathogenic but does cause elevated glucose levels due to hormones such as HPL and estriol:

iron deficiency anemia

This condition is correlated with low iron and high TIBC:

diabetes insipidus

This condition is not related to carbohydrate metabolism and is the result of a deficiency of ADH but has some similar symptoms to DM:

hyperinsulinemia

This condition is typically caused by insulin-secreting tumors of the pancreas:

metabolic alkalosis

This condition results from buildup of ammonia in the bloodstream:

hereditary galactosemia

This condition results from failure of the isomerase enzyme responsible for converting galactose into glucose:

acromegaly

This condition results in enlargements of only the flat woven bones such as the jaw:

jaundice

This condition results when excess bilirubin builds up in the body, resulting in a yellow tinge to the skin and eyes:

gigantism

This condition, which affects the entire body, results in enlargements of all the bones:

hashimoto's disease

This disease is caused by autoimmune destruction of the thyroid:

msds

This document provides handling, storage, reactivity, and toxicity information on a given chemical and must be available 24/7 to all employees.

magnesium

This element must be added in large quantities to reagents used in the hexokinase reaction:

phospho-enol pyruvate carboxylase

This enzymatic reaction may be used to assay bicarbonate and uses oxidation of NADH:

aldolase

This enzyme breaks hexose sugars into two 3-carbon sugars during glycolysis:

albumin

This globular protein binds unconjugated bilirubin to solubilize it.

cortisol

This hormone acts slowly to raise blood sugar by stimulating glycogenolysis:

insulin

This hormone lowers blood glucose by facilitating the entrance of glucose into cells:

glucagon

This hormone produced by the pancreas raises blood glucose by stimulating the liver to break down glycogen stores:

growth hormone

This hormone raises blood sugar solely by gluconeogenesis:

human placental lactogen

This hormone, produced primarily by the fetus, acts to raise blood sugar by glycogenolysis in pregnant women:

standard deviation index

This is a measure of how well a lab might do on their next proficiency testing cycle:

flint glass

This is a proprietary brand of soda lime glass:

levey-jennings

This is a(n) ______ wall chart.

shewhart

This is a(n) ______ wall chart.

competency testing

This is mandated by CLIA and tests individual technologists:

proficiency testing

This is mandated by CLIA and tests the entire lab including instruments, personnel, etc:

oxalate/fluoride

This is the anticoagulant of choice for collecting glucose and lactate samples:

white blood cell casts in urine

This is the characteristic finding of pyelonephritis:

beta-hydroxybutric acid

This ketone body specifically is responsible for lowering blood pH in ketoacidosis:

o-toluidine

This methodology can be used to test for any sugar:

vapor pressure depression

This methodology evaporates the specimen to measure osmolality:

potentiometric assay

This methodology involves measuring the electrical potential build-up between the specimen sample and a known standard:

reflectance photometry

This methodology is effectively the opposite of colorimetry, instead measuring the amount of light reflected by the specimen:

nephelometry

This methodology is similar to tubidimetry in that it measures light scatter, but is much more sensitive:

turbidimetric analysis

This methodology measures light scatter rather than colorimetric absorbance to determine the concentration of an analyte:

biochemiluminescence

This methodology produces a scintillation reaction using an enzyme:

freezing point depression

This methodology super-cools the specimen to measure osmolality:

colloid osmotic pressure

This methodology uses a semipermeable membrane to measure osmolality:

fluorimetry

This methodology uses either naturally fluorescent analytes or fluorescent-labeled probes to measure concentration:

chemiluminescence

This methodology uses scintillation to measure the concentration of an analyte:

polarographic method

This methodology uses the same basic concept as the glucose oxidase reaction but measures the pressure of the oxygen created:

urea

This molecule is the end product of protein metabolism in healthy individuals:

uric acid

This molecule is the end product of purine catabolism:

creatine

This molecule is used within muscle tissue to store energy by phosphorylation:

creatinine hydrolase

This multi-step reaction involves converting creatinine back to creatine and eventually producing NAD from NADH.

epinephrine

This neurotransmitter hormone acts quickly to raise blood sugar by stimulating glycogenolysis:

standard deviation

This number is a measure of the variance of the values in the dataset in terms of distance from the mean:

mean

This number is the average value of a dataset:

photomultiplier

This part of a spectrophotometer collects light transmitted through the cuvette and amplifies it:

readout device

This part of the spectrophotometer converts the light amplified by the photomultiplier into a number and displays it:

primary exit slit

This part of the spectrophotometer limits the light hitting the specimen to a small range of λ based on what is needed for the test:

hemolytic disease of the newborn

This potentially fatal condition results from blood type incompatibility between mother and baby, in which antibodies produced by the mother attack the baby's blood:

glycogenolysis

This process involves breaking down glycogen in the liver into glucose molecules:

spectrophotometric linearity

This property is tested by using increasingly absorbant filters to ensure that absorbance of a standard known to be linear shows as linear:

westgard rules

This set of criteria is the industry standard for determining whether to pass or fail QC:

basement membrane

This specific structure of the glomerulus is injured by glomerulonephritis:

serum

This specimen is technically acceptable for collection of glucose but not ideal because of the long processing time:

type I

This subtype of Crigler-Najjar Syndrome is a complete lack of enzymatic activity and results in death from kernicterus shortly after birth:

type II

This subtype of Crigler-Najjar Syndrome represents decreased enzymatic activity and results in elevated bilirubin:

xylose

This sugar is not typically consumed in the diet and cannot be catabolized for energy, though the intestine can absorb it:

glucose tolerance test

This test involves administering a glucose solution and observing changes in blood glucose:

xylose tolerance test

This test involves giving an oral load of xylose and then measuring all urine collected over the next 5 hours for xylose:

osmolality

This test is a measure of osmotic pressure:

pO2(50)

This test is a measure of oxygen binding affinity:

fructosamine

This test is similar in methodology to glycosylated hemoglobin but measures the amount of glucose bound to albumin:

epinephrine tolerance test

This test is used to diagnose VonGierke's disease:

bichromatic analysis

This test methodology involves measuring the concentration of an analyte by taking absorbance readings at multiple λ:

urine dipstick

This test, performed at virtually every laboratory, uses reflectance photometry:

5HGTT

This tolerance test is the "gold standard" for diagnosing dysfunction of carbohydrate metabolism:

ion specific electrode

This type of assay binds ammonia to glass using specific antibiotics then measures the change in electric potential:

immunoassay

This type of assay can be used to quantify cereuloplasmin:

water blank

This type of blank may be used when the desired reagent does not absorb light:

reagent blank

This type of blank must be used when the desired reagent does absorb light:

type II

This type of diabetes mellitus was formerly called adult-onset or non-insulin dependent diabetes:

type I

This type of diabetes mellitus was formerly called juvenile or insulin-dependent diabetes:

isomerase

This type of enzyme is capable of converting any hexose sugar into glucose so it may be absorbed:

milipore

This type of filter is often used in combination with deionizing columns to remove large particles such as algae:

soda lime

This type of glass can leach minerals such as Na, K, Mg, Ca, and phosphates:

borosilicate

This type of glass is used to make laboratory glassware because it does not leech minerals which would affect chemistry assays:

fetal

This type of hemoglobin has an exceptionally high binding affinity for oxygen:

polystyrene

This type of lab plastic is hard and brittle:

polypropylene

This type of lab plastic is soft and flexible:

prism

This type of monochromator is ineffective and rather crude at splitting light, but provides a good example of the function of a monochromator:

interference filter

This type of monochromator is not a true monochromator in that it does not split light, merely limit the incident light striking it to several selected λ:

diffraction grating

This type of monochromator is the most effective at splitting light and consists of a mirrored surface with many small etches:

muscle

This type of tissue stores glycogen, but for local use only:

distilled

This type of water is produced by boiling water and capturing the purified water vapor:

reverse osmosis

This type of water is produced by forcing pressurized water through special selective membrane filters:

tap

This type of water is suitable for handwashing and drinking, but not for use in laboratory functions:

diacetyl monoxime-thisemicarbazide

This urea assay was used in the 50's-70's, is very stable, and is highly reproducible:

serum potassium

This value is always elevated in acute glomerulonephritis:

o2 saturation

This value is the amount of oxygen in the blood compared to the total carrying capacity of blood, and is expressed as a percent:

TBIL

This vitros slide is not FDA approved and should never be used on neonates:

jaffe method

This was the gold standard creatinine assay before enzymatic methods became available:

2 hour

Though the 5HGTT is considered the gold standard, a ____ GTT is far more commonly ordered.

H2SO4

To be measured by an electrode, CO2 must first be converted to gas by reacting it with:

primary-secondary

To calculate the actual absorbance in a bichromatic analysis, this equation should be used:

half

To check the bandpass of an instrument, increase and decrease the λ until absorbance is _____ of what it is at the peak.

multiply by 2.14

To convert BUN to urea:

multiply by 0.03

To convert mmHg of CO2 to mmol/L carbonic acid:

divide by 2.14

To convert urea to BUN:

What is the purpose of the Direct Antiglobulin Test (DAT)

To detect in vivo sensitized red blood cells

2

To determine a confidence interval for QC, the control material must be assayed many times, and after outliers are removed the confidence interval is ∓___σ around the mean.

361 +- 1nm

To ensure that an instrument is giving the proper λ, a standard of holmium oxide should be tested and give a peak at:

diluted volume, original volume

To find the dilution factor, divide ______ by ______.

Why is it important to check for previous records

To know if a clinically significant antibody was identified

-7

To perform FPD osmometry, the sample is super-cooled to ___ degrees C then agitated to induce crystalization and warmed.

condenses, all condensation is gone

To perform VPD osmometry, the sample is inoculated on filter paper and placed in a sealed container, cooled until it _______, then warmed until ______.

30, 45

To perform an epinephrine tolerance test, epinephrine is injected and glucose levels are checked at ____ and ____ minutes.

high carbohydrate, fast

To prepare for a GTT, the patient should consume a(n) _____ diet 3 days prior to the test then _____ for the last 12 hours.

For both the DAT and the IAT the cell button should be washed at least three times prior to adding AHG. Why

To remove globulins from the serum/plasma surrounding the cells

Adult Bilirubin

Total 0.2 to 1.0 mg/dL Direct 0.0 to 0.4 mg/dL

bicarbonate, carbonic acid, carbon dioxide gas, carbonates

Total CO2 is the sum of these forms of carbon dioxide:

direct, indirect

Total bilirubin = _____ + ______.

800-1800

Total daily urinary excretion of creatinine for males should be ___-___ mg/day.

33

Transferrin "prefers" to be about ___% saturated.

If a patient receives a bag of single donor platelets and his platelet count does not increase (In fact, it sometimes decreases.), we say he may be refractory to platelets. How do we attempt to overcome this refractoriness

Transfuse HLA matched platelets

Which of the following is an iatrogenic transfusion reaction(If you want to get technical, I guess you could say they are all iatrogenic, but your text does focus on one.)

Transfusion Associated Circulatory Overload

lactate

Trauma and shock cause blood to move away from peripheral tissues and into the core of the body. One could measure this by testing this analyte in peripheral blood:

low

Traumatic conditions such as burns or acute tubular necrosis will cause sodium levels to be:

Immature T Cells

Travel to the Thymus, where they rearrange their DNA for a random receptor, express low amounts of both CD4 and CD8.

Intrinsic Apoptosis

Triggered via cytoplasm, activating Caspase 9.

Extrinsic Apoptosis

Triggered via membrane FAS receptor, activating intracellular Caspase 8.

Coffin-lid shaped crystals

Tripple phosphate stones

urine, csf protein

Turbidemetry is commonly used to measure:

urea, glucose

Two examples of "true molecules", which are not removed by deionization and may skew assay results are:

IgG Structure

Two heavy chains (four loops each, VH and CH1-3) and two light chains (two loops each, VL and CL), each with a single variable loop at the binding tips, secreted by activated B-Cells.

disaccharide

Two simple sugars bonded together is termed a(n):

virus

Type I diabetes is caused by autoimmune destruction of the pancreas, likely triggered by a(n):

complete insulin deficiency

Type I diabetes is characterized by:

10

Type I reagent water has a resistance of ≥_____ mOms/cm.

insulin resistance or dysfunction

Type II diabetes is characterized by:

2

Type II reagent water has a resistance of ≥_____ mOms/cm.

0.1

Type III reagent water has a resistance of ≥_____ mOms/cm.

0.1, 5-7

Typically, every ___ mg/dL increase in serum creatinine represents ___-___% kidney function loss.

increases

Typically, pO2 decreases as pCO2 _______, and vice versa.

unbound iron binding capacity

UIBC stands for:

pharmaceutical

USP/NP grade is also known as _____ grade.

Nitrites in the urine is indicative of

UTI except with Staph. Saprophyticus

Bacteria in urine

UTI or contamination

White Blood Cells in the urine is indicative of

UTI, glomerulonephritis, vaginal contamination

insoluble

Unconjugated bilirubin is ______ in water.

Adult Thymus

Undergoes atrophy after puberty, degenerates and fills with adipose -> reduced levels of blood Thymosin, becomes nonfunctional with old age.

Barrel-shaped crystals in urine

Uric acid

renally

Uric acid is excreted:

crystalize

Uric acid is not a particularly toxic compound, but is not very soluble in water and has a tendency to ______ in high concentrations.

2.6-6.0

Uric acid normal range for females is ___-___ mg/dL.

3.5-7.2

Uric acid normal range for males is ___-___ mg/dL.

Insect Immunity

Use TLR and antibacterial peptides only. Compensate with hyper-reproduction strategy.

ab T-Cell Receptors

Use a transmembrane-bound linear receptor, one alpha chain and one beta chain, with adjacent signal transducer CD3 complex. Recognize short, linear peptides as presented to them by other cells via MHC.

cutoff filter

Use of a(n) _____ can detect the creation of stray light within a spectrophotometer.

B-Cell Receptors

Use transmembrane-bound IgM antibodies, two each of heavy and light chains, both with constant and variable regions, two identical binding sites at tip. Signal transduction molecules adjacent. Only binds intact 3-D antigen.

IFNg

Used by TH1 cells to inhibit differentiation of type TH2. Also stimulates Macrophages, TH1 development, antibody class switching, and general MHC expression.

IL10

Used by TH2 cells to inhibit differentiation of type TH1.

MHC1B

Used by cells to present Stress Proteins, recognized by gh T-Cells and NK Cells for induced apoptosis.

Lactoferrin

Used in Neutrophil lysosomes, acts to chelate iron and interrupt bacterial metabolism.

SNAP Test

Used to detect FIV antibodies. Serum Ab sample is mixed with conjugated Ag, added to matrix/well with fixed Ag, snap releases wash and substrate. Vaccine and passive transfer cause false positives.

MIC Receptors

Used to present stress proteins by affected cells, bound by NK and gd T Cells for induced apoptosis.

Immuno-Affinity Chromatography

Used to purify antigens or antibodies, covalently bound to insoluble beads, mixed solution run through to retain compound of interest, followed by pH or ion change to reverse binding and elute pure compound. Can also bind pathogen to purify all antibodies, then bind an antibody to retain a subset of antibodies.

Helper T-Cell

Uses its CD4 to bind a cell's MHC2, antigen presented is interpreted as belonging to a pathogen, triggers proliferation into a subset type based on signal.

specific standards

Using ______ allows use of the o-toluidine test for specific sugars.

Antitoxin

Vaccine made with antibodies targeting a specific toxin, imparts passive immunity.

poikilocytosis

Variation in the shape of red blood cells

Bicarbonate (mEq/L = mMol/L)

Venous 22 to 29 mMol/L Arterial 21 to 28 mMol/L Capillary 21 to 28 mMol/L

Total CO2 (mEq/L = mMol/L)

Venous 23 to 30 mMol/L Arterial 22 to 29 mMol/L Capillary 22 to 29 mMol/L

Blood pH

Venous 7.32 to 7.42 Arterial 7.35 to 7.45 Capillary 7.35 to 7.45

Basophil

Very rare with an unknown half-life, polymorphic nucleus, contains Histamine for parasites and allergic reactions.

Adaptive Cytokines

Via activated T Cells. Regulate lymphocyte activation, enhance Innate Immunity, and may physically bind and inactivate viruses or cause MHC1 upregulation for CD8-induced apoptosis of infected cells.

Macrophage Activation

Via cytokines, may be innate or adaptive, but cell-mediated is best. Can be reciprocally activated by T-Cells using cytokines. Once active, can activate others.

Eosinophil Activation

Via surface receptors for IgE, Macrophages releasing chemotactics and correct PAMPs, and Cytokines from activated Helper T-Cells.

hemolysis

What is shown here?

icterus

What is shown here?

lipemia

What is shown here?

-CD45

What is the "leukocyte common antigen" present on all WBCs and WBC progenitors?

-osteoblast -have a characteristic extruding nucleus with numerous nucleoli -large, oval shape with feathery edges and deeply basophilic cytoplasm

What type of cell(s) is this picture demonstrating?

-osteoclast -have numerous well-separated and uniformly-sized nuclei

What type of cell(s) is this picture demonstrating?

-CFU-GEMM: form myeloid, erythroid, and megakaryocytic lines -CFU-GM: form granulocytes and macrophages -CFU-G: form only granulocytes -CFU-E: form erythrocytes -CFU-Mk: form megakaryocytes

What types of cell(s) differentiate from the following cell lines: -CFU-GEMM -CFU-GM -CFU-G -CFU-E -CFU-Mk

bun/creatinine ratio

When BUN and/or creatinine are elevated, this can be calculated to provide more information about the cause:

carbonic acid

When CO2 gas enters a CO2 electrode, it is dissolved into a buffer solution and becomes:

specimen blank

When a specimen has color, such as due to hemolysis, icterus, or lipemia, a(n) _____ can be used to cancel it out.

18

When calculating osmolality, glucose (in mg/dL) must be divided by:

1.86

When calculating osmolality, sodium (in mmol/L) must be multiplied by:

repeats back

When calling a panic value, it is critical that the provider __________ the information.

7nm

When checking bandpass, the peak absorbance reading is at 361nm and absorbance is halved at 354nm and 368nm. What is the bandpass?

creatinine

When creatine passes through a cell membrane, it converts spontaneously to:

absorbance curve

When creating a new test, to find the complementary λ a(n) _____ should be made.

evaporate

When exposed to air, some ammonium will reconvert to ammonia and:

hemosiderin

When iron is high, _____ is used as an overflow storage protein.

cathode

When measuring anions, the _____ is usually sealed and used as a standard.

blank

When performing a two-point kinetic assay, the first absorbance reading is essentially a(n):

once

When performing an end-point assay, the absorbance is taken:

magnesium carbonate

When running a TIBC assay, excess iron is chelated by:

respiratory alkalosis

When the condition is prolonged, the body will attempt to correct metabolic acidosis with:

metabolic alkalosis

When the condition is prolonged, the body will attempt to correct respiratory acidosis with:

trinder

When the uricase reaction is used, this additional type of reaction is used to quantify uric acid based on the amount of peroxide produced:

When is it permissible to do only an immediate spin crossmatch

When there is no history of a clinically significant antibody and no clinically significant antibody is detected in the current sample.

Bu + Bc

When using vitros instruments, rather than using the TBIL slide to determine bilirubin in neonates, this equation should be used instead:

point of care testing

Whole blood for glucose analysis is typically used for:

high fasting blood glucose

Why was this patient not a good candidate for a 5HGTT?

A

You made this absorbance curve while inventing an assay which measures the caffeine concentration of an MLS student. Which curve represents an ideal range of λs to use for your assay?

24 hour excretion of urea in grams per day

[Urine BUN mg/dL x 2.14 divided by 100 (to convert to g/L)] X Liters 24 hour urine volume

5

___% of renal function must be lost before creatinine becomes elevated.

30-60

___-___% of patients with gestational diabetes will go on to develop full type II diabetes at some point in their life.

hemolytic

____ jaundice is the result of conditions which cause excess bilirubin production, such as transfusion reactions and certain types of anemias.

hydrogen/deuterium, mercury arc

____ or _____ bulbs provide a light source at λ=180-380 nm.

2

____ or more consecutive control failures always indicates a problem.

4.5

____% of the time, QC will fall outside 2 standard deviations from the mean because of normal random error.

all

_____ dissolved substances exert osmotic pressure.

pure water

_____ does not conduct electricity.

bandpass

_____ is defined as the range of λ that are actually displayed when a certain λ is requested from the spectrophotometer.

contamination

______ should also be suspected if many analytes are decreased, especially unexpectedly.

heavy metal

________ poisoning can cause pre-renal azotemia.

minutes

pH changes due to retention or reduction of CO2 by respiratory means occur on the time scale of:

hours to days

pH changes due to retention or reduction of bicarbonate by metabolic means occur on the time scale of:

unexpected low O2sat

pO2(50) is usually performed to follow up:

-probably due to combination of homozygous mild β+- and α-thalassemia -reduced α-chain precipitation and more effective erythropoesis, less hemolysis -symptomatic with mild anemia, don't require regular transfusions -can also be due to inheritance of hereditary persistence of Hb F which decreases number of free excess α chains -patients may have splenomegaly, bone deformities, leg ulcers, gallstones, and infections -may present with iron overload due to excessive iron absorption from underlying dys-erythropoesis

thalassemia intermedia

-homozygous carrier of β globin mutation; require blood transfusions -usually present during first year of life with severe symptoms: failure to thrive, recurrent bacterial infections, severe anemia, extramedullary hematopoesis, hepatosplenomegaly, bone marrow expansion, long bone and cranial bone distortions -marked shortening of RBC lifespan, and ineffective erythropoesis, folate deficiency -hairy appearance of bones on radiograph due to BM expansion -increase in Hb A2 and Hb F; depending on the type (β+ or 0), may still have Hb A

thalassemia major

-arrows indicate nucleated RBCs in the peripheral blood smear (leukoerythroblastosis) -asterisk shows a macrocytic RBC with nuclear remnants or Howell-Jolly Bodies -these are indicative of hemolytic anemia, hypoxia, and myeloproliferative disorders -erythroblastosis is caused by premature release of immature cells from the BM; occurs when there is a large increase in RBC production to overcome chronic or severe loss

what do the arrows indicate? what does the asterisk indicate? what disease are these associated with?

-heinz bodies -particles of denatured hemoglobin attached (usually) to the inner face of RBC membrane -drugs that induce oxidative denaturation of Hb will produce them, as well as G-6-P deficiency

what do these cells demonstrate? what are these aggregates made of? what can cause these?

-often caused by gene deletions -normally, there are two copies of the α globin chain on each gene (4 per person) of chromosome 16 -deletion of one (α+) or both copies (α0) may occur

α-thalassemias

-caused mostly by point mutations rather than gene deletions; can be defects in transcription, translation, truncation, etc. -either no normal β chain produced (β0), or production is very reduced (β+) -a cause of hemolytic anemia- excess of α chains which precipitate in erythroblasts and RBCs causing ineffective erythropoesis and hemolysis -excess α chains combine with β, ɣ, and δ chains, increasing quantities of Hb A2, Hb F, and small amounts of Hb A -heme synthesis is unaffected -symptoms range from symptomless to severe

β-thalassemias

nanometers

λ is measured in:


Related study sets

Chp. 19 Section 19.2-19.5 Dynamic Study Molecule

View Set

Ch 2 Test (Becoming a Professional)

View Set

Chapter 13 Urinary System Diseases and Disorders

View Set

EXAM 2 : CHAPTERS 17 & 18 : PRACTICE EXAM

View Set

Anthropology Unit 6: Material Culture - How Humans Are Handy

View Set